WorldWideScience

Sample records for mathematical word problems

  1. Mathematical Tasks without Words and Word Problems: Perceptions of Reluctant Problem Solvers

    Science.gov (United States)

    Holbert, Sydney Margaret

    2013-01-01

    This qualitative research study used a multiple, holistic case study approach (Yin, 2009) to explore the perceptions of reluctant problem solvers related to mathematical tasks without words and word problems. Participants were given a choice of working a mathematical task without words or a word problem during four problem-solving sessions. Data…

  2. Graphic Organizer in Action: Solving Secondary Mathematics Word Problems

    Directory of Open Access Journals (Sweden)

    Khoo Jia Sian

    2016-09-01

    Full Text Available Mathematics word problems are one of the most challenging topics to learn and teach in secondary schools. This is especially the case in countries where English is not the first language for the majority of the people, such as in Brunei Darussalam. Researchers proclaimed that limited language proficiency and limited Mathematics strategies are the possible causes to this problem. However, whatever the reason is behind difficulties students face in solving Mathematical word problems, it is perhaps the teaching and learning of the Mathematics that need to be modified. For example, the use of four-square-and-a-diamond graphic organizer that infuses model drawing skill; and Polya’s problem solving principles, to solve Mathematical word problems may be some of the strategies that can help in improving students’ word problem solving skills. This study, through quantitative analysis found that the use of graphic organizer improved students’ performance in terms of Mathematical knowledge, Mathematical strategy and Mathematical explanation in solving word problems. Further qualitative analysis revealed that the use of graphic organizer boosted students’ confidence level and positive attitudes towards solving word problems.Keywords: Word Problems, Graphic Organizer, Algebra, Action Research, Secondary School Mathematics DOI: http://dx.doi.org/10.22342/jme.7.2.3546.83-90

  3. The Association between Mathematical Word Problems and Reading Comprehension

    Science.gov (United States)

    Vilenius-Tuohimaa, Piia Maria; Aunola, Kaisa; Nurmi, Jari-Erik

    2008-01-01

    This study aimed to investigate the interplay between mathematical word problem skills and reading comprehension. The participants were 225 children aged 9-10 (Grade 4). The children's text comprehension and mathematical word problem-solving performance was tested. Technical reading skills were investigated in order to categorise participants as…

  4. Students’ Representation in Mathematical Word Problem-Solving: Exploring Students’ Self-efficacy

    Science.gov (United States)

    Sahendra, A.; Budiarto, M. T.; Fuad, Y.

    2018-01-01

    This descriptive qualitative research aims at investigating student represented in mathematical word problem solving based on self-efficacy. The research subjects are two eighth graders at a school in Surabaya with equal mathematical ability consisting of two female students with high and low self-efficacy. The subjects were chosen based on the results of test of mathematical ability, documentation of the result of middle test in even semester of 2016/2017 academic year, and results of questionnaire of mathematics word problem in terms of self-efficacy scale. The selected students were asked to do mathematical word problem solving and be interviewed. The result of this study shows that students with high self-efficacy tend to use multiple representations of sketches and mathematical models, whereas students with low self-efficacy tend to use single representation of sketches or mathematical models only in mathematical word problem-solving. This study emphasizes that teachers should pay attention of student’s representation as a consideration of designing innovative learning in order to increase the self-efficacy of each student to achieve maximum mathematical achievement although it still requires adjustment to the school situation and condition.

  5. The Motivation of Secondary School Students in Mathematical Word Problem Solving

    Science.gov (United States)

    Gasco, Javier; Villarroel, Jose-Domingo

    2014-01-01

    Introduction: Motivation is an important factor in the learning of mathematics. Within this area of education, word problem solving is central in most mathematics curricula of Secondary School. The objective of this research is to detect the differences in motivation in terms of the strategies used to solve word problems. Method: It analyzed the…

  6. Language and modeling word problems in mathematics among bilinguals.

    Science.gov (United States)

    Bernardo, Allan B I

    2005-09-01

    The study was conducted to determine whether the language of math word problems would affect how Filipino-English bilingual problem solvers would model the structure of these word problems. Modeling the problem structure was studied using the problem-completion paradigm, which involves presenting problems without the question. The paradigm assumes that problem solvers can infer the appropriate question of a word problem if they correctly grasp its problem structure. Arithmetic word problems in Filipino and English were given to bilingual students, some of whom had Filipino as a first language and others who had English as a first language. The problem-completion data and solution data showed similar results. The language of the problem had no effect on problem-structure modeling. The results were discussed in relation to a more circumscribed view about the role of language in word problem solving among bilinguals. In particular, the results of the present study showed that linguistic factors do not affect the more mathematically abstract components of word problem solving, although they may affect the other components such as those related to reading comprehension and understanding.

  7. The effects of stating problems in bilingual students' first and second languages on solving mathematical word problems.

    Science.gov (United States)

    Bernardo, Allan B I; Calleja, Marissa O

    2005-03-01

    Researchers have suggested that among bilinguals, solving word problems in mathematics is influenced by linguistic factors (K. Durkin & B. Shire, 1991; L. Verschaffel, B. Greer, & E. De Corte, 2000). Others have suggested that students exhibit a strong tendency to exclude real-world constraints in solving mathematics word problems (L. Verschaffel, E. De Corte, & S. Lasure, 1994). In the present study, the authors explored the effects of stating word problems in either Filipino or English on how Filipino-English bilingual students solved word problems in which the solution required the application of real-world knowledge. The authors asked bilingual students to solve word problems in either their first or second language. For some of the word problems, real-life constraints prevented straightforward application of mathematical procedures. The authors analyzed the students' solutions to determine whether the language of the word problems affected the tendency to apply real-life constraints in the solution. Results showed that the bilingual students (a) rarely considered real-life constraints in their solutions, (b) were more successful in understanding and solving word problems that were stated in their first language, and (c) were more likely to experience failure in finding a solution to problems stated in their second language. The results are discussed in terms of the relationship between linguistic and mathematical problem-solving processes among bilinguals.

  8. Helping Students with Emotional and Behavioral Disorders Solve Mathematics Word Problems

    Science.gov (United States)

    Alter, Peter

    2012-01-01

    The author presents a strategy for helping students with emotional and behavioral disorders become more proficient at solving math word problems. Math word problems require students to go beyond simple computation in mathematics (e.g., adding, subtracting, multiplying, and dividing) and use higher level reasoning that includes recognizing relevant…

  9. Strategies of solving arithmetic word problems in students with learning difficulties in mathematics

    OpenAIRE

    Kalan, Marko

    2015-01-01

    Problem solving as an important skill is, beside arithmetic, measure and algebra, included in standards of school mathematics (National Council of Teachers of Mathematics) (NCTM, 2000) and needed as a necessary skill for successfulness in science, technology, engineering and mathematics (STEM) (National Mathematics Advisory Panel, 2008). Since solving of human problems is connected to the real life, the arithmetic word problems (in short AWP) are an important kind of mathematics tasks in scho...

  10. Working memory components as predictors of children's mathematical word problem solving.

    Science.gov (United States)

    Zheng, Xinhua; Swanson, H Lee; Marcoulides, George A

    2011-12-01

    This study determined the working memory (WM) components (executive, phonological loop, and visual-spatial sketchpad) that best predicted mathematical word problem-solving accuracy of elementary school children in Grades 2, 3, and 4 (N=310). A battery of tests was administered to assess problem-solving accuracy, problem-solving processes, WM, reading, and math calculation. Structural equation modeling analyses indicated that (a) all three WM components significantly predicted problem-solving accuracy, (b) reading skills and calculation proficiency mediated the predictive effects of the central executive system and the phonological loop on solution accuracy, and (c) academic mediators failed to moderate the relationship between the visual-spatial sketchpad and solution accuracy. The results support the notion that all components of WM play a major role in predicting problem-solving accuracy, but basic skills acquired in specific academic domains (reading and math) can compensate for some of the influence of WM on children's mathematical word problem solving. Copyright © 2011 Elsevier Inc. All rights reserved.

  11. Effect of Personalisation of Instruction on Students’ Motivation to learn Mathematics Word Problems in Nigeria

    OpenAIRE

    Adeneye Olarewaju Awofala

    2016-01-01

    This study investigated the effect of personalisation of instruction on the motivation to learn mathematics word problems of 450 senior secondary students in Nigeria within the blueprint of quasi-experimental research of Solomon Four non-equivalent control group design. It also examined the influence of gender on motivation to learn mathematics word problems and personalisation was accomplished by incorporating selected information with students’ personal preferences into their mathematics wo...

  12. How Can One Learn Mathematical Word Problems in a Second Language? A Cognitive Load Perspective

    Science.gov (United States)

    Moussa-Inaty, Jase; Causapin, Mark; Groombridge, Timothy

    2015-01-01

    Language may ordinarily account for difficulties in solving word problems and this is particularly true if mathematical word problems are taught in a language other than one's native language. Research into cognitive load may offer a clear theoretical framework when investigating word problems because memory, specifically working memory, plays a…

  13. Contribution of Equal-Sign Instruction beyond Word-Problem Tutoring for Third-Grade Students with Mathematics Difficulty.

    Science.gov (United States)

    Powell, Sarah R; Fuchs, Lynn S

    2010-05-01

    Elementary school students often misinterpret the equal sign (=) as an operational rather than a relational symbol. Such misunderstanding is problematic because solving equations with missing numbers may be important for higher-order mathematics skills including word problems. Research indicates equal-sign instruction can alter how typically-developing students use the equal sign, but no study has examined effects for students with mathematics difficulty (MD) or how equal-sign instruction contributes to word-problem skill for students with or without MD. The present study assessed the efficacy of equal-sign instruction within word-problem tutoring. Third-grade students with MD (n = 80) were assigned to word-problem tutoring, word-problem tutoring plus equal-sign instruction (combined) tutoring, or no-tutoring control. Combined tutoring produced better improvement on equal sign tasks and open equations compared to the other 2 conditions. On certain forms of word problems, combined tutoring but not word-problem tutoring alone produced better improvement than control. When compared at posttest to 3(rd)-grade students without MD on equal sign tasks and open equations, only combined tutoring students with MD performed comparably.

  14. Students' Mathematics Word Problem-Solving Achievement in a Computer-Based Story

    Science.gov (United States)

    Gunbas, N.

    2015-01-01

    The purpose of this study was to investigate the effect of a computer-based story, which was designed in anchored instruction framework, on sixth-grade students' mathematics word problem-solving achievement. Problems were embedded in a story presented on a computer as computer story, and then compared with the paper-based version of the same story…

  15. Are middle school mathematics teachers able to solve word problems without using variable?

    Science.gov (United States)

    Gökkurt Özdemir, Burçin; Erdem, Emrullah; Örnek, Tuğba; Soylu, Yasin

    2018-01-01

    Many people consider problem solving as a complex process in which variables such as x, y are used. Problems may not be solved by only using 'variable.' Problem solving can be rationalized and made easier using practical strategies. When especially the development of children at younger ages is considered, it is obvious that mathematics teachers should solve problems through concrete processes. In this context, middle school mathematics teachers' skills to solve word problems without using variables were examined in the current study. Through the case study method, this study was conducted with 60 middle school mathematics teachers who have different professional experiences in five provinces in Turkey. A test consisting of five open-ended word problems was used as the data collection tool. The content analysis technique was used to analyze the data. As a result of the analysis, it was seen that the most of the teachers used trial-and-error strategy or area model as the solution strategy. On the other hand, the teachers who solved the problems using variables such as x, a, n or symbols such as Δ, □, ○, * and who also felt into error by considering these solutions as without variable were also seen in the study.

  16. Secondary School Pre-Service Mathematics Teachers' Content Knowledge of Algebraic Word Problem in Nigeria

    Science.gov (United States)

    Usman, Ahmed Ibrahim

    2015-01-01

    Knowledge and understanding of mathematical operations serves as a pre-reequisite for the successful translation of algebraic word problems. This study explored pre-service teachers' ability to recognize mathematical operations as well as use of those capabilities in constructing algebraic expressions, equations, and their solutions. The outcome…

  17. Teacher-Student Interaction in Joint Word Problem Solving. The Role of Situational and Mathematical Knowledge in Mainstream Classrooms

    Science.gov (United States)

    Rosales, Javier; Vicente, Santiago; Chamoso, Jose M.; Munez, David; Orrantia, Josetxu

    2012-01-01

    Word problem solving involves the construction of two different mental representations, namely, mathematical and situational. Although educational research in word problem solving has documented different kinds of instruction at these levels, less is known about how both representational levels are evoked during word problem solving in day-to-day…

  18. Effects of Mathematics Anxiety and Mathematical Metacognition on Word Problem Solving in Children with and without Mathematical Learning Difficulties

    Science.gov (United States)

    Lai, Yinghui; Zhu, Xiaoshuang; Chen, Yinghe; Li, Yanjun

    2015-01-01

    Mathematics is one of the most objective, logical, and practical academic disciplines. Yet, in addition to cognitive skills, mathematical problem solving also involves affective factors. In the current study, we first investigated effects of mathematics anxiety (MA) and mathematical metacognition on word problem solving (WPS). We tested 224 children (116 boys, M = 10.15 years old, SD = 0.56) with the Mathematics Anxiety Scale for Children, the Chinese Revised-edition Questionnaire of Pupil’s Metacognitive Ability in Mathematics, and WPS tasks. The results indicated that mathematical metacognition mediated the effect of MA on WPS after controlling for IQ. Second, we divided the children into four mathematics achievement groups including high achieving (HA), typical achieving (TA), low achieving (LA), and mathematical learning difficulty (MLD). Because mathematical metacognition and MA predicted mathematics achievement, we compared group differences in metacognition and MA with IQ partialled out. The results showed that children with MLD scored lower in self-image and higher in learning mathematics anxiety (LMA) than the TA and HA children, but not in mathematical evaluation anxiety (MEA). MLD children’s LMA was also higher than that of their LA counterparts. These results provide insight into factors that may mediate poor WPS performance which emerges under pressure in mathematics. These results also suggest that the anxiety during learning mathematics should be taken into account in mathematical learning difficulty interventions. PMID:26090806

  19. Effects of Mathematics Anxiety and Mathematical Metacognition on Word Problem Solving in Children with and without Mathematical Learning Difficulties.

    Directory of Open Access Journals (Sweden)

    Yinghui Lai

    Full Text Available Mathematics is one of the most objective, logical, and practical academic disciplines. Yet, in addition to cognitive skills, mathematical problem solving also involves affective factors. In the current study, we first investigated effects of mathematics anxiety (MA and mathematical metacognition on word problem solving (WPS. We tested 224 children (116 boys, M = 10.15 years old, SD = 0.56 with the Mathematics Anxiety Scale for Children, the Chinese Revised-edition Questionnaire of Pupil's Metacognitive Ability in Mathematics, and WPS tasks. The results indicated that mathematical metacognition mediated the effect of MA on WPS after controlling for IQ. Second, we divided the children into four mathematics achievement groups including high achieving (HA, typical achieving (TA, low achieving (LA, and mathematical learning difficulty (MLD. Because mathematical metacognition and MA predicted mathematics achievement, we compared group differences in metacognition and MA with IQ partialled out. The results showed that children with MLD scored lower in self-image and higher in learning mathematics anxiety (LMA than the TA and HA children, but not in mathematical evaluation anxiety (MEA. MLD children's LMA was also higher than that of their LA counterparts. These results provide insight into factors that may mediate poor WPS performance which emerges under pressure in mathematics. These results also suggest that the anxiety during learning mathematics should be taken into account in mathematical learning difficulty interventions.

  20. Effects of Mathematics Anxiety and Mathematical Metacognition on Word Problem Solving in Children with and without Mathematical Learning Difficulties.

    Science.gov (United States)

    Lai, Yinghui; Zhu, Xiaoshuang; Chen, Yinghe; Li, Yanjun

    2015-01-01

    Mathematics is one of the most objective, logical, and practical academic disciplines. Yet, in addition to cognitive skills, mathematical problem solving also involves affective factors. In the current study, we first investigated effects of mathematics anxiety (MA) and mathematical metacognition on word problem solving (WPS). We tested 224 children (116 boys, M = 10.15 years old, SD = 0.56) with the Mathematics Anxiety Scale for Children, the Chinese Revised-edition Questionnaire of Pupil's Metacognitive Ability in Mathematics, and WPS tasks. The results indicated that mathematical metacognition mediated the effect of MA on WPS after controlling for IQ. Second, we divided the children into four mathematics achievement groups including high achieving (HA), typical achieving (TA), low achieving (LA), and mathematical learning difficulty (MLD). Because mathematical metacognition and MA predicted mathematics achievement, we compared group differences in metacognition and MA with IQ partialled out. The results showed that children with MLD scored lower in self-image and higher in learning mathematics anxiety (LMA) than the TA and HA children, but not in mathematical evaluation anxiety (MEA). MLD children's LMA was also higher than that of their LA counterparts. These results provide insight into factors that may mediate poor WPS performance which emerges under pressure in mathematics. These results also suggest that the anxiety during learning mathematics should be taken into account in mathematical learning difficulty interventions.

  1. The Use of a Bar Model Drawing to Teach Word Problem Solving to Students with Mathematics Difficulties

    Science.gov (United States)

    Morin, Lisa L.; Watson, Silvana M. R.; Hester, Peggy; Raver, Sharon

    2017-01-01

    For students with mathematics difficulties (MD), math word problem solving is especially challenging. The purpose of this study was to examine the effects of a problem-solving strategy, bar model drawing, on the mathematical problem-solving skills of students with MD. The study extended previous research that suggested that schematic-based…

  2. Exploring the Learning of Mathematics Word Problems by African Immigrant Early Learners

    Science.gov (United States)

    Mahofa, Ernest; Adendorff, Stanley; Kwenda, Chiwimbiso

    2018-01-01

    The aim of this study was to explore the learning of mathematics word problems by African immigrant early learners in the Western Cape Province of South Africa (SA). Phenomenology was used as the philosophical underpinning for this study and also informed the research method. Purposive sampling methods were used to select 10 African immigrant…

  3. Embedding Number-Combinations Practice Within Word-Problem Tutoring

    Science.gov (United States)

    Powell, Sarah R.; Fuchs, Lynn S.; Fuchs, Douglas

    2012-01-01

    Two aspects of mathematics with which students with mathematics learning difficulty (MLD) often struggle are word problems and number-combination skills. This article describes a math program in which students receive instruction on using algebraic equations to represent the underlying problem structure for three word-problem types. Students also learn counting strategies for answering number combinations that they cannot retrieve from memory. Results from randomized-control trials indicated that embedding the counting strategies for number combinations produces superior word-problem and number-combination outcomes for students with MLD beyond tutoring programs that focus exclusively on number combinations or word problems. PMID:22661880

  4. Primary School Text Comprehension Predicts Mathematical Word Problem-Solving Skills in Secondary School

    Science.gov (United States)

    Björn, Piia Maria; Aunola, Kaisa; Nurmi, Jari-Erik

    2016-01-01

    This longitudinal study aimed to investigate the extent to which primary school text comprehension predicts mathematical word problem-solving skills in secondary school among Finnish students. The participants were 224 fourth graders (9-10 years old at the baseline). The children's text-reading fluency, text comprehension and basic calculation…

  5. VStops: A Thinking Strategy and Visual Representation Approach in Mathematical Word Problem Solving toward Enhancing STEM Literacy

    Science.gov (United States)

    Abdullah, Nasarudin; Halim, Lilia; Zakaria, Effandi

    2014-01-01

    This study aimed to determine the impact of strategic thinking and visual representation approaches (VStops) on the achievement, conceptual knowledge, metacognitive awareness, awareness of problem-solving strategies, and student attitudes toward mathematical word problem solving among primary school students. The experimental group (N = 96)…

  6. Cognitive Skills Used to Solve Mathematical Word Problems and Numerical Operations: A Study of 6- to 7-Year-Old Children

    Science.gov (United States)

    Bjork, Isabel Maria; Bowyer-Crane, Claudine

    2013-01-01

    This study investigates the relationship between skills that underpin mathematical word problems and those that underpin numerical operations, such as addition, subtraction, division and multiplication. Sixty children aged 6-7 years were tested on measures of mathematical ability, reading accuracy, reading comprehension, verbal intelligence and…

  7. A Review of the Effects of Visual-Spatial Representations and Heuristics on Word Problem Solving in Middle School Mathematics

    Science.gov (United States)

    Kribbs, Elizabeth E.; Rogowsky, Beth A.

    2016-01-01

    Mathematics word-problems continue to be an insurmountable challenge for many middle school students. Educators have used pictorial and schematic illustrations within the classroom to help students visualize these problems. However, the data shows that pictorial representations can be more harmful than helpful in that they only display objects or…

  8. Original article Key factors for successful solving of mathematical word problems in fifth-grade learners

    Directory of Open Access Journals (Sweden)

    Marija Kavkler

    2014-05-01

    Full Text Available BACKGROUND Difficulties in solving mathematical word problems (MWP are one of the most common reasons for weak mathematics performance, and poor mathematical literacy has important implications for an individual’s further education, employment opportunities, mental health and quality of life in today’s modern technological society. The purpose of the study was to examine whether Slovenian good and poor MWP solvers differ in arithmetic knowledge and skills, non-verbal reasoning, pupils’ self-evaluations of MWP abilities, teachers’ assessment of their mathematical knowledge and what strategies fifth- grade learners use in solving MWP. PARTICIPANTS AND PROCEDURE The larger sample included 233 pupils from 14 fifth-grade classes (mean age 10 years 3 months and 14 teachers. On the basis of the teachers’ opinions and the results of MWP solving two sub-samples of 24 students were formed, good and poor MWP solvers. Several tests were used to determine MWP solving ability, automation of arithmetic facts and procedures as well as Raven’s SPM. Questionnaires for pupils were used to assess pupils’ estimations of MWP tasks’ difficulty, their own ability to solve them and the strategies used. To assess pupils’ knowledge a questionnaire for teachers was used. RESULTS Slovenian 5 th graders in the larger sample generally used very few empirically proven effective cognitive and metacognitive strategies to solve MWP. Pupils with lower achievement in solving MWP, compared to pupils with higher achievement demonstrated significantly less automated arithmetic facts and procedures of the algorithm, less flexible use of arithmetic skills, as well as qualitatively different MWP solving, which is also related to their lower non-verbal reasoning. Teachers’ assessments and pupils’ self-assessments matched the achieved test results. CONCLUSIONS The results exposed important key factors for successful solving of mathematical word problems with

  9. Word Problem Solving in Contemporary Math Education: A Plea for Reading Comprehension Skills Training

    Science.gov (United States)

    Boonen, Anton J. H.; de Koning, Björn B.; Jolles, Jelle; van der Schoot, Menno

    2016-01-01

    Successfully solving mathematical word problems requires both mental representation skills and reading comprehension skills. In Realistic Math Education (RME), however, students primarily learn to apply the first of these skills (i.e., representational skills) in the context of word problem solving. Given this, it seems legitimate to assume that students from a RME curriculum experience difficulties when asked to solve semantically complex word problems. We investigated this assumption under 80 sixth grade students who were classified as successful and less successful word problem solvers based on a standardized mathematics test. To this end, students completed word problems that ask for both mental representation skills and reading comprehension skills. The results showed that even successful word problem solvers had a low performance on semantically complex word problems, despite adequate performance on semantically less complex word problems. Based on this study, we concluded that reading comprehension skills should be given a (more) prominent role during word problem solving instruction in RME. PMID:26925012

  10. Word problem solving in contemporary math education: A plea for reading comprehension skills training

    Directory of Open Access Journals (Sweden)

    Anton eBoonen

    2016-02-01

    Full Text Available Successfully solving mathematical word problems requires both mental representation skills and reading comprehension skills. In Realistic Math Education (RME, however, students primarily learn to apply the first of these skills (i.e., representational skills in the context of word problem solving. Given this, it seems legitimate to assume that students from a RME curriculum experience difficulties when asked to solve semantically complex word problems. We investigated this assumption under 80 sixth grade students who were classified as successful and less successful word problem solvers based on a standardized mathematics test. To this end, students completed word problems that ask for both mental representation skills and reading comprehension skills. The results showed that even successful word problem solvers had a low performance on semantically complex word problems, despite adequate performance on semantically less complex word problems. Based on this study, we concluded that reading comprehension skills should be given a (more prominent role during word problem solving instruction in RME.

  11. Word Problem Solving in Contemporary Math Education: A Plea for Reading Comprehension Skills Training.

    Science.gov (United States)

    Boonen, Anton J H; de Koning, Björn B; Jolles, Jelle; van der Schoot, Menno

    2016-01-01

    Successfully solving mathematical word problems requires both mental representation skills and reading comprehension skills. In Realistic Math Education (RME), however, students primarily learn to apply the first of these skills (i.e., representational skills) in the context of word problem solving. Given this, it seems legitimate to assume that students from a RME curriculum experience difficulties when asked to solve semantically complex word problems. We investigated this assumption under 80 sixth grade students who were classified as successful and less successful word problem solvers based on a standardized mathematics test. To this end, students completed word problems that ask for both mental representation skills and reading comprehension skills. The results showed that even successful word problem solvers had a low performance on semantically complex word problems, despite adequate performance on semantically less complex word problems. Based on this study, we concluded that reading comprehension skills should be given a (more) prominent role during word problem solving instruction in RME.

  12. Student’s thinking process in solving word problems in geometry

    Science.gov (United States)

    Khasanah, V. N.; Usodo, B.; Subanti, S.

    2018-05-01

    This research aims to find out the thinking process of seventh grade of Junior High School in solve word problem solving of geometry. This research was descriptive qualitative research. The subject of the research was selected based on sex and differences in mathematical ability. Data collection was done based on student’s work test, interview, and observation. The result of the research showed that there was no difference of thinking process between male and female with high mathematical ability, and there were differences of thinking process between male and female with moderate and low mathematical ability. Also, it was found that male with moderate mathematical ability took a long time in the step of making problem solving plans. While female with moderate mathematical ability took a long time in the step of understanding the problems. The importance of knowing the thinking process of students in solving word problem solving were that the teacher knows the difficulties faced by students and to minimize the occurrence of the same error in problem solving. Teacher could prepare the right learning strategies which more appropriate with student’s thinking process.

  13. Word Problem Solving of Students with Autistic Spectrum Disorders and Students with Typical Development

    Science.gov (United States)

    Bae, Young Seh

    2013-01-01

    Mathematical Word Problem Solving of Students with Autistic Spectrum Disorders and Students with Typical Development Young Seh Bae This study investigated mathematical word problem solving and the factors associated with the solution paths adopted by two groups of participants (N=40), students with autism spectrum disorders (ASDs) and typically…

  14. Problem Solving Frameworks for Mathematics and Software Development

    Science.gov (United States)

    McMaster, Kirby; Sambasivam, Samuel; Blake, Ashley

    2012-01-01

    In this research, we examine how problem solving frameworks differ between Mathematics and Software Development. Our methodology is based on the assumption that the words used frequently in a book indicate the mental framework of the author. We compared word frequencies in a sample of 139 books that discuss problem solving. The books were grouped…

  15. Why Do Disadvantaged Filipino Children Find Word Problems in English Difficult?

    Science.gov (United States)

    Bautista, Debbie; Mulligan, Joanne

    2010-01-01

    Young Filipino students are expected to solve mathematical word problems in English, a language that many encounter only in schools. Using individual interviews of 17 Filipino children, we investigated why word problems in English are difficult and the extent to which the language interferes with performance. Results indicate that children could…

  16. Young Filipino Students Making Sense of Arithmetic Word Problems in English

    Science.gov (United States)

    Bautista, Debbie; Mulligan, Joanne; Mitchelmore, Michael

    2009-01-01

    Young Filipino children are expected to solve mathematical word problems in English, a task which they typically encounter only in schools. In this exploratory study, task-based interviews were conducted with seven Filipino children from a public school. The children were asked to read and solve addition and subtraction word problems in English or…

  17. Cognitive Strategy Instruction for Teaching Word Problems to Primary-Level Struggling Students

    Science.gov (United States)

    Pfannenstiel, Kathleen Hughes; Bryant, Diane Pedrotty; Bryant, Brian R.; Porterfield, Jennifer A.

    2015-01-01

    Students with mathematics difficulties and learning disabilities (LD) typically struggle with solving word problems. These students often lack knowledge about efficient, cognitive strategies to utilize when solving word problems. Cognitive strategy instruction has been shown to be effective in teaching struggling students how to solve word…

  18. The relation between early constructive play and mathematical word problem solving is mediated by spatial ability. A path analysis in sixth grade students.

    NARCIS (Netherlands)

    Oostermeijer, M.; Boonen, A.J.H.; Jolles, J.

    2014-01-01

    The scientific literature shows that constructive play activities are positively related to children's spatial ability. Likewise, a close positive relation is found between spatial ability and mathematical word problem-solving performances. The relation between children's constructive play and their

  19. Students' errors in solving linear equation word problems: Case ...

    African Journals Online (AJOL)

    kofi.mereku

    Development in most areas of life is based on effective knowledge of science and ... Problem solving, as used in mathematics education literature, refers ... word problems, on the other hand, are those linear equation tasks or ... taught LEWPs in the junior high school, many of them reach the senior high school without a.

  20. The semantic system is involved in mathematical problem solving.

    Science.gov (United States)

    Zhou, Xinlin; Li, Mengyi; Li, Leinian; Zhang, Yiyun; Cui, Jiaxin; Liu, Jie; Chen, Chuansheng

    2018-02-01

    Numerous studies have shown that the brain regions around bilateral intraparietal cortex are critical for number processing and arithmetical computation. However, the neural circuits for more advanced mathematics such as mathematical problem solving (with little routine arithmetical computation) remain unclear. Using functional magnetic resonance imaging (fMRI), this study (N = 24 undergraduate students) compared neural bases of mathematical problem solving (i.e., number series completion, mathematical word problem solving, and geometric problem solving) and arithmetical computation. Direct subject- and item-wise comparisons revealed that mathematical problem solving typically had greater activation than arithmetical computation in all 7 regions of the semantic system (which was based on a meta-analysis of 120 functional neuroimaging studies on semantic processing). Arithmetical computation typically had greater activation in the supplementary motor area and left precentral gyrus. The results suggest that the semantic system in the brain supports mathematical problem solving. Copyright © 2017 Elsevier Inc. All rights reserved.

  1. Block Model Approach in Problem Solving: Effects on Problem Solving Performance of the Grade V Pupils in Mathematics

    Science.gov (United States)

    de Guzman, Niño Jose P.; Belecina, Rene R.

    2012-01-01

    The teaching of mathematics involves problem solving skills which prove to be difficult on the part of the pupils due to misrepresentation of the word problems. Oftentimes, pupils tend to represent the phrase "more than" as addition and the word difference as "- ". This paper aims to address the problem solving skills of grade…

  2. MONTO: A Machine-Readable Ontology for Teaching Word Problems in Mathematics

    Science.gov (United States)

    Lalingkar, Aparna; Ramnathan, Chandrashekar; Ramani, Srinivasan

    2015-01-01

    The Indian National Curriculum Framework has as one of its objectives the development of mathematical thinking and problem solving ability. However, recent studies conducted in Indian metros have expressed concern about students' mathematics learning. Except in some private coaching academies, regular classroom teaching does not include problem…

  3. Elementary Students' Spontaneous Metacognitive Functions in Different Types of Mathematical Problems

    Science.gov (United States)

    Mokos, Evagelos; Kafoussi, Sonia

    2013-01-01

    Metacognition is the mind's ability to monitor and control itself or, in other words, the ability to know about our knowing (Dunlosky & Bjork, 2008). In mathematics education, the importance of the investigation of students' metacognition during their mathematical activity has been focused on the area of mathematics problem solving. This study…

  4. Language and mathematical problem solving among bilinguals.

    Science.gov (United States)

    Bernardo, Allan B I

    2002-05-01

    Does using a bilingual's 1st or 2nd language have an effect on problem solving in semantically rich domains like school mathematics? The author conducted a study to determine whether Filipino-English bilingual students' understanding and solving of word problems in arithmetic differed when the problems were in the students' 1st and 2nd languages. Two groups participated-students whose 1st language was Filipino and students whose 1st language was English-and easy and difficult arithmetic problems were used. The author used a recall paradigm to assess how students understood the word problems and coded the solution accuracy to assess problem solving. The results indicated a 1st-language advantage; that is, the students were better able to understand and solve problems in their 1st language, whether the 1st language was English or Filipino. Moreover, the advantage was more marked with the easy problems. The theoretical and practical implications of the results are discussed.

  5. Factors Influencing Filipino Children's Solutions to Addition and Subtraction Word Problems

    Science.gov (United States)

    Bautista, Debbie; Mitchelmore, Michael; Mulligan, Joanne

    2009-01-01

    Young Filipino children are expected to solve mathematical word problems in English, which is not their mother tongue. Because of this, it is often assumed that Filipino children have difficulties in solving problems because they cannot read or comprehend what they have read. This study tested this assumption by determining whether presenting word…

  6. A Comparative Analysis of Word Problems in Selected United States and Russian First Grade Textbooks

    Science.gov (United States)

    Grishchenko, Svetlana

    2009-01-01

    The purpose of this study was to explore word problems as a subject matter in mathematics textbook curricula. The motivation for the study derived from the following evidence: (a) American students find some word problems are more difficult than others (Garcia, Jimenez, & Hess, 2006; Riley & Green, 1988; Stern, 2001), and (b) one of the…

  7. Teaching Mathematical Word Problem Solving: The Quality of Evidence for Strategy Instruction Priming the Problem Structure

    Science.gov (United States)

    Jitendra, Asha K.; Petersen-Brown, Shawna; Lein, Amy E.; Zaslofsky, Anne F.; Kunkel, Amy K.; Jung, Pyung-Gang; Egan, Andrea M.

    2015-01-01

    This study examined the quality of the research base related to strategy instruction priming the underlying mathematical problem structure for students with learning disabilities and those at risk for mathematics difficulties. We evaluated the quality of methodological rigor of 18 group research studies using the criteria proposed by Gersten et…

  8. Teaching mathematical word problem solving: the quality of evidence for strategy instruction priming the problem structure.

    Science.gov (United States)

    Jitendra, Asha K; Petersen-Brown, Shawna; Lein, Amy E; Zaslofsky, Anne F; Kunkel, Amy K; Jung, Pyung-Gang; Egan, Andrea M

    2015-01-01

    This study examined the quality of the research base related to strategy instruction priming the underlying mathematical problem structure for students with learning disabilities and those at risk for mathematics difficulties. We evaluated the quality of methodological rigor of 18 group research studies using the criteria proposed by Gersten et al. and 10 single case design (SCD) research studies using criteria suggested by Horner et al. and the What Works Clearinghouse. Results indicated that 14 group design studies met the criteria for high-quality or acceptable research, whereas SCD studies did not meet the standards for an evidence-based practice. Based on these findings, strategy instruction priming the mathematics problem structure is considered an evidence-based practice using only group design methodological criteria. Implications for future research and for practice are discussed. © Hammill Institute on Disabilities 2013.

  9. Chinese Number Words, Culture, and Mathematics Learning

    Science.gov (United States)

    Ng, Sharon Sui Ngan; Rao, Nirmala

    2010-01-01

    This review evaluates the role of language--specifically, the Chinese-based system of number words and the simplicity of Chinese mathematical terms--in explaining the relatively superior performance of Chinese and other East Asian students in cross-national studies of mathematics achievement. Relevant research is critically reviewed focusing on…

  10. The Role of Cognitive Processes, Foundational Math Skill, and Calculation Accuracy and Fluency in Word-Problem Solving versus Pre-Algebraic Knowledge

    Science.gov (United States)

    Fuchs, Lynn S.; Gilbert, Jennifer K.; Powell, Sarah R.; Cirino, Paul T.; Fuchs, Douglas; Hamlett, Carol L.; Seethaler, Pamela M.; Tolar, Tammy D.

    2016-01-01

    The purpose of this study was to examine child-level pathways in development of pre-algebraic knowledge versus word-problem solving, while evaluating the contribution of calculation accuracy and fluency as mediators of foundational skills/processes. Children (n = 962; mean 7.60 years) were assessed on general cognitive processes and early calculation, word-problem, and number knowledge at start of grade 2; calculation accuracy and calculation fluency at end of grade 2; and pre-algebraic knowledge and word-problem solving at end of grade 4. Important similarities in pathways were identified, but path analysis also indicated that language comprehension is more critical for later word-problem solving than pre-algebraic knowledge. We conclude that pathways in development of these forms of 4th-grade mathematics performance are more alike than different, but demonstrate the need to fine-tune instruction for strands of the mathematics curriculum in ways that address individual students’ foundational mathematics skills or cognitive processes. PMID:27786534

  11. The compressed word problem for groups

    CERN Document Server

    Lohrey, Markus

    2014-01-01

    The Compressed Word Problem for Groups provides a detailed exposition of known results on the compressed word problem, emphasizing efficient algorithms for the compressed word problem in various groups. The author presents the necessary background along with the most recent results on the compressed word problem to create a cohesive self-contained book accessible to computer scientists as well as mathematicians. Readers will quickly reach the frontier of current research which makes the book especially appealing for students looking for a currently active research topic at the intersection of group theory and computer science. The word problem introduced in 1910 by Max Dehn is one of the most important decision problems in group theory. For many groups, highly efficient algorithms for the word problem exist. In recent years, a new technique based on data compression for providing more efficient algorithms for word problems, has been developed, by representing long words over group generators in a compres...

  12. The Model Method: Singapore Children's Tool for Representing and Solving Algebraic Word Problems

    Science.gov (United States)

    Ng, Swee Fong; Lee, Kerry

    2009-01-01

    Solving arithmetic and algebraic word problems is a key component of the Singapore elementary mathematics curriculum. One heuristic taught, the model method, involves drawing a diagram to represent key information in the problem. We describe the model method and a three-phase theoretical framework supporting its use. We conducted 2 studies to…

  13. Does Early Algebraic Reasoning Differ as a Function of Students' Difficulty with Calculations versus Word Problems?

    Science.gov (United States)

    Powell, Sarah R; Fuchs, Lynn S

    2014-08-01

    According to national mathematics standards, algebra instruction should begin at kindergarten and continue through elementary school. Most often, teachers address algebra in the elementary grades with problems related to solving equations or understanding functions. With 789 2 nd - grade students, we administered (a) measures of calculations and word problems in the fall and (b) an assessment of pre-algebraic reasoning, with items that assessed solving equations and functions, in the spring. Based on the calculation and word-problem measures, we placed 148 students into 1 of 4 difficulty status categories: typically performing, calculation difficulty, word-problem difficulty, or difficulty with calculations and word problems. Analyses of variance were conducted on the 148 students; path analytic mediation analyses were conducted on the larger sample of 789 students. Across analyses, results corroborated the finding that word-problem difficulty is more strongly associated with difficulty with pre-algebraic reasoning. As an indicator of later algebra difficulty, word-problem difficulty may be a more useful predictor than calculation difficulty, and students with word-problem difficulty may require a different level of algebraic reasoning intervention than students with calculation difficulty.

  14. Does Early Algebraic Reasoning Differ as a Function of Students’ Difficulty with Calculations versus Word Problems?

    Science.gov (United States)

    Powell, Sarah R.; Fuchs, Lynn S.

    2014-01-01

    According to national mathematics standards, algebra instruction should begin at kindergarten and continue through elementary school. Most often, teachers address algebra in the elementary grades with problems related to solving equations or understanding functions. With 789 2nd- grade students, we administered (a) measures of calculations and word problems in the fall and (b) an assessment of pre-algebraic reasoning, with items that assessed solving equations and functions, in the spring. Based on the calculation and word-problem measures, we placed 148 students into 1 of 4 difficulty status categories: typically performing, calculation difficulty, word-problem difficulty, or difficulty with calculations and word problems. Analyses of variance were conducted on the 148 students; path analytic mediation analyses were conducted on the larger sample of 789 students. Across analyses, results corroborated the finding that word-problem difficulty is more strongly associated with difficulty with pre-algebraic reasoning. As an indicator of later algebra difficulty, word-problem difficulty may be a more useful predictor than calculation difficulty, and students with word-problem difficulty may require a different level of algebraic reasoning intervention than students with calculation difficulty. PMID:25309044

  15. The Effect of Contextual and Conceptual Rewording on Mathematical Problem-Solving Performance

    Science.gov (United States)

    Haghverdi, Majid; Wiest, Lynda R.

    2016-01-01

    This study shows how separate and combined contextual and conceptual problem rewording can positively influence student performance in solving mathematical word problems. Participants included 80 seventh-grade Iranian students randomly assigned in groups of 20 to three experimental groups involving three types of rewording and a control group. All…

  16. COMPUTER TOOLS OF DYNAMIC MATHEMATIC SOFTWARE AND METHODICAL PROBLEMS OF THEIR USE

    Directory of Open Access Journals (Sweden)

    Olena V. Semenikhina

    2014-08-01

    Full Text Available The article presents results of analyses of standard computer tools of dynamic mathematic software which are used in solving tasks, and tools on which the teacher can support in the teaching of mathematics. Possibility of the organization of experimental investigating of mathematical objects on the basis of these tools and the wording of new tasks on the basis of the limited number of tools, fast automated check are specified. Some methodological comments on application of computer tools and methodological features of the use of interactive mathematical environments are presented. Problems, which are arising from the use of computer tools, among which rethinking forms and methods of training by teacher, the search for creative problems, the problem of rational choice of environment, check the e-solutions, common mistakes in the use of computer tools are selected.

  17. Effects of the SOLVE Strategy on the Mathematical Problem Solving Skills of Secondary Students with Learning Disabilities

    Science.gov (United States)

    Freeman-Green, Shaqwana M.; O'Brien, Chris; Wood, Charles L.; Hitt, Sara Beth

    2015-01-01

    This study examined the effects of explicit instruction in the SOLVE Strategy on the mathematical problem solving skills of six Grade 8 students with specific learning disabilities. The SOLVE Strategy is an explicit instruction, mnemonic-based learning strategy designed to help students in solving mathematical word problems. Using a multiple probe…

  18. Automatic Item Generation via Frame Semantics: Natural Language Generation of Math Word Problems.

    Science.gov (United States)

    Deane, Paul; Sheehan, Kathleen

    This paper is an exploration of the conceptual issues that have arisen in the course of building a natural language generation (NLG) system for automatic test item generation. While natural language processing techniques are applicable to general verbal items, mathematics word problems are particularly tractable targets for natural language…

  19. Understanding the Role of Linguistic Processes in the Solution of Arithmetic Word Problems.

    Science.gov (United States)

    LeBlanc, Mark D.

    Ongoing work toward developing a learning environment that will perform real-time diagnoses of students' difficulties in solving mathematical word problems is described. The learning environment designed consists of a microworld and expert modules. The microworld (or toolbox) is a collection of mouse-driven interfaces that facilitate a transition…

  20. Math word problems for dummies

    CERN Document Server

    Sterling, Mary Jane

    2008-01-01

    Covers percentages, probability, proportions, and moreGet a grip on all types of word problems by applying them to real lifeAre you mystified by math word problems? This easy-to-understand guide shows you how to conquer these tricky questions with a step-by-step plan for finding the right solution each and every time, no matter the kind or level of problem. From learning math lingo and performing operations to calculating formulas and writing equations, you''ll get all the skills you need to succeed!Discover how to: * Translate word problems into plain English* Brush up on basic math skills* Plug in the right operation or formula* Tackle algebraic and geometric problems* Check your answers to see if they work

  1. Improvement of Word Problem Solving and Basic Mathematics Competencies in Students with Attention Deficit/Hyperactivity Disorder and Mathematical Learning Difficulties

    Science.gov (United States)

    González-Castro, Paloma; Cueli, Marisol; Areces, Débora; Rodríguez, Celestino; Sideridis, Georgios

    2016-01-01

    Problem solving represents a salient deficit in students with mathematical learning difficulties (MLD) primarily caused by difficulties with informal and formal mathematical competencies. This study proposes a computerized intervention tool, the integrated dynamic representation (IDR), for enhancing the early learning of basic mathematical…

  2. The effects of using diagramming as a representational technique on high school students' achievement in solving math word problems

    Science.gov (United States)

    Banerjee, Banmali

    Methods and procedures for successfully solving math word problems have been, and continue to be a mystery to many U.S. high school students. Previous studies suggest that the contextual and mathematical understanding of a word problem, along with the development of schemas and their related external representations, positively contribute to students' accomplishments when solving word problems. Some studies have examined the effects of diagramming on students' abilities to solve word problems that only involved basic arithmetic operations. Other studies have investigated how instructional models that used technology influenced students' problem solving achievements. Still other studies have used schema-based instruction involving students with learning disabilities. No study has evaluated regular high school students' achievements in solving standard math word problems using a diagramming technique without technological aid. This study evaluated students' achievement in solving math word problems using a diagramming technique. Using a quasi-experimental experimental pretest-posttest research design, quantitative data were collected from 172 grade 11 Hispanic English language learners (ELLS) and African American learners whose first language is English (EFLLs) in 18 classes at an inner city high school in Northern New Jersey. There were 88 control and 84 experimental students. The pretest and posttest of each participating student and samples of the experimental students' class assignments provided the qualitative data for the study. The data from this study exhibited that the diagramming method of solving math word problems significantly improved student achievement in the experimental group (pvocabulary and symbols used in word problems and that both ELLs and EFLLs improved their problem solving success through careful attention to the creation and labeling of diagrams to represent the mathematics involved in standard word problems. Although Learnertype (ELL, EFLL

  3. Word Problem Wizardry.

    Science.gov (United States)

    Cassidy, Jack

    1991-01-01

    Presents suggestions for teaching math word problems to elementary students. The strategies take into consideration differences between reading in math and reading in other areas. A problem-prediction game and four self-checking activities are included along with a magic password challenge. (SM)

  4. Word problems and make-believe: Using frame analysis and ethnomethodology to explore aspects of the culture of schooling

    Directory of Open Access Journals (Sweden)

    Benincasa Luciana

    2017-12-01

    Full Text Available The paper applies Goffman’s frame analysis and ethnomethodology to student performance on mathematical word problems. In educational research, frame analysis has usually been limited to primary frames. Instead, in this paper I focus on the kind of secondary frame that Goffman calls ‘utilitarian make-believe’. The data consist of a fragment of verbal interaction between a teacher and a 12-year-old pupil during an oral mathematics exam. By evoking the idea of ‘as-ifness’, word problems introduce pupils to a make-believe world. The text consists only of ‘filler words’ because what really matters are the figures. Word problems and possibly other aspects of schooling can be interpreted in terms of a utilitarian make-believe key. Readiness to adopt this make-believe frame when required may be the difference between school success and failure. I argue that maths achievement takes more than just ‘being good with numbers’. It is a joint enterprise of people interacting within a culturally-shaped setting, organized so as to make some phenomena stand out rather than others. Finally, I argue that ‘word problems and possibly other ‘school genres’ could be added to the list of utilitarian make-believe frames provided by Goffman.

  5. Three-M in Word Problem Solving

    Science.gov (United States)

    Hajra, Sayonita Ghosh; Kofman, Victoria

    2018-01-01

    We describe three activities that help undergraduates (pre-service teachers) to develop scientific vocabulary on measurable attributes and units of measurement. Measurable attributes are important features in understanding a word problem and solving the problem. These activities help students comprehend word problems better by identifying…

  6. Syntactic Awareness and Arithmetic Word Problem Solving in Children with and without Learning Disabilities

    Science.gov (United States)

    Peake, Christian; Jiménez, Juan E.; Rodríguez, Cristina; Bisschop, Elaine; Villarroel, Rebeca

    2015-01-01

    Arithmetic word problem (AWP) solving is a highly demanding task for children with learning disabilities (LD) since verbal and mathematical information have to be integrated. This study examines specifically how syntactic awareness (SA), the ability to manage the grammatical structures of language, affects AWP solving. Three groups of children in…

  7. Open problems in mathematics

    CERN Document Server

    Nash, Jr, John Forbes

    2016-01-01

    The goal in putting together this unique compilation was to present the current status of the solutions to some of the most essential open problems in pure and applied mathematics. Emphasis is also given to problems in interdisciplinary research for which mathematics plays a key role. This volume comprises highly selected contributions by some of the most eminent mathematicians in the international mathematical community on longstanding problems in very active domains of mathematical research. A joint preface by the two volume editors is followed by a personal farewell to John F. Nash, Jr. written by Michael Th. Rassias. An introduction by Mikhail Gromov highlights some of Nash’s legendary mathematical achievements. The treatment in this book includes open problems in the following fields: algebraic geometry, number theory, analysis, discrete mathematics, PDEs, differential geometry, topology, K-theory, game theory, fluid mechanics, dynamical systems and ergodic theory, cryptography, theoretical computer sc...

  8. Some unsolved problems in discrete mathematics and mathematical cybernetics

    Science.gov (United States)

    Korshunov, Aleksei D.

    2009-10-01

    There are many unsolved problems in discrete mathematics and mathematical cybernetics. Writing a comprehensive survey of such problems involves great difficulties. First, such problems are rather numerous and varied. Second, they greatly differ from each other in degree of completeness of their solution. Therefore, even a comprehensive survey should not attempt to cover the whole variety of such problems; only the most important and significant problems should be reviewed. An impersonal choice of problems to include is quite hard. This paper includes 13 unsolved problems related to combinatorial mathematics and computational complexity theory. The problems selected give an indication of the author's studies for 50 years; for this reason, the choice of the problems reviewed here is, to some extent, subjective. At the same time, these problems are very difficult and quite important for discrete mathematics and mathematical cybernetics. Bibliography: 74 items.

  9. Some unsolved problems in discrete mathematics and mathematical cybernetics

    International Nuclear Information System (INIS)

    Korshunov, Aleksei D

    2009-01-01

    There are many unsolved problems in discrete mathematics and mathematical cybernetics. Writing a comprehensive survey of such problems involves great difficulties. First, such problems are rather numerous and varied. Second, they greatly differ from each other in degree of completeness of their solution. Therefore, even a comprehensive survey should not attempt to cover the whole variety of such problems; only the most important and significant problems should be reviewed. An impersonal choice of problems to include is quite hard. This paper includes 13 unsolved problems related to combinatorial mathematics and computational complexity theory. The problems selected give an indication of the author's studies for 50 years; for this reason, the choice of the problems reviewed here is, to some extent, subjective. At the same time, these problems are very difficult and quite important for discrete mathematics and mathematical cybernetics. Bibliography: 74 items.

  10. Some unsolved problems in discrete mathematics and mathematical cybernetics

    Energy Technology Data Exchange (ETDEWEB)

    Korshunov, Aleksei D [S.L. Sobolev Institute for Mathematics, Siberian Branch of the Russian Academy of Sciences, Novosibirsk (Russian Federation)

    2009-10-31

    There are many unsolved problems in discrete mathematics and mathematical cybernetics. Writing a comprehensive survey of such problems involves great difficulties. First, such problems are rather numerous and varied. Second, they greatly differ from each other in degree of completeness of their solution. Therefore, even a comprehensive survey should not attempt to cover the whole variety of such problems; only the most important and significant problems should be reviewed. An impersonal choice of problems to include is quite hard. This paper includes 13 unsolved problems related to combinatorial mathematics and computational complexity theory. The problems selected give an indication of the author's studies for 50 years; for this reason, the choice of the problems reviewed here is, to some extent, subjective. At the same time, these problems are very difficult and quite important for discrete mathematics and mathematical cybernetics. Bibliography: 74 items.

  11. How do open-ended problems promote mathematical creativity? A reflection of bare mathematics problem and contextual problem

    Science.gov (United States)

    Wijaya, A.

    2018-03-01

    Creativity is often seen as one of the fundamental aspects of character education. As one of the 21st century skills, creativity has also been considered as an important goal of education across the world. This paper reports a study on promoting mathematical creativity through the use of open-ended mathematics problems. A total of 53 undergraduate students participated in the study. These students worked on open-ended problems in two types, i.e. bare mathematics problem and contextual problem. The contextual problem was presented in the form of paper-based and Geogebra-based. The students’ works were analysed qualitatively in order to describe how students’ mathematical creativity developed. It was found that the open-ended problems successfully promote students’ creativity as indicated by various solutions or strategies that were used by students to solve the problems. The analysis of students’ works show that students’ creativity developed through three kinds of exploration, i. e. (1) exploration of contexts, (2) exploration of software features, and (3) exploration of mathematics concepts. The use of metacognitive questioning was found to be helpful to develop the first two explorations into mathematical exploration.

  12. Mathematical problems for chemistry students

    CERN Document Server

    Pota, Gyorgy

    2011-01-01

    Mathematical Problems for Chemistry Students has been compiled and written (a) to help chemistrystudents in their mathematical studies by providing them with mathematical problems really occurring in chemistry (b) to help practising chemists to activate their applied mathematical skills and (c) to introduce students and specialistsof the chemistry-related fields (physicists, mathematicians, biologists, etc.) intothe world of the chemical applications.Some problems of the collection are mathematical reformulations of those in the standard textbooks of chemistry, others we

  13. The Effect of Dynamic and Interactive Mathematics Learning Environments (DIMLE), Supporting Multiple Representations, on Perceptions of Elementary Mathematics Pre-Service Teachers in Problem Solving Process

    Science.gov (United States)

    Ozdemir, S.; Reis, Z. Ayvaz

    2013-01-01

    Mathematics is an important discipline, providing crucial tools, such as problem solving, to improve our cognitive abilities. In order to solve a problem, it is better to envision and represent through multiple means. Multiple representations can help a person to redefine a problem with his/her own words in that envisioning process. Dynamic and…

  14. Child-Level Predictors of Responsiveness to Evidence-Based Mathematics Intervention.

    Science.gov (United States)

    Powell, Sarah R; Cirino, Paul T; Malone, Amelia S

    2017-07-01

    We identified child-level predictors of responsiveness to 2 types of mathematics (calculation and word-problem) intervention among 2nd-grade children with mathematics difficulty. Participants were 250 children in 107 classrooms in 23 schools pretested on mathematics and general cognitive measures and posttested on mathematics measures. We assigned classrooms randomly assigned to calculation intervention, word-problem intervention, or business-as-usual control. Intervention lasted 17 weeks. Path analyses indicated that scores on working memory and language comprehension assessments moderated responsiveness to calculation intervention. No moderators were identified for responsiveness to word-problem intervention. Across both intervention groups and the control group, attentive behavior predicted both outcomes. Initial calculation skill predicted the calculation outcome, and initial language comprehension predicted word-problem outcomes. These results indicate that screening for calculation intervention should include a focus on working memory, language comprehension, attentive behavior, and calculations. Screening for word-problem intervention should focus on attentive behavior and word problems.

  15. Solving applied mathematical problems with Matlab

    CERN Document Server

    Xue, Dingyu

    2008-01-01

    Computer Mathematics Language-An Overview. Fundamentals of MATLAB Programming. Calculus Problems. MATLAB Computations of Linear Algebra Problems. Integral Transforms and Complex Variable Functions. Solutions to Nonlinear Equations and Optimization Problems. MATLAB Solutions to Differential Equation Problems. Solving Interpolations and Approximations Problems. Solving Probability and Mathematical Statistics Problems. Nontraditional Solution Methods for Mathematical Problems.

  16. How to solve mathematical problems

    CERN Document Server

    Wickelgren, Wayne A

    1995-01-01

    Seven problem-solving techniques include inference, classification of action sequences, subgoals, contradiction, working backward, relations between problems, and mathematical representation. Also, problems from mathematics, science, and engineering with complete solutions.

  17. Word Problems: A "Meme" for Our Times.

    Science.gov (United States)

    Leamnson, Robert N.

    1996-01-01

    Discusses a novel approach to word problems that involves linear relationships between variables. Argues that working stepwise through intermediates is the way our minds actually work and therefore this should be used in solving word problems. (JRH)

  18. How Readability Factors Are Differentially Associated with Performance for Students of Different Backgrounds When Solving Mathematics Word Problems

    Science.gov (United States)

    Walkington, Candace; Clinton, Virginia; Shivraj, Pooja

    2018-01-01

    The link between reading and mathematics achievement is well known, and an important question is whether readability factors in mathematics problems are differentially impacting student groups. Using 20 years of data from the National Assessment of Educational Progress and the Trends in International Mathematics and Science Study, we examine how…

  19. Fraction Multiplication and Division Word Problems Posed by Different Years of Pre-Service Elementary Mathematics Teachers

    Directory of Open Access Journals (Sweden)

    Tuba Aydogdu Iskenderoglu

    2018-04-01

    Full Text Available It is important for pre-service teachers to know the conceptual difficulties they have experienced regarding the concepts of multiplication and division in fractions and problem posing is a way to learn these conceptual difficulties. Problem posing is a synthetic activity that fundamentally has multiple answers. The purpose of this study is to analyze the multiplication and division of fractions problems posed by pre-service elementary mathematics teachers and to investigate how the problems posed change according to the year of study the pre-service teachers are in. The study employed developmental research methods. A total of 213 pre-service teachers enrolled in different years of the Elementary Mathematics Teaching program at a state university in Turkey took part in the study. The “Problem Posing Test” was used as the data collecting tool. In this test, there are 3 multiplication and 3 division operations. The data were analyzed using qualitative descriptive analysis. The findings suggest that, regardless of the year, pre-service teachers had more conceptual difficulties in problem posing about the division of fractions than in problem posing about the multiplication of fractions.

  20. Mathematical Thinking and Creativity through Mathematical Problem Posing and Solving

    Directory of Open Access Journals (Sweden)

    María F. Ayllón

    2016-04-01

    Full Text Available This work shows the relationship between the development of mathematical thinking and creativity with mathematical problem posing and solving. Creativity and mathematics are disciplines that do not usually appear together. Both concepts constitute complex processes sharing elements, such as fluency (number of ideas, flexibility (range of ideas, novelty (unique idea and elaboration (idea development. These factors contribute, among others, to the fact that schoolchildren are competent in mathematics. The problem solving and posing are a very powerful evaluation tool that shows the mathematical reasoning and creative level of a person. Creativity is part of the mathematics education and is a necessary ingredient to perform mathematical assignments. This contribution presents some important research works about problem posing and solving related to the development of mathematical knowledge and creativity. To that end, it is based on various beliefs reflected in the literature with respect to notions of creativity, problem solving and posing.

  1. Evaluation of the Effectiveness of a Tablet Computer Application (App) in Helping Students with Visual Impairments Solve Mathematics Problems

    Science.gov (United States)

    Beal, Carole R.; Rosenblum, L. Penny

    2018-01-01

    Introduction: The authors examined a tablet computer application (iPad app) for its effectiveness in helping students studying prealgebra to solve mathematical word problems. Methods: Forty-three visually impaired students (that is, those who are blind or have low vision) completed eight alternating mathematics units presented using their…

  2. Problem solving through recreational mathematics

    CERN Document Server

    Averbach, Bonnie

    1999-01-01

    Historically, many of the most important mathematical concepts arose from problems that were recreational in origin. This book takes advantage of that fact, using recreational mathematics - problems, puzzles and games - to teach students how to think critically. Encouraging active participation rather than just observation, the book focuses less on mathematical results than on how these results can be applied to thinking about problems and solving them. Each chapter contains a diverse array of problems in such areas as logic, number and graph theory, two-player games of strategy, solitaire ga

  3. Secondary Teachers’ Mathematics-related Beliefs and Knowledge about Mathematical Problem-solving

    Science.gov (United States)

    E Siswono, T. Y.; Kohar, A. W.; Hartono, S.

    2017-02-01

    This study investigates secondary teachers’ belief about the three mathematics-related beliefs, i.e. nature of mathematics, teaching mathematics, learning mathematics, and knowledge about mathematical problem solving. Data were gathered through a set of task-based semi-structured interviews of three selected teachers with different philosophical views of teaching mathematics, i.e. instrumental, platonist, and problem solving. Those teachers were selected from an interview using a belief-related task from purposively selected teachers in Surabaya and Sidoarjo. While the interviews about knowledge examine teachers’ problem solving content and pedagogical knowledge, the interviews about beliefs examine their views on several cases extracted from each of such mathematics-related beliefs. Analysis included the categorization and comparison on each of beliefs and knowledge as well as their interaction. Results indicate that all the teachers did not show a high consistency in responding views of their mathematics-related beliefs, while they showed weaknesses primarily on problem solving content knowledge. Findings also point out that teachers’ beliefs have a strong relationship with teachers’ knowledge about problem solving. In particular, the instrumental teacher’s beliefs were consistent with his insufficient knowledge about problem-solving, while both platonist and problem-solving teacher’s beliefs were consistent with their sufficient knowledge of either content or pedagogical problem solving.

  4. Mathematical problems in meteorological modelling

    CERN Document Server

    Csomós, Petra; Faragó, István; Horányi, András; Szépszó, Gabriella

    2016-01-01

    This book deals with mathematical problems arising in the context of meteorological modelling. It gathers and presents some of the most interesting and important issues from the interaction of mathematics and meteorology. It is unique in that it features contributions on topics like data assimilation, ensemble prediction, numerical methods, and transport modelling, from both mathematical and meteorological perspectives. The derivation and solution of all kinds of numerical prediction models require the application of results from various mathematical fields. The present volume is divided into three parts, moving from mathematical and numerical problems through air quality modelling, to advanced applications in data assimilation and probabilistic forecasting. The book arose from the workshop “Mathematical Problems in Meteorological Modelling” held in Budapest in May 2014 and organized by the ECMI Special Interest Group on Numerical Weather Prediction. Its main objective is to highlight the beauty of the de...

  5. USING TASK LIKE PISA’S PROBLEM TO SUPPORT STUDENT’S CREATIVITY IN MATHEMATICS

    Directory of Open Access Journals (Sweden)

    Rita Novita

    2016-01-01

    Full Text Available Creativity is one of keys to success in the evolving global economy and also be a fundamental skill that is absolutely necessary in the 21st century. Also In mathematics, creativity or thinking creatively is important to be developed because creativity is an integral part of mathematics. However, limiting the use of creativity in the classroom reduces mathematics to a set of skills to master and rules to memorize. Doing so causes many children’s natural curiosity and enthusiasm for mathematics to disappear as they get older, creating a tremendous problem for mathematics educators who are trying to instil these very qualities. In order to investigate the increase in awareness of elementary school students’ creativity in solving mathematics’ problems by using task like PISA’s Question, a qualitative research emphasizing on holistic description was conducted. We used a formative evaluation type of development research as a mean to develop mathematical tasks like PISA’s question that have potential effect to support students’ creativity in mathematics. Ten elementary school students of grade 6 in Palembang were involved in this research. They judged the task given for them is very challenging and provokes their curiosity. The result showed that task like PISA’s question can encourage students to more creatively in mathematics.Key Words: PISA, Problem Solving, Creativity in Mathematics DOI: http://dx.doi.org/10.22342/jme.7.1.2815.31-42

  6. The Construction of Mathematical Literacy Problems for Geometry

    Science.gov (United States)

    Malasari, P. N.; Herman, T.; Jupri, A.

    2017-09-01

    The students of junior high school should have mathematical literacy ability to formulate, apply, and interpret mathematics in problem solving of daily life. Teaching these students are not enough by giving them ordinary mathematics problems. Teaching activities for these students brings consequence for teacher to construct mathematical literacy problems. Therefore, the aim of this study is to construct mathematical literacy problems to assess mathematical literacy ability. The steps of this study that consists of analysing, designing, theoretical validation, revising, limited testing to students, and evaluating. The data was collected with written test to 38 students of grade IX at one of state junior high school. Mathematical literacy problems consist of three essays with three indicators and three levels at polyhedron subject. The Indicators are formulating and employing mathematics. The results show that: (1) mathematical literacy problems which are constructed have been valid and practical, (2) mathematical literacy problems have good distinguishing characteristics and adequate distinguishing characteristics, (3) difficulty levels of problems are easy and moderate. The final conclusion is mathematical literacy problems which are constructed can be used to assess mathematical literacy ability.

  7. Current problems in applied mathematics and mathematical physics

    Science.gov (United States)

    Samarskii, A. A.

    Papers are presented on such topics as mathematical models in immunology, mathematical problems of medical computer tomography, classical orthogonal polynomials depending on a discrete variable, and boundary layer methods for singular perturbation problems in partial derivatives. Consideration is also given to the computer simulation of supernova explosion, nonstationary internal waves in a stratified fluid, the description of turbulent flows by unsteady solutions of the Navier-Stokes equations, and the reduced Galerkin method for external diffraction problems using the spline approximation of fields.

  8. Processes involved in solving mathematical problems

    Science.gov (United States)

    Shahrill, Masitah; Putri, Ratu Ilma Indra; Zulkardi, Prahmana, Rully Charitas Indra

    2018-04-01

    This study examines one of the instructional practices features utilized within the Year 8 mathematics lessons in Brunei Darussalam. The codes from the TIMSS 1999 Video Study were applied and strictly followed, and from the 183 mathematics problems recorded, there were 95 problems with a solution presented during the public segments of the video-recorded lesson sequences of the four sampled teachers. The analyses involved firstly, identifying the processes related to mathematical problem statements, and secondly, examining the different processes used in solving the mathematical problems for each problem publicly completed during the lessons. The findings revealed that for three of the teachers, their problem statements coded as `using procedures' ranged from 64% to 83%, while the remaining teacher had 40% of his problem statements coded as `making connections.' The processes used when solving the problems were mainly `using procedures', and none of the problems were coded as `giving results only'. Furthermore, all four teachers made use of making the relevant connections in solving the problems given to their respective students.

  9. Word Problems and Make-Believe: Using Frame Analysis and Ethnomethodology to Explore Aspects of the Culture of Schooling

    Science.gov (United States)

    Benincasa, Luciana

    2017-01-01

    The paper applies Goffman's frame analysis and ethnomethodology to student performance on mathematical word problems. In educational research, frame analysis has usually been limited to primary frames. Instead, in this paper I focus on the kind of secondary frame that Goffman calls 'utilitarian make-believe'. The data consist of a fragment of…

  10. Modern problems in insurance mathematics

    CERN Document Server

    Martin-Löf, Anders

    2014-01-01

    This book is a compilation of 21 papers presented at the International Cramér Symposium on Insurance Mathematics (ICSIM) held at Stockholm University in June, 2013. The book comprises selected contributions from several large research communities in modern insurance mathematics and its applications. The main topics represented in the book are modern risk theory and its applications, stochastic modelling of insurance business, new mathematical problems in life and non-life insurance, and related topics in applied and financial mathematics. The book is an original and useful source of inspiration and essential reference for a broad spectrum of theoretical and applied researchers, research students and experts from the insurance business. In this way, Modern Problems in Insurance Mathematics will contribute to the development of research and academy–industry co-operation in the area of insurance mathematics and its applications.

  11. Mathematical Profiles and Problem Solving Abilities of Mathematically Promising Students

    Science.gov (United States)

    Budak, Ibrahim

    2012-01-01

    Mathematically promising students are defined as those who have the potential to become the leaders and problem solvers of the future. The purpose of this research is to reveal what problem solving abilities mathematically promising students show in solving non-routine problems and type of profiles they present in the classroom and during problem…

  12. Effects of computer-based graphic organizers to solve one-step word problems for middle school students with mild intellectual disability: A preliminary study.

    Science.gov (United States)

    Sheriff, Kelli A; Boon, Richard T

    2014-08-01

    The purpose of this study was to examine the effects of computer-based graphic organizers, using Kidspiration 3© software, to solve one-step word problems. Participants included three students with mild intellectual disability enrolled in a functional academic skills curriculum in a self-contained classroom. A multiple probe single-subject research design (Horner & Baer, 1978) was used to evaluate the effectiveness of computer-based graphic organizers to solving mathematical one-step word problems. During the baseline phase, the students completed a teacher-generated worksheet that consisted of nine functional word problems in a traditional format using a pencil, paper, and a calculator. In the intervention and maintenance phases, the students were instructed to complete the word problems using a computer-based graphic organizer. Results indicated that all three of the students improved in their ability to solve the one-step word problems using computer-based graphic organizers compared to traditional instructional practices. Limitations of the study and recommendations for future research directions are discussed. Copyright © 2014 Elsevier Ltd. All rights reserved.

  13. Research Mathematicians' Practices in Selecting Mathematical Problems

    Science.gov (United States)

    Misfeldt, Morten; Johansen, Mikkel Willum

    2015-01-01

    Developing abilities to create, inquire into, qualify, and choose among mathematical problems is an important educational goal. In this paper, we elucidate how mathematicians work with mathematical problems in order to understand this mathematical process. More specifically, we investigate how mathematicians select and pose problems and discuss to…

  14. Metacognition Process of Students with High Mathematics Anxiety in Mathematics Problem-Solving

    OpenAIRE

    Patrisius Afrisno Udil; Tri Atmojo Kusmayadi; Riyadi Riyadi

    2017-01-01

    This study aims to find out students’ metacognition process while solving the mathematics problem. It focuses on analyzing the metacognition process of students with high mathematics anxiety based on Polya’s problem solving phases. This study uses qualitative research with case study strategy. The subjects consist of 8 students of 7th grade selected through purposive sampling. Data in the form of Mathematics Anxiety Scale (MAS) result and recorded interview while solving mathematics problems ...

  15. It's Not a Math Lesson--We're Learning to Draw! Teachers' Use of Visual Representations in Instructing Word Problem Solving in Sixth Grade of Elementary School

    Science.gov (United States)

    Boonen, Anton J. H.; Reed, Helen C.; Schoonenboom, Judith; Jolles, Jelle

    2016-01-01

    Non-routine word problem solving is an essential feature of the mathematical development of elementary school students worldwide. Many students experience difficulties in solving these problems due to erroneous problem comprehension. These difficulties could be alleviated by instructing students how to use visual representations that clarify the…

  16. Why does placing the question before an arithmetic word problem improve performance? A situation model account.

    Science.gov (United States)

    Thevenot, Catherine; Devidal, Michel; Barrouillet, Pierre; Fayol, Michel

    2007-01-01

    The aim of this paper is to investigate the controversial issue of the nature of the representation constructed by individuals to solve arithmetic word problems. More precisely, we consider the relevance of two different theories: the situation or mental model theory (Johnson-Laird, 1983; Reusser, 1989) and the schema theory (Kintsch & Greeno, 1985; Riley, Greeno, & Heller, 1983). Fourth-graders who differed in their mathematical skills were presented with problems that varied in difficulty and with the question either before or after the text. We obtained the classic effect of the position of the question, with better performance when the question was presented prior to the text. In addition, this effect was more marked in the case of children who had poorer mathematical skills and in the case of more difficult problems. We argue that this pattern of results is compatible only with the situation or mental model theory, and not with the schema theory.

  17. Solving Mathematical Problems A Personal Perspective

    CERN Document Server

    Tao, Terence

    2006-01-01

    Authored by a leading name in mathematics, this engaging and clearly presented text leads the reader through the tactics involved in solving mathematical problems at the Mathematical Olympiad level. With numerous exercises and assuming only basic mathematics, this text is ideal for students of 14 years and above in pure mathematics.

  18. Learning via problem solving in mathematics education

    Directory of Open Access Journals (Sweden)

    Piet Human

    2009-09-01

    Full Text Available Three forms of mathematics education at school level are distinguished: direct expository teaching with an emphasis on procedures, with the expectation that learners will at some later stage make logical and functional sense of what they have learnt and practised (the prevalent form, mathematically rigorous teaching in terms of fundamental mathematical concepts, as in the so-called “modern mathematics” programmes of the sixties, teaching and learning in the context of engaging with meaningful problems and focused both on learning to become good problem solvers (teaching for problem solving andutilising problems as vehicles for the development of mathematical knowledge andproficiency by learners (problem-centred learning, in conjunction with substantialteacher-led social interaction and mathematical discourse in classrooms.Direct expository teaching of mathematical procedures dominated in school systems after World War II, and was augmented by the “modern mathematics” movement in the period 1960-1970. The latter was experienced as a major failure, and was soon abandoned. Persistent poor outcomes of direct expository procedural teaching of mathematics for the majority of learners, as are still being experienced in South Africa, triggered a world-wide movement promoting teaching mathematics for and via problem solving in the seventies and eighties of the previous century. This movement took the form of a variety of curriculum experiments in which problem solving was the dominant classroom activity, mainly in the USA, Netherlands, France and South Africa. While initially focusing on basic arithmetic (computation with whole numbers and elementary calculus, the problem-solving movement started to address other mathematical topics (for example, elementary statistics, algebra, differential equations around the turn of the century. The movement also spread rapidly to other countries, including Japan, Singapore and Australia. Parallel with the

  19. Affect and mathematical problem solving a new perspective

    CERN Document Server

    Adams, Verna

    1989-01-01

    Research on cognitive aspects of mathematical problem solving has made great progress in recent years, but the relationship of affective factors to problem-solving performance has been a neglected research area. The purpose of Affect and Mathematical Problem Solving: A New Perspective is to show how the theories and methods of cognitive science can be extended to include the role of affect in mathematical problem solving. The book presents Mandler's theory of emotion and explores its implications for the learning and teaching of mathematical problem solving. Also, leading researchers from mathematics, education, and psychology report how they have integrated affect into their own cognitive research. The studies focus on metacognitive processes, aesthetic influences on expert problem solvers, teacher decision-making, technology and teaching problem solving, and beliefs about mathematics. The results suggest how emotional factors like anxiety, frustration, joy, and satisfaction can help or hinder performance in...

  20. Improving mathematical problem solving : A computerized approach

    NARCIS (Netherlands)

    Harskamp, EG; Suhre, CJM

    Mathematics teachers often experience difficulties in teaching students to become skilled problem solvers. This paper evaluates the effectiveness of two interactive computer programs for high school mathematics problem solving. Both programs present students with problems accompanied by instruction

  1. Effects of a Mathematics Fluency Program on Mathematics Performance of Students with Challenging Behaviors

    Science.gov (United States)

    Whitney, Todd; Hirn, Regina G.; Lingo, Amy S.

    2016-01-01

    In the present study, we examined the effects of a fluency-building mathematics program called Great Leaps Math on fluency of basic addition mathematics facts zero to nine and word problem solving using a multiple probe design across participants. Three elementary students with challenging behaviors and mathematics difficulty participated in the…

  2. Students’ Mathematical Problem-Solving Abilities Through The Application of Learning Models Problem Based Learning

    Science.gov (United States)

    Nasution, M. L.; Yerizon, Y.; Gusmiyanti, R.

    2018-04-01

    One of the purpose mathematic learning is to develop problem solving abilities. Problem solving is obtained through experience in questioning non-routine. Improving students’ mathematical problem-solving abilities required an appropriate strategy in learning activities one of them is models problem based learning (PBL). Thus, the purpose of this research is to determine whether the problem solving abilities of mathematical students’ who learn to use PBL better than on the ability of students’ mathematical problem solving by applying conventional learning. This research included quasi experiment with static group design and population is students class XI MIA SMAN 1 Lubuk Alung. Class experiment in the class XI MIA 5 and class control in the class XI MIA 6. The instrument of final test students’ mathematical problem solving used essay form. The result of data final test in analyzed with t-test. The result is students’ mathematical problem solving abilities with PBL better then on the ability of students’ mathematical problem solving by applying conventional learning. It’s seen from the high percentage achieved by the group of students who learn to use PBL for each indicator of students’ mathematical problem solving.

  3. Anticipation Guides: Reading for Mathematics Understanding

    Science.gov (United States)

    Adams, Anne E.; Pegg, Jerine; Case, Melissa

    2015-01-01

    With the acceptance by many states of the Common Core State Standards for Mathematics, new emphasis is being placed on students' ability to engage in mathematical practices such as understanding problems (including word problems), reading and critiquing arguments, and making explicit use of definitions (CCSSI 2010). Engaging students in…

  4. Mathematical Thinking and Creativity through Mathematical Problem Posing and Solving

    Science.gov (United States)

    Ayllón, María F.; Gómez, Isabel A.; Ballesta-Claver, Julio

    2016-01-01

    This work shows the relationship between the development of mathematical thinking and creativity with mathematical problem posing and solving. Creativity and mathematics are disciplines that do not usually appear together. Both concepts constitute complex processes sharing elements, such as fluency (number of ideas), flexibility (range of ideas),…

  5. The Music of Mathematics: Toward a New Problem Typology

    Science.gov (United States)

    Quarfoot, David

    Halmos (1980) once described problems and their solutions as "the heart of mathematics". Following this line of thinking, one might naturally ask: "What, then, is the heart of problems?". In this work, I attempt to answer this question using techniques from statistics, information visualization, and machine learning. I begin the journey by cataloging the features of problems delineated by the mathematics and mathematics education communities. These dimensions are explored in a large data set of students working thousands of problems at the Art of Problem Solving, an online company that provides adaptive mathematical training for students around the world. This analysis is able to concretely show how the fabric of mathematical problems changes across different subjects, difficulty levels, and students. Furthermore, it locates problems that stand out in the crowd -- those that synergize cognitive engagement, learning, and difficulty. This quantitatively-heavy side of the dissertation is partnered with a qualitatively-inspired portion that involves human scoring of 105 problems and their solutions. In this setting, I am able to capture elusive features of mathematical problems and derive a fuller picture of the space of mathematical problems. Using correlation matrices, principal components analysis, and clustering techniques, I explore the relationships among those features frequently discussed in mathematics problems (e.g., difficulty, creativity, novelty, affective engagement, authenticity). Along the way, I define a new set of uncorrelated features in problems and use these as the basis for a New Mathematical Problem Typology (NMPT). Grounded in the terminology of classical music, the NMPT works to quickly convey the essence and value of a problem, just as terms like "etude" and "mazurka" do for musicians. Taken together, these quantitative and qualitative analyses seek to terraform the landscape of mathematical problems and, concomitantly, the current thinking

  6. Minimalism as a Guiding Principle: Linking Mathematical Learning to Everyday Knowledge

    Science.gov (United States)

    Inoue, Noriyuki

    2008-01-01

    Studies report that students often fail to consider familiar aspects of reality in solving mathematical word problems. This study explored how different features of mathematical problems influence the way that undergraduate students employ realistic considerations in mathematical problem solving. Incorporating familiar contents in the word…

  7. Students' errors in solving linear equation word problems: Case ...

    African Journals Online (AJOL)

    The study examined errors students make in solving linear equation word problems with a view to expose the nature of these errors and to make suggestions for classroom teaching. A diagnostic test comprising 10 linear equation word problems, was administered to a sample (n=130) of senior high school first year Home ...

  8. The universe in zero words the story of mathematics as told through equations

    CERN Document Server

    Mackenzie, Dana

    2012-01-01

    Most popular books about science, and even about mathematics, tiptoe around equations as if they were something to be hidden from the reader's tender eyes. Dana Mackenzie starts from the opposite premise: He celebrates equations. No history of art would be complete without pictures. Why, then, should a history of mathematics--the universal language of science--keep the masterpieces of the subject hidden behind a veil? The Universe in Zero Words tells the history of twenty-four great and beautiful equations that have shaped mathematics, science, and society--from the elementary

  9. Elementary school students’ mathematical intelligence based on mathematics learning using classical music of the baroque era as the backsound

    Directory of Open Access Journals (Sweden)

    Karlimah

    2018-01-01

    Full Text Available Many studies suggest that classical music can inccrease the listeners’ intelligence, including mathematical intelligence [3, 12, 2, 11]. In this research, we used the classical music of Baroque era as the backsound during math learning. The research method used was quasi experiment with nonequivalent pretest-posttest control group design to grade V SD students in Tasikmalaya city. The results show that the use of classical music of Baroque era during the learning of mathematics gave a high contribution to the mathematical intelligence of fifth grade elementary school students. The student's mathematical intelligence can be seen in the cognitive abilities which were at the high level in the knowledge up to analysis, and at the low level in the synthesis and evaluation. Low mathematical intelligence was shown by students in calculating amount and difference of time, and projecting word problem into the form of mathematical problems. High mathematical intelligence arose in reading and writing integers in words and numbers. Thus, the mathematical intelligence of fifth grade Elementary School students will be better if classical music of Baroque era is used as the backsound in mathematics learning about solving math problems.

  10. What Is the Problem in Problem-Based Learning in Higher Education Mathematics

    Science.gov (United States)

    Dahl, Bettina

    2018-01-01

    Problem and Project-Based Learning (PBL) emphasise collaborate work on problems relevant to society and emphases the relation between theory and practice. PBL fits engineering students as preparation for their future professions but what about mathematics? Mathematics is not just applied mathematics, but it is also a body of abstract knowledge…

  11. Exploring mathematics problem-solving and proof

    CERN Document Server

    Grieser, Daniel

    2018-01-01

    Have you ever faced a mathematical problem and had no idea how to approach it? Or perhaps you had an idea but got stuck halfway through? This book guides you in developing your creativity, as it takes you on a voyage of discovery into mathematics. Readers will not only learn strategies for solving problems and logical reasoning, but they will also learn about the importance of proofs and various proof techniques. Other topics covered include recursion, mathematical induction, graphs, counting, elementary number theory, and the pigeonhole, extremal and invariance principles. Designed to help students make the transition from secondary school to university level, this book provides readers with a refreshing look at mathematics and deep insights into universal principles that are valuable far beyond the scope of this book. Aimed especially at undergraduate and secondary school students as well as teachers, this book will appeal to anyone interested in mathematics. Only basic secondary school mathematics is requi...

  12. A Cognitive Analysis of Students’ Mathematical Problem Solving Ability on Geometry

    Science.gov (United States)

    Rusyda, N. A.; Kusnandi, K.; Suhendra, S.

    2017-09-01

    The purpose of this research is to analyze of mathematical problem solving ability of students in one of secondary school on geometry. This research was conducted by using quantitative approach with descriptive method. Population in this research was all students of that school and the sample was twenty five students that was chosen by purposive sampling technique. Data of mathematical problem solving were collected through essay test. The results showed the percentage of achievement of mathematical problem solving indicators of students were: 1) solve closed mathematical problems with context in math was 50%; 2) solve the closed mathematical problems with the context beyond mathematics was 24%; 3) solving open mathematical problems with contexts in mathematics was 35%; And 4) solving open mathematical problems with contexts outside mathematics was 44%. Based on the percentage, it can be concluded that the level of achievement of mathematical problem solving ability in geometry still low. This is because students are not used to solving problems that measure mathematical problem solving ability, weaknesses remember previous knowledge, and lack of problem solving framework. So the students’ ability of mathematical problems solving need to be improved with implement appropriate learning strategy.

  13. Developing Instructional Mathematical Physics Book Based on Inquiry Approach to Improve Students’ Mathematical Problem Solving Ability

    Directory of Open Access Journals (Sweden)

    Syarifah Fadillah

    2017-03-01

    Full Text Available The problem in this research is to know how the process of developing mathematics physics instructional book based on inquiry approach and its supporting documents to improve students' mathematical problem-solving ability. The purpose of this research is to provide mathematical physics instruction based on inquiry approach and its supporting documents (semester learning activity plan, lesson plan and mathematical problem-solving test to improve students' mathematical problem-solving ability. The development of textbook refers to the ADDIE model, including analysis, design, development, implementation, and evaluation. The validation result from the expert team shows that the textbook and its supporting documents are valid. The test results of the mathematical problem-solving skills show that all test questions are valid and reliable. The result of the incorporation of the textbook in teaching and learning process revealed that students' mathematical problem-solving ability using mathematical physics instruction based on inquiry approach book was better than the students who use the regular book.

  14. Vision in elementary mathematics

    CERN Document Server

    Sawyer, W W

    2003-01-01

    Sure-fire techniques of visualizing, dramatizing, and analyzing numbers promise to attract and retain students' attention and understanding. Topics include basic multiplication and division, algebra, word problems, graphs, negative numbers, fractions, many other practical applications of elementary mathematics. 1964 ed. Answers to Problems.

  15. Investigating Mathematics Teachers Candidates' Knowledge about Problem Solving Strategies through Problem Posing

    Science.gov (United States)

    Ünlü, Melihan

    2017-01-01

    The aim of the study was to determine mathematics teacher candidates' knowledge about problem solving strategies through problem posing. This qualitative research was conducted with 95 mathematics teacher candidates studying at education faculty of a public university during the first term of the 2015-2016 academic year in Turkey. Problem Posing…

  16. What is the problem in problem-based learning in higher education mathematics

    Science.gov (United States)

    Dahl, Bettina

    2018-01-01

    Problem and Project-Based Learning (PBL) emphasise collaborate work on problems relevant to society and emphases the relation between theory and practice. PBL fits engineering students as preparation for their future professions but what about mathematics? Mathematics is not just applied mathematics, but it is also a body of abstract knowledge where the application in society is not always obvious. Does mathematics, including pure mathematics, fit into a PBL curriculum? This paper argues that it does for two reasons: (1) PBL resembles the working methods of research mathematicians. (2) The concept of society includes the society of researchers to whom theoretical mathematics is relevant. The paper describes two cases of university PBL projects in mathematics; one in pure mathematics and the other in applied mathematics. The paper also discusses that future engineers need to understand the world of mathematics as well as how engineers fit into a process of fundamental-research-turned-into-applied-science.

  17. Visual Representations in Mathematics Teaching: An Experiment with Students

    Science.gov (United States)

    Debrenti, Edith

    2015-01-01

    General problem-solving skills are of central importance in school mathematics achievement. Word problems play an important role not just in mathematical education, but in general education as well. Meaningful learning and understanding are basic aspects of all kinds of learning and it is even more important in the case of learning mathematics. In…

  18. The Role of Expository Writing in Mathematical Problem Solving

    Science.gov (United States)

    Craig, Tracy S.

    2016-01-01

    Mathematical problem-solving is notoriously difficult to teach in a standard university mathematics classroom. The project on which this article reports aimed to investigate the effect of the writing of explanatory strategies in the context of mathematical problem solving on problem-solving behaviour. This article serves to describe the…

  19. Following the Template: Transferring Modeling Skills to Nonstandard Problems

    Science.gov (United States)

    Tyumeneva, Yu. A.; Goncharova, M. V.

    2017-01-01

    This study seeks to analyze how students apply a mathematical modeling skill that was previously learned by solving standard word problems to the solution of word problems with nonstandard contexts. During the course of an experiment involving 106 freshmen, we assessed how well they were able to transfer the mathematical modeling skill that is…

  20. Are Mathematics Problems a Problem for Women and Girls?

    Science.gov (United States)

    Schonberger, Ann K.

    The primary questions investigated are: Is it true that males excel in mathematical problem solving and, if so, when does this superiority develop? An examination of recent research showed that sex-related differences did exist, although small, even after controlling for mathematics background. Differences appeared in early adolescence and were…

  1. Pre-service mathematics teachers’ ability in solving well-structured problem

    Science.gov (United States)

    Paradesa, R.

    2018-01-01

    This study aimed to describe the mathematical problem-solving ability of undergraduate students of mathematics education in solving the well-structured problem. The type of this study was qualitative descriptive. The subjects in this study were 100 undergraduate students of Mathematics Education at one of the private universities in Palembang city. The data in this study was collected through two test items with essay form. The results of this study showed that, from the first problem, only 8% students can solve it, but do not check back again to validate the process. Based on a scoring rubric that follows Polya strategy, their answer satisfied 2 4 2 0 patterns. But, from the second problem, 45% students satisfied it. This is because the second problem imitated from the example that was given in learning process. The average score of undergraduate students mathematical problem-solving ability in solving well-structured problems showed 56.00 with standard deviation was 13.22. It means that, from 0 - 100 scale, undergraduate students mathematical problem-solving ability can be categorized low. From this result, the conclusion was undergraduate students of mathematics education in Palembang still have a problem in solving mathematics well-structured problem.

  2. MATHEMATICAL PROBLEMS OF INTEGRATIVE CONTENTS

    Directory of Open Access Journals (Sweden)

    V. Kushnir

    2014-09-01

    Full Text Available The tasks of integrative content requires the use of knowledge and skills on various themes both one discipline and different disciplines. Mostly in the classroom (or in homework the tasks on the properties absorption of different concepts using different theories are considered. Thus knowledge within only one discipline is formed, knowledge of the narrow sense (one subject. Such knowledge is "prescriptional", we call it idealized. After all, it is far from models of the real professional problems and problems of life in general, in order to solve them it is necessary to apply knowledge and skills acquired in different themes of the same objects,life experience. Practical formation of integrative knowledge requires statement of the educational problems before the subjects of studying, the problems within the "narrow objectivity" can not be resolved at all, or such kind of solving is too difficult to solve, for example, the nature and the context of solving problems (scientific approaches to solving problems, creating mathematical models, methods for solving such models, means of solving, application of methods, analysis of the models solution and the right choice, the inspection of solutions, etc. will sink in the conglomeration of technical operations. The problems with integrative content are usually more complicated than the problems of "narrow objectivity." In our problems the index of such difficulty is the essence of educational content, which is disclosed in the previous paragraph. The problems solution proposed in this article requires knowledge of the structural geometry (circle construction, touching two or three laps: with analytic geometry (method of coordinates on the plane; the distance between two points on the coordinate plane; algebra (system drawing irrational equations, method for solving such system, the solution of the system, analysis of the results and the right choose of the desired solution for found criterion, testing

  3. Applied Mathematical Problems in Engineering

    Directory of Open Access Journals (Sweden)

    Carlos Llopis-Albert

    2016-10-01

    Full Text Available There is a close relationship between engineering and mathematics, which has led to the development of new techniques in recent years. Likewise the developments in technology and computers have led to new ways of teaching mathematics for engineering students and the use of modern techniques and methods.  This research aims to provide insight on how to deal with mathematical problems for engineering students. This is performed by means of a fuzzy set/Qualitative Comparative Analysis applied to conflict resolution of Public Participation Projects in support to the EU Water Framework Directive.

  4. Processing of Words Related to the Demands of a Previously Solved Problem

    Directory of Open Access Journals (Sweden)

    Kowalczyk Marek

    2014-06-01

    Full Text Available Earlier research by the author brought about findings suggesting that people in a special way process words related to demands of a problem they previously solved, even when they do not consciously notice this relationship. The findings concerned interference in the task in which the words appeared, a shift in affective responses to them that depended on sex of the participants, and impaired memory of the words. The aim of this study was to replicate these effects and to find out whether they are related to working memory (WM span of the participants, taken as a measure of the individual’s ability to control attention. Participants in the experimental group solved a divergent problem, then performed an ostensibly unrelated speeded affective classification task concerning each of a series of nouns, and then performed an unexpected cued recall task for the nouns. Afterwards, a task measuring WM span was administered. In the control group there was no problem-solving phase. Response latencies for words immediately following problem-related words in the classification task were longer in the experimental than in the control group, but there was no relationship between this effect and WM span. Solving the problem, in interaction with sex of the participants and, independently, with their WM span, influenced affective responses to problem-related words. Recall of these words, however, was not impaired in the experimental group.

  5. An Evaluation of Grades 9 and 10 Mathematics Textbooks Vis-À-Vis ...

    African Journals Online (AJOL)

    the bid system. Besides, the interview result ... Key words: Problem-solving, Mathematics textbook analysis, Heuristics. *An Associate Professor .... 1. Do mathematics textbooks contain problems that require inquiry and discovery methods of ...

  6. Writing and mathematical problem solving in Grade 3

    Directory of Open Access Journals (Sweden)

    Belinda Petersen

    2017-06-01

    Full Text Available This article looks at writing tasks as a methodology to support learners’ mathematical problemsolving strategies in the South African Foundation Phase context. It is a qualitative case study and explores the relation between the use of writing in mathematics and development of learners’ problem-solving strategies and conceptual understanding. The research was conducted in a suburban Foundation Phase school in Cape Town with a class of Grade 3 learners involved in a writing and mathematics intervention. Writing tasks were modelled to learners and implemented by them while they were engaged in mathematical problem solving. Data were gathered from a sample of eight learners of different abilities and included written work, interviews, field notes and audio recordings of ability group discussions. The results revealed an improvement in the strategies and explanations learners used when solving mathematical problems compared to before the writing tasks were implemented. Learners were able to reflect critically on their thinking through their written strategies and explanations. The writing tasks appeared to support learners in providing opportunities to construct and apply mathematical knowledge and skills in their development of problem-solving strategies.

  7. Students’ Mathematical Creative Thinking through Problem Posing Learning

    Science.gov (United States)

    Ulfah, U.; Prabawanto, S.; Jupri, A.

    2017-09-01

    The research aims to investigate the differences in enhancement of students’ mathematical creative thinking ability of those who received problem posing approach assisted by manipulative media and students who received problem posing approach without manipulative media. This study was a quasi experimental research with non-equivalent control group design. Population of this research was third-grade students of a primary school in Bandung city in 2016/2017 academic year. Sample of this research was two classes as experiment class and control class. The instrument used is a test of mathematical creative thinking ability. Based on the results of the research, it is known that the enhancement of the students’ mathematical creative thinking ability of those who received problem posing approach with manipulative media aid is higher than the ability of those who received problem posing approach without manipulative media aid. Students who get learning problem posing learning accustomed in arranging mathematical sentence become matter of story so it can facilitate students to comprehend about story

  8. Improving mathematical problem solving skills through visual media

    Science.gov (United States)

    Widodo, S. A.; Darhim; Ikhwanudin, T.

    2018-01-01

    The purpose of this article was to find out the enhancement of students’ mathematical problem solving by using visual learning media. The ability to solve mathematical problems is the ability possessed by students to solve problems encountered, one of the problem-solving model of Polya. This preliminary study was not to make a model, but it only took a conceptual approach by comparing the various literature of problem-solving skills by linking visual learning media. The results of the study indicated that the use of learning media had not been appropriated so that the ability to solve mathematical problems was not optimal. The inappropriateness of media use was due to the instructional media that was not adapted to the characteristics of the learners. Suggestions that can be given is the need to develop visual media to increase the ability to solve problems.

  9. Mathematical Problems in Biology : Victoria Conference

    CERN Document Server

    1974-01-01

    A conference on "Some Mathematical Problems in Biology" was held at the University of Victoria, Victoria, B. C. , Canada, from May 7 - 10, 1973. The participants and invited speakers were mathematicians interested in problems of a biological nature, and scientists actively engaged in developing mathematical models in biological fields. One aim of the conference was to attempt to assess what the recent rapid growth of mathematical interaction with the biosciences has accomplished and may accomplish in the near future. The conference also aimed to expose the problems of communication bet~",een mathematicians and biological scientists, and in doing so to stimulate the interchange of ideas. It was recognised that the topic spans an enormous breadth, and little attempt was made to balance the very diverse areas. Widespread active interest was shown in the conference, and just over one hundred people registered. The varied departments and institutions across North America from which the participants came made it bo...

  10. Open problems in mathematical physics

    Science.gov (United States)

    Coley, Alan A.

    2017-09-01

    We present a list of open questions in mathematical physics. After a historical introduction, a number of problems in a variety of different fields are discussed, with the intention of giving an overall impression of the current status of mathematical physics, particularly in the topical fields of classical general relativity, cosmology and the quantum realm. This list is motivated by the recent article proposing 42 fundamental questions (in physics) which must be answered on the road to full enlightenment (Allen and Lidstrom 2017 Phys. Scr. 92 012501). But paraphrasing a famous quote by the British football manager Bill Shankly, in response to the question of whether mathematics can answer the Ultimate Question of Life, the Universe, and Everything, mathematics is, of course, much more important than that.

  11. Teaching Problem Solving to Students Receiving Tiered Interventions Using the Concrete-Representational-Abstract Sequence and Schema-Based Instruction

    Science.gov (United States)

    Flores, Margaret M.; Hinton, Vanessa M.; Burton, Megan E.

    2016-01-01

    Mathematical word problems are the most common form of mathematics problem solving implemented in K-12 schools. Identifying key words is a frequent strategy taught in classrooms in which students struggle with problem solving and show low success rates in mathematics. Researchers show that using the concrete-representational-abstract (CRA)…

  12. Open-Start Mathematics Problems: An Approach to Assessing Problem Solving

    Science.gov (United States)

    Monaghan, John; Pool, Peter; Roper, Tom; Threlfall, John

    2009-01-01

    This article describes one type of mathematical problem, open-start problems, and discusses their potential for use in assessment. In open-start problems how one starts to address the problem can vary but they have a correct answer. We argue that the use of open-start problems in assessment could positively influence classroom mathematics…

  13. African Journal of Educational Studies in Mathematics and Sciences ...

    African Journals Online (AJOL)

    African Journal of Educational Studies in Mathematics and Sciences. ... on senior high school students' proficiency in solving linear equation word problems ... from parents and teachers' influence on students' mathematics-related self-beliefs ...

  14. ABC Problem in Elementary Mathematics Education: Arithmetic "before" Comprehension

    Science.gov (United States)

    Boote, Stacy K.; Boote, David N.

    2018-01-01

    Mathematical habits of prospective teachers affect problem comprehension and success and expose their beliefs about mathematics. Prospective elementary teachers (PSTs) (n = 121) engaged in a problem solving activity each week in class. Data were collected from PSTs enrolled in an undergraduate elementary mathematics methods course at a…

  15. Mathematical Problem Solving Ability of Junior High School Students through Ang’s Framework for Mathematical Modelling Instruction

    Science.gov (United States)

    Fasni, N.; Turmudi, T.; Kusnandi, K.

    2017-09-01

    This research background of this research is the importance of student problem solving abilities. The purpose of this study is to find out whether there are differences in the ability to solve mathematical problems between students who have learned mathematics using Ang’s Framework for Mathematical Modelling Instruction (AFFMMI) and students who have learned using scientific approach (SA). The method used in this research is a quasi-experimental method with pretest-postest control group design. Data analysis of mathematical problem solving ability using Indepent Sample Test. The results showed that there was a difference in the ability to solve mathematical problems between students who received learning with Ang’s Framework for Mathematical Modelling Instruction and students who received learning with a scientific approach. AFFMMI focuses on mathematical modeling. This modeling allows students to solve problems. The use of AFFMMI is able to improve the solving ability.

  16. Open problems in mathematical physics

    International Nuclear Information System (INIS)

    Coley, Alan A

    2017-01-01

    We present a list of open questions in mathematical physics. After a historical introduction, a number of problems in a variety of different fields are discussed, with the intention of giving an overall impression of the current status of mathematical physics, particularly in the topical fields of classical general relativity, cosmology and the quantum realm. This list is motivated by the recent article proposing 42 fundamental questions (in physics) which must be answered on the road to full enlightenment (Allen and Lidstrom 2017 Phys. Scr . 92 012501). But paraphrasing a famous quote by the British football manager Bill Shankly, in response to the question of whether mathematics can answer the Ultimate Question of Life, the Universe, and Everything, mathematics is, of course, much more important than that. (invited comment)

  17. Addressing Mathematization Obstacles with Unformalized Problems in Physics Education

    DEFF Research Database (Denmark)

    Niss, Martin

    2018-01-01

    Abstract: Solving a physics problem requires that the problem solver either implicitly or explicitly structure the problem situation in such a way that she can set up the mathematical equations based on the relevant physics. This part of the mathematization process has been shown to cause obstacles...... for students (Niss, 2016). In the paper, we show how the students’ ability to perform this mathematization process can be trained by using so-called unformalized physics problems. Some examples of how this training can be done are provided from a course on problem solving in physics taught at Roskilde...

  18. Pre-Service Mathematics Teachers’ Problem Solving Processes with Geometer’s Sketchpad: Mirror Problem

    OpenAIRE

    ÖÇAL, Mehmet Fatih; ŞİMŞEK, Mertkan

    2016-01-01

    Problem solving skill is the core of mathematics education and its importance cannot be denied. This study specifically examined 56 freshmen pre-service mathematics teachers’ problem solving processes on a specific problem with the help of Geometer’s Sketchpad (GSP). They were grouped into two-person teams to solve a problem called "the mirror problem". They were expected to solve it by means of GSP. According to their works on GSP and related reflections, there appeared two differe...

  19. Problems in mathematical analysis III integration

    CERN Document Server

    Kaczor, W J

    2003-01-01

    We learn by doing. We learn mathematics by doing problems. This is the third volume of Problems in Mathematical Analysis. The topic here is integration for real functions of one real variable. The first chapter is devoted to the Riemann and the Riemann-Stieltjes integrals. Chapter 2 deals with Lebesgue measure and integration. The authors include some famous, and some not so famous, integral inequalities related to Riemann integration. Many of the problems for Lebesgue integration concern convergence theorems and the interchange of limits and integrals. The book closes with a section on Fourier series, with a concentration on Fourier coefficients of functions from particular classes and on basic theorems for convergence of Fourier series. The book is primarily geared toward students in analysis, as a study aid, for problem-solving seminars, or for tutorials. It is also an excellent resource for instructors who wish to incorporate problems into their lectures. Solutions for the problems are provided in the boo...

  20. THE QUALITY OF MATHEMATICAL PROBLEMS - EVALUATION AND SELF-EVALUATION

    Directory of Open Access Journals (Sweden)

    Patáková, Eva

    2013-09-01

    Full Text Available The research presented in the article consists of two parts. Firstly, opinions on mathematical problem quality are explored within four groups of participants (novices, specialists and experts in problem posing; high school students who never posed their own problems. Secondly, self-reflections written by the participants who have some experience in problem posing (novices, specialists and experts are explored and compared with the general view of problem quality received in the first part of the research. The more experienced problem posers have more requirements on problem quality (both as general requirements and within their own work on posing problems. There is a slight decrease in ability to notice important features of mathematical problem quality after the first experience in problem posing. Experts lay stress on mathematical features of the problem whilst novices and specialists more on problem – student interaction.

  1. Math in plain english literacy strategies for the mathematics classroom

    CERN Document Server

    Benjamin, Amy

    2013-01-01

    Do word problems and math vocabulary confuse students in your mathematics classes? Do simple keywords like ""value"" and ""portion"" seem to mislead them? Many words that students already know can have a different meaning in mathematics. To grasp that difference, students need to connect English literacy skills to math. Successful students speak, read, write, and listen to each other so they can understand, retain, and apply mathematics concepts. This book explains how to use 10 classroom-ready literacy strategies in concert with your mathematics instruction. You'll learn how to develop stude

  2. Developing Mathematics Problems Based on Pisa Level

    Directory of Open Access Journals (Sweden)

    Shahibul Ahyan

    2014-01-01

    Full Text Available This research aims to produce mathematics problems based on PISA level with valid and practical content of change and relationships and has potential effect for Junior High School students. A development research method developed by Akker, Gravemeijer, McKenney and Nieveen is used this research. In the first stage, the researcher analyzed students, algebra material in school-based curricula (KTSP and mathematics problems of PISA 2003 of change and relationships content. The second stage, the researcher designed 13 problems with content of change and relationships. The last, the researcher used formative evaluation design developed by Tessmer which includes self evaluation, one-to-one, expert review, small group, and field test. The data collect by walk through, interview, and questionnaire. The result of this research indicated that 12 mathematical problems based on PISA level of change and relationships content that developed have validity, practically, and potential effects for Junior High School students.

  3. Exercises and problems in mathematical methods of physics

    CERN Document Server

    Cicogna, Giampaolo

    2018-01-01

    This book presents exercises and problems in the mathematical methods of physics with the aim of offering undergraduate students an alternative way to explore and fully understand the mathematical notions on which modern physics is based. The exercises and problems are proposed not in a random order but rather in a sequence that maximizes their educational value. Each section and subsection starts with exercises based on first definitions, followed by groups of problems devoted to intermediate and, subsequently, more elaborate situations. Some of the problems are unavoidably "routine", but others bring to the forenontrivial properties that are often omitted or barely mentioned in textbooks. There are also problems where the reader is guided to obtain important results that are usually stated in textbooks without complete proofs. In all, some 350 solved problems covering all mathematical notions useful to physics are included. While the book is intended primarily for undergraduate students of physics, students...

  4. Pattern of mathematic representation ability in magnetic electricity problem

    Science.gov (United States)

    Hau, R. R. H.; Marwoto, P.; Putra, N. M. D.

    2018-03-01

    The mathematic representation ability in solving magnetic electricity problem gives information about the way students understand magnetic electricity. Students have varied mathematic representation pattern ability in solving magnetic electricity problem. This study aims to determine the pattern of students' mathematic representation ability in solving magnet electrical problems.The research method used is qualitative. The subject of this study is the fourth semester students of UNNES Physics Education Study Program. The data collection is done by giving a description test that refers to the test of mathematical representation ability and interview about field line topic and Gauss law. The result of data analysis of student's mathematical representation ability in solving magnet electric problem is categorized into high, medium and low category. The ability of mathematical representations in the high category tends to use a pattern of making known and asked symbols, writing equations, using quantities of physics, substituting quantities into equations, performing calculations and final answers. The ability of mathematical representation in the medium category tends to use several patterns of writing the known symbols, writing equations, using quantities of physics, substituting quantities into equations, performing calculations and final answers. The ability of mathematical representations in the low category tends to use several patterns of making known symbols, writing equations, substituting quantities into equations, performing calculations and final answer.

  5. Problem posing as a didactic resource in formal mathematics courses to train future secondary school mathematics teachers

    Directory of Open Access Journals (Sweden)

    Lorena Salazar Solórzano

    2015-06-01

    Full Text Available Beginning university training programs must focus on different competencies for mathematics teachers, i.e., not only on solving problems, but also on posing them and analyzing the mathematical activity. This paper reports the results of an exploratory study conducted with future secondary school mathematics teachers on the introduction of problem-posing tasks in formal mathematics courses, specifically in abstract algebra and real analysis courses. Evidence was found that training which includes problem-posing tasks has a positive impact on the students’ understanding of definitions, theorems and exercises within formal mathematics, as well as on their competency in reflecting on the mathematical activity. 

  6. Formal languages, automata and numeration systems introduction to combinatorics on words

    CERN Document Server

    Rigo, Michel

    2014-01-01

    Formal Languages, Automaton and Numeration Systems presents readers with a review of research related to formal language theory, combinatorics on words or numeration systems, such as Words, DLT (Developments in Language Theory), ICALP, MFCS (Mathematical Foundation of Computer Science), Mons Theoretical Computer Science Days, Numeration, CANT (Combinatorics, Automata and Number Theory). Combinatorics on words deals with problems that can be stated in a non-commutative monoid, such as subword complexity of finite or infinite words, construction and properties of infinite words, unavoidabl

  7. Leveling of Critical Thinking Abilities of Students of Mathematics Education in Mathematical Problem Solving

    Science.gov (United States)

    Rasiman

    2015-01-01

    This research aims to determine the leveling of critical thinking abilities of students of mathematics education in mathematical problem solving. It includes qualitative-explorative study that was conducted at University of PGRI Semarang. The generated data in the form of information obtained problem solving question and interview guides. The…

  8. Rethinking Mathematics Teaching in Liberia: Realistic Mathematics Education

    Science.gov (United States)

    Stemn, Blidi S.

    2017-01-01

    In some African cultures, the concept of division does not necessarily mean sharing money or an item equally. How an item is shared might depend on the ages of the individuals involved. This article describes the use of the Realistic Mathematics Education (RME) approach to teach division word problems involving money in a 3rd-grade class in…

  9. COMPUTER TOOLS OF DYNAMIC MATHEMATIC SOFTWARE AND METHODICAL PROBLEMS OF THEIR USE

    OpenAIRE

    Olena V. Semenikhina; Maryna H. Drushliak

    2014-01-01

    The article presents results of analyses of standard computer tools of dynamic mathematic software which are used in solving tasks, and tools on which the teacher can support in the teaching of mathematics. Possibility of the organization of experimental investigating of mathematical objects on the basis of these tools and the wording of new tasks on the basis of the limited number of tools, fast automated check are specified. Some methodological comments on application of computer tools and ...

  10. Pre-Service Class Teacher' Ability in Solving Mathematical Problems and Skills in Solving Daily Problems

    Science.gov (United States)

    Aljaberi, Nahil M.; Gheith, Eman

    2016-01-01

    This study aims to investigate the ability of pre-service class teacher at University of Petrain solving mathematical problems using Polya's Techniques, their level of problem solving skills in daily-life issues. The study also investigates the correlation between their ability to solve mathematical problems and their level of problem solving…

  11. Word-based Morphology: Some Problems from a Polysynthetic Language.

    Science.gov (United States)

    Axelrod, Melissa

    Some of the problems inherent in a word-based hypothesis asserting that the word/stem is taken as the minimal sign not only for syntax but also for morphology are examined in an analysis of a polysynthetic language, Koyukon, an Athabaskan language of Alaska. Data from the Central dialect is considered in the analysis. A brief sketch of the verbal…

  12. The effects of presenting multidigit mathematics problems in a realistic context on sixth graders' problem solving

    NARCIS (Netherlands)

    Hickendorff, M.

    2013-01-01

    Mathematics education and assessments increasingly involve arithmetic problems presented in context: a realistic situation that requires mathematical modeling. This study assessed the effects of such typical school mathematics contexts on two aspects of problem solving: performance and strategy use.

  13. Problem solving of student with visual impairment related to mathematical literacy problem

    Science.gov (United States)

    Pratama, A. R.; Saputro, D. R. S.; Riyadi

    2018-04-01

    The student with visual impairment, total blind category depends on the sense of touch and hearing in obtaining information. In fact, the two senses can receive information less than 20%. Thus, students with visual impairment of the total blind categories in the learning process must have difficulty, including learning mathematics. This study aims to describe the problem-solving process of the student with visual impairment, total blind category on mathematical literacy issues based on Polya phase. This research using test method similar problems mathematical literacy in PISA and in-depth interviews. The subject of this study was a student with visual impairment, total blind category. Based on the result of the research, problem-solving related to mathematical literacy based on Polya phase is quite good. In the phase of understanding the problem, the student read about twice by brushing the text and assisted with information through hearing three times. The student with visual impairment in problem-solving based on the Polya phase, devising a plan by summoning knowledge and experience gained previously. At the phase of carrying out the plan, students with visual impairment implement the plan in accordance with pre-made. In the looking back phase, students with visual impairment need to check the answers three times but have not been able to find a way.

  14. The philosophical aspect of learning inverse problems of mathematical physics

    Directory of Open Access Journals (Sweden)

    Виктор Семенович Корнилов

    2018-12-01

    Full Text Available The article describes specific questions student learning inverse problems of mathematical physics. When teaching inverse problems of mathematical physics to the understanding of the students brought the information that the inverse problems of mathematical physics with a philosophical point of view are the problems of determining the unknown causes of known consequences, and the search for their solutions have great scientific and educational potential. The reasons are specified in the form of unknown coefficients, right side, initial conditions of the mathematical model of inverse problems, and as a consequence are functionals of the solution of this mathematical model. In the process of learning the inverse problems of mathematical physics focuses on the philosophical aspects of the phenomenon of information and identify cause-effect relations. It is emphasized that in the process of logical analysis applied and humanitarian character, students realize that information is always related to the fundamental philosophical questions that the analysis applied and the humanitarian aspects of the obtained results the inverse problem of mathematical physics allows students to make appropriate inferences about the studied process and to, ultimately, new information, to study its properties and understand its value. Philosophical understanding of the notion of information opens up to students a new methodological opportunities to comprehend the world and helps us to reinterpret existing science and philosophy of the theory related to the disclosure of the interrelationship of all phenomena of reality.

  15. Calculus Problem Solving Behavior of Mathematic Education Students

    Science.gov (United States)

    Rizal, M.; Mansyur, J.

    2017-04-01

    The purpose of this study is to obtain a description of the problem-solving behaviour of mathematics education students. The attainment of the purpose consisted of several stages: (1) to gain the subject from the mathematic education of first semester students, each of them who has a high, medium, and low competence of mathematic case. (2) To give two mathematical problems with different characteristics. The first problem (M1), the statement does not lead to a resolution. The second problem (M2), a statement leads to problem-solving. (3) To explore the behaviour of problem-solving based on the step of Polya (Rizal, 2011) by way of thinking aloud and in-depth interviews. The obtained data are analysed as suggested by Miles and Huberman (1994) but at first, time triangulation is done or data’s credibility by providing equivalent problem contexts and at different times. The results show that the behavioral problem solvers (mathematic education students) who are capable of high mathematic competency (ST). In understanding M1, ST is more likely to pay attention to an image first, read the texts piecemeal and repeatedly, then as a whole and more focus to the sentences that contain equations, numbers or symbols. As a result, not all information can be received well. When understanding the M2, ST can link the information from a problem that is stored in the working memory to the information on the long-term memory. ST makes planning to the solution of M1 and M2 by using a formula based on similar experiences which have been ever received before. Another case when implementing the troubleshooting plans, ST complete the M1 according to the plan, but not all can be resolved correctly. In contrast to the implementation of the solving plan of M2, ST can solve the problem according to plan quickly and correctly. According to the solving result of M1 and M2, ST conducts by reading the job based on an algorithm and reasonability. Furthermore, when SS and SR understand the

  16. The Relationship between Students' Performance on Conventional Standardized Mathematics Assessments and Complex Mathematical Modeling Problems

    Science.gov (United States)

    Kartal, Ozgul; Dunya, Beyza Aksu; Diefes-Dux, Heidi A.; Zawojewski, Judith S.

    2016-01-01

    Critical to many science, technology, engineering, and mathematics (STEM) career paths is mathematical modeling--specifically, the creation and adaptation of mathematical models to solve problems in complex settings. Conventional standardized measures of mathematics achievement are not structured to directly assess this type of mathematical…

  17. Problem Posing with Realistic Mathematics Education Approach in Geometry Learning

    Science.gov (United States)

    Mahendra, R.; Slamet, I.; Budiyono

    2017-09-01

    One of the difficulties of students in the learning of geometry is on the subject of plane that requires students to understand the abstract matter. The aim of this research is to determine the effect of Problem Posing learning model with Realistic Mathematics Education Approach in geometry learning. This quasi experimental research was conducted in one of the junior high schools in Karanganyar, Indonesia. The sample was taken using stratified cluster random sampling technique. The results of this research indicate that the model of Problem Posing learning with Realistic Mathematics Education Approach can improve students’ conceptual understanding significantly in geometry learning especially on plane topics. It is because students on the application of Problem Posing with Realistic Mathematics Education Approach are become to be active in constructing their knowledge, proposing, and problem solving in realistic, so it easier for students to understand concepts and solve the problems. Therefore, the model of Problem Posing learning with Realistic Mathematics Education Approach is appropriately applied in mathematics learning especially on geometry material. Furthermore, the impact can improve student achievement.

  18. Advanced Problems in Mathematics : Preparing for University

    OpenAIRE

    Siklos, Stephen

    2016-01-01

    " This book is intended to help candidates prepare for entrance examinations in mathematics and scientific subjects, including STEP (Sixth Term Examination Paper). STEP is an examination used by Cambridge colleges as the basis for conditional offers. They are also used by Warwick University, and many other mathematics departments recommend that their applicants practice on the past papers even if they do not take the examination. Advanced Problems in Mathematics is recommended as preparati...

  19. On Teaching Problem Solving in School Mathematics

    Directory of Open Access Journals (Sweden)

    Erkki Pehkonen

    2013-12-01

    Full Text Available The article begins with a brief overview of the situation throughout the world regarding problem solving. The activities of the ProMath group are then described, as the purpose of this international research group is to improve mathematics teaching in school. One mathematics teaching method that seems to be functioning in school is the use of open problems (i.e., problem fields. Next we discuss the objectives of the Finnish curriculum that are connected with problem solving. Some examples and research results are taken from a Finnish–Chilean research project that monitors the development of problem-solving skills in third grade pupils. Finally, some ideas on “teacher change” are put forward. It is not possible to change teachers, but only to provide hints for possible change routes: the teachers themselves should work out the ideas and their implementation.

  20. Assessing the Effect of Language Demand in Bundles of Math Word Problems

    Science.gov (United States)

    Banks, Kathleen; Jeddeeni, Ahmad; Walker, Cindy M.

    2016-01-01

    Differential bundle functioning (DBF) analyses were conducted to determine whether seventh and eighth grade second language learners (SLLs) had lower probabilities of answering bundles of math word problems correctly that had heavy language demands, when compared to non-SLLs of equal math proficiency. Math word problems on each of four test forms…

  1. Investigating and Developing Engineering Students' Mathematical Modelling and Problem-Solving Skills

    Science.gov (United States)

    Wedelin, Dag; Adawi, Tom; Jahan, Tabassum; Andersson, Sven

    2015-01-01

    How do engineering students approach mathematical modelling problems and how can they learn to deal with such problems? In the context of a course in mathematical modelling and problem solving, and using a qualitative case study approach, we found that the students had little prior experience of mathematical modelling. They were also inexperienced…

  2. Understanding the Problems of Learning Mathematics.

    Science.gov (United States)

    Semilla-Dube, Lilia

    1983-01-01

    A model is being developed to categorize problems in teaching and learning mathematics. Categories include problems due to language difficulties, lack of prerequisite knowledge, and those related to the affective domain. This paper calls on individuals to share teaching and learning episodes; those submitted will then be compiled and categorized.…

  3. Plato's problem an introduction to mathematical platonism

    CERN Document Server

    Panza, M

    2013-01-01

    What is mathematics about? And how can we have access to the reality it is supposed to describe? The book tells the story of this problem, first raised by Plato, through the views of Aristotle, Proclus, Kant, Frege, Gödel, Benacerraf, up to the most recent debate on mathematical platonism.

  4. Using the Wonder of Inequalities between Averages for Mathematics Problems Solving

    Science.gov (United States)

    Shaanan, Rachel Mogilevsky; Gordon, Moshe Stupel

    2016-01-01

    The study presents an introductory idea of using mathematical averages as a tool for enriching mathematical problem solving. Throughout students' activities, a research was conducted on their ability to solve mathematical problems, and how to cope with a variety of mathematical tasks, in a variety of ways, using the skills, tools and experiences…

  5. Methods of solving nonstandard problems

    CERN Document Server

    Grigorieva, Ellina

    2015-01-01

    This book, written by an accomplished female mathematician, is the second to explore nonstandard mathematical problems – those that are not directly solved by standard mathematical methods but instead rely on insight and the synthesis of a variety of mathematical ideas.   It promotes mental activity as well as greater mathematical skills, and is an ideal resource for successful preparation for the mathematics Olympiad. Numerous strategies and techniques are presented that can be used to solve intriguing and challenging problems of the type often found in competitions.  The author uses a friendly, non-intimidating approach to emphasize connections between different fields of mathematics and often proposes several different ways to attack the same problem.  Topics covered include functions and their properties, polynomials, trigonometric and transcendental equations and inequalities, optimization, differential equations, nonlinear systems, and word problems.   Over 360 problems are included with hints, ...

  6. Using Video Prompting to Teach Mathematical Problem Solving of Real-World Video-Simulation Problems

    Science.gov (United States)

    Saunders, Alicia F.; Spooner, Fred; Ley Davis, Luann

    2018-01-01

    Mathematical problem solving is necessary in many facets of everyday life, yet little research exists on how to teach students with more severe disabilities higher order mathematics like problem solving. Using a multiple probe across participants design, three middle school students with moderate intellectual disability (ID) were taught to solve…

  7. Strategy Instruction in Mathematics.

    Science.gov (United States)

    Goldman, Susan R.

    1989-01-01

    Experiments in strategy instruction for mathematics have been conducted using three models (direct instruction, self-instruction, and guided learning) applied to the tasks of computation and word problem solving. Results have implications for effective strategy instruction for learning disabled students. It is recommended that strategy instruction…

  8. A mathematical approach to research problems of science and technology theoretical basis and developments in mathematical modeling

    CERN Document Server

    Ei, Shin-ichiro; Koiso, Miyuki; Ochiai, Hiroyuki; Okada, Kanzo; Saito, Shingo; Shirai, Tomoyuki

    2014-01-01

    This book deals with one of the most novel advances in mathematical modeling for applied scientific technology, including computer graphics, public-key encryption, data visualization, statistical data analysis, symbolic calculation, encryption, error correcting codes, and risk management. It also shows that mathematics can be used to solve problems from nature, e.g., slime mold algorithms. One of the unique features of this book is that it shows readers how to use pure and applied mathematics, especially those mathematical theory/techniques developed in the twentieth century, and developing now, to solve applied problems in several fields of industry. Each chapter includes clues on how to use "mathematics" to solve concrete problems faced in industry as well as practical applications. The target audience is not limited to researchers working in applied mathematics and includes those in engineering, material sciences, economics, and life sciences.

  9. Science modelling in pre-calculus: how to make mathematics problems contextually meaningful

    Science.gov (United States)

    Sokolowski, Andrzej; Yalvac, Bugrahan; Loving, Cathleen

    2011-04-01

    'Use of mathematical representations to model and interpret physical phenomena and solve problems is one of the major teaching objectives in high school math curriculum' (National Council of Teachers of Mathematics (NCTM), Principles and Standards for School Mathematics, NCTM, Reston, VA, 2000). Commonly used pre-calculus textbooks provide a wide range of application problems. However, these problems focus students' attention on evaluating or solving pre-arranged formulas for given values. The role of scientific content is reduced to provide a background for these problems instead of being sources of data gathering for inducing mathematical tools. Students are neither required to construct mathematical models based on the contexts nor are they asked to validate or discuss the limitations of applied formulas. Using these contexts, the instructor may think that he/she is teaching problem solving, where in reality he/she is teaching algorithms of the mathematical operations (G. Kulm (ed.), New directions for mathematics assessment, in Assessing Higher Order Thinking in Mathematics, Erlbaum, Hillsdale, NJ, 1994, pp. 221-240). Without a thorough representation of the physical phenomena and the mathematical modelling processes undertaken, problem solving unintentionally appears as simple algorithmic operations. In this article, we deconstruct the representations of mathematics problems from selected pre-calculus textbooks and explicate their limitations. We argue that the structure and content of those problems limits students' coherent understanding of mathematical modelling, and this could result in weak student problem-solving skills. Simultaneously, we explore the ways to enhance representations of those mathematical problems, which we have characterized as lacking a meaningful physical context and limiting coherent student understanding. In light of our discussion, we recommend an alternative to strengthen the process of teaching mathematical modelling - utilization

  10. Scientific Approach to Improve Mathematical Problem Solving Skills Students of Grade V

    Science.gov (United States)

    Roheni; Herman, T.; Jupri, A.

    2017-09-01

    This study investigates the skills of elementary school students’ in problem solving through the Scientific Approach. The purpose of this study is to determine mathematical problem solving skills of students by using Scientific Approach is better than mathematical problem solving skills of students by using Direct Instruction. This study is using quasi-experimental method. Subject of this study is students in grade V in one of state elementary school in Cirebon Regency. Instrument that used in this study is mathematical problem solving skills. The result of this study showed that mathematical problem solving skills of students who learn by using Scientific Approach is more significant than using Direct Instruction. Base on result and analysis, the conclusion is that Scientific Approach can improve students’ mathematical problem solving skills.

  11. An Investigation of Secondary Teachers’ Understanding and Belief on Mathematical Problem Solving

    Science.gov (United States)

    Yuli Eko Siswono, Tatag; Wachidul Kohar, Ahmad; Kurniasari, Ika; Puji Astuti, Yuliani

    2016-02-01

    Weaknesses on problem solving of Indonesian students as reported by recent international surveys give rise to questions on how Indonesian teachers bring out idea of problem solving in mathematics lesson. An explorative study was undertaken to investigate how secondary teachers who teach mathematics at junior high school level understand and show belief toward mathematical problem solving. Participants were teachers from four cities in East Java province comprising 45 state teachers and 25 private teachers. Data was obtained through questionnaires and written test. The results of this study point out that the teachers understand pedagogical problem solving knowledge well as indicated by high score of observed teachers‘ responses showing understanding on problem solving as instruction as well as implementation of problem solving in teaching practice. However, they less understand on problem solving content knowledge such as problem solving strategies and meaning of problem itself. Regarding teacher's difficulties, teachers admitted to most frequently fail in (1) determining a precise mathematical model or strategies when carrying out problem solving steps which is supported by data of test result that revealed transformation error as the most frequently observed errors in teachers’ work and (2) choosing suitable real situation when designing context-based problem solving task. Meanwhile, analysis of teacher's beliefs on problem solving shows that teachers tend to view both mathematics and how students should learn mathematics as body static perspective, while they tend to believe to apply idea of problem solving as dynamic approach when teaching mathematics.

  12. DOE Fundamentals Handbook: Mathematics, Volume 1

    International Nuclear Information System (INIS)

    1992-06-01

    The Mathematics Fundamentals Handbook was developed to assist nuclear facility operating contractors provide operators, maintenance personnel, and the technical staff with the necessary fundamentals training to ensure a basic understanding of mathematics and its application to facility operation. The handbook includes a review of introductory mathematics and the concepts and functional use of algebra, geometry, trigonometry, and calculus. Word problems, equations, calculations, and practical exercises that require the use of each of the mathematical concepts are also presented. This information will provide personnel with a foundation for understanding and performing basic mathematical calculations that are associated with various DOE nuclear facility operations

  13. DOE Fundamentals Handbook: Mathematics, Volume 2

    International Nuclear Information System (INIS)

    1992-06-01

    The Mathematics Fundamentals Handbook was developed to assist nuclear facility operating contractors provide operators, maintenance personnel, and the technical staff with the necessary fundamentals training to ensure a basic understanding of mathematics and its application to facility operation. The handbook includes a review of introductory mathematics and the concepts and functional use of algebra, geometry, trigonometry, and calculus. Word problems, equations, calculations, and practical exercises that require the use of each of the mathematical concepts are also presented. This information will provide personnel with a foundation for understanding and performing basic mathematical calculations that are associated with various DOE nuclear facility operations

  14. How to solve applied mathematics problems

    CERN Document Server

    Moiseiwitsch, B L

    2011-01-01

    This workbook bridges the gap between lectures and practical applications, offering students of mathematics, engineering, and physics the chance to practice solving problems from a wide variety of fields. 2011 edition.

  15. Developing non-routine problems for assessing students’ mathematical literacy

    Science.gov (United States)

    Murdiyani, N. M.

    2018-03-01

    The purpose of this study is to develop non-routine problems for assessing the mathematics literacy skills of students, which is valid, practical, and effective. It is based on the previous research said that Indonesian students’ mathematical literacy is still low. The results of this study can be used as a guide in developing the evaluation questions that can train students to improve the ability of solving non-routine problems in everyday life. This research type is formative evaluation that consists of preliminary, self evaluation, expert reviews, one-to-one, small group, and field test. The sample of this research is grade 8 students at one of Junior High School in Yogyakarta. This study results in mathematics literacy problems prototype consisting of level 1 to level 6 problems similar to PISA problems. This study also discusses the examples of students’ answer and their reasoning.

  16. Effectiveness of discovery learning model on mathematical problem solving

    Science.gov (United States)

    Herdiana, Yunita; Wahyudin, Sispiyati, Ririn

    2017-08-01

    This research is aimed to describe the effectiveness of discovery learning model on mathematical problem solving. This research investigate the students' problem solving competency before and after learned by using discovery learning model. The population used in this research was student in grade VII in one of junior high school in West Bandung Regency. From nine classes, class VII B were randomly selected as the sample of experiment class, and class VII C as control class, which consist of 35 students every class. The method in this research was quasi experiment. The instrument in this research is pre-test, worksheet and post-test about problem solving of mathematics. Based on the research, it can be conclude that the qualification of problem solving competency of students who gets discovery learning model on level 80%, including in medium category and it show that discovery learning model effective to improve mathematical problem solving.

  17. Obstacle problems in mathematical physics

    CERN Document Server

    Rodrigues, J-F

    1987-01-01

    The aim of this research monograph is to present a general account of the applicability of elliptic variational inequalities to the important class of free boundary problems of obstacle type from a unifying point of view of classical Mathematical Physics.The first part of the volume introduces some obstacle type problems which can be reduced to variational inequalities. Part II presents some of the main aspects of the theory of elliptic variational inequalities, from the abstract hilbertian framework to the smoothness of the variational solution, discussing in general the properties of the free boundary and including some results on the obstacle Plateau problem. The last part examines the application to free boundary problems, namely the lubrication-cavitation problem, the elastoplastic problem, the Signorini (or the boundary obstacle) problem, the dam problem, the continuous casting problem, the electrochemical machining problem and the problem of the flow with wake in a channel past a profile.

  18. Teaching Elementary Mathematics through Problem Solving and Its Relationship to Mathematics Achievement

    Science.gov (United States)

    Bullock, Audrey N.

    2017-01-01

    Problem solving in mathematics has been a goal for students for decades. In the reviewed literature, problem solving was most often treated as the dependent variable and was defined very broadly; however, few studies were found that included problem solving as a treatment or independent variable. The purpose of this study was to investigate the…

  19. Student’s scheme in solving mathematics problems

    Science.gov (United States)

    Setyaningsih, Nining; Juniati, Dwi; Suwarsono

    2018-03-01

    The purpose of this study was to investigate students’ scheme in solving mathematics problems. Scheme are data structures for representing the concepts stored in memory. In this study, we used it in solving mathematics problems, especially ratio and proportion topics. Scheme is related to problem solving that assumes that a system is developed in the human mind by acquiring a structure in which problem solving procedures are integrated with some concepts. The data were collected by interview and students’ written works. The results of this study revealed are students’ scheme in solving the problem of ratio and proportion as follows: (1) the content scheme, where students can describe the selected components of the problem according to their prior knowledge, (2) the formal scheme, where students can explain in construct a mental model based on components that have been selected from the problem and can use existing schemes to build planning steps, create something that will be used to solve problems and (3) the language scheme, where students can identify terms, or symbols of the components of the problem.Therefore, by using the different strategies to solve the problems, the students’ scheme in solving the ratio and proportion problems will also differ.

  20. How students process equations in solving quantitative synthesis problems? Role of mathematical complexity in students’ mathematical performance

    Directory of Open Access Journals (Sweden)

    Bashirah Ibrahim

    2017-10-01

    Full Text Available We examine students’ mathematical performance on quantitative “synthesis problems” with varying mathematical complexity. Synthesis problems are tasks comprising multiple concepts typically taught in different chapters. Mathematical performance refers to the formulation, combination, and simplification of equations. Generally speaking, formulation and combination of equations require conceptual reasoning; simplification of equations requires manipulation of equations as computational tools. Mathematical complexity is operationally defined by the number and the type of equations to be manipulated concurrently due to the number of unknowns in each equation. We use two types of synthesis problems, namely, sequential and simultaneous tasks. Sequential synthesis tasks require a chronological application of pertinent concepts, and simultaneous synthesis tasks require a concurrent application of the pertinent concepts. A total of 179 physics major students from a second year mechanics course participated in the study. Data were collected from written tasks and individual interviews. Results show that mathematical complexity negatively influences the students’ mathematical performance on both types of synthesis problems. However, for the sequential synthesis tasks, it interferes only with the students’ simplification of equations. For the simultaneous synthesis tasks, mathematical complexity additionally impedes the students’ formulation and combination of equations. Several reasons may explain this difference, including the students’ different approaches to the two types of synthesis problems, cognitive load, and the variation of mathematical complexity within each synthesis type.

  1. Improving mathematical problem solving ability through problem-based learning and authentic assessment for the students of Bali State Polytechnic

    Science.gov (United States)

    Darma, I. K.

    2018-01-01

    This research is aimed at determining: 1) the differences of mathematical problem solving ability between the students facilitated with problem-based learning model and conventional learning model, 2) the differences of mathematical problem solving ability between the students facilitated with authentic and conventional assessment model, and 3) interaction effect between learning and assessment model on mathematical problem solving. The research was conducted in Bali State Polytechnic, using the 2x2 experiment factorial design. The samples of this research were 110 students. The data were collected using a theoretically and empirically-validated test. Instruments were validated by using Aiken’s approach of technique content validity and item analysis, and then analyzed using anova stylistic. The result of the analysis shows that the students facilitated with problem-based learning and authentic assessment models get the highest score average compared to the other students, both in the concept understanding and mathematical problem solving. The result of hypothesis test shows that, significantly: 1) there is difference of mathematical problem solving ability between the students facilitated with problem-based learning model and conventional learning model, 2) there is difference of mathematical problem solving ability between the students facilitated with authentic assessment model and conventional assessment model, and 3) there is interaction effect between learning model and assessment model on mathematical problem solving. In order to improve the effectiveness of mathematics learning, collaboration between problem-based learning model and authentic assessment model can be considered as one of learning models in class.

  2. Financial Education Through Mathematics and IT Curricula: Pocket Money Management

    OpenAIRE

    Gortcheva, Iordanka

    2013-01-01

    Report published in the Proceedings of the National Conference on "Education in the Information Society", Plovdiv, May, 2013 Mathematics and IT classes in the Bulgarian school provide various opportunities for developing students’ logical, mathematical, and technological thinking. Being an important part of mathematical literacy, financial literacy can be systematically built in the frame of national mathematics and IT curricula. Following that objective, exemplary word problems ...

  3. The role of problem solving method on the improvement of mathematical learning

    Directory of Open Access Journals (Sweden)

    Saeed Mokhtari-Hassanabad

    2012-10-01

    Full Text Available In history of education, problem solving is one of the important educational goals and teachers or parents have intended that their students have capacity of problem solving. In present research, it is tried that study the problem solving method for mathematical learning. This research is implemented via quasi-experimental method on 49 boy students at high school. The results of Leven test and T-test indicated that problem solving method has more effective on the improvement of mathematical learning than traditional instruction method. Therefore it seems that teachers of mathematics must apply the problem solving method in educational systems till students became self-efficiency in mathematical problem solving.

  4. The Effect of Some Constraints on Mathematics Instructors' Problem ...

    African Journals Online (AJOL)

    This study was designed to examine the effect of perceived constraints on four universities mathematics department instructors' classroom practices of problem solving in teaching mathematics. To this end, the target population of the study includes mathematics instructors in the Amhara Regional state universities. From a ...

  5. Design based Investigation on Construction of Mathematical Modelling Problems: Example of Financial Content

    Directory of Open Access Journals (Sweden)

    Melike TURAL SÖNMEZ

    2017-12-01

    Full Text Available The purpose of this study is to examine the construction of mathematical modelling problems process in the content of financial literacy. It is also aimed to create design proposals for construction of mathematical modelling problems. A design based research method was used in this study. The participants were three seventh grade students, six finance experts and nine mathematics education experts. Data collection tools were transcription of video and tapes group discussions, presentations and worksheets during mathematical modelling activities, and participant experts’ feedback form about mathematical modelling problems. There were three stages in this study. First stage was application of preliminary study. This stage gave information about convenience of problems to grade level, students’ timing for solution of problems, clarity of problems and students’ background about content. In second stage, finance experts commented on convenience of mathematical modelling problems to financial literacy standards. In third stage, mathematics education experts commented on convenience of problems to students’ grade level, mathematical modelling principles and seventh grade mathematics lesson objectives. They also gave suggestion on progress. The frequency value of theme in feedback forms was calculated and experts’ expressions were given as citation. It was given suggestion about stages and application of the design guide

  6. Recent Trends in Japanese Mathematics Textbooks for Elementary Grades: Supporting Teachers to Teach Mathematics through Problem Solving

    Science.gov (United States)

    Takahashi, Akihiko

    2016-01-01

    Problem solving has been a major theme in Japanese mathematics curricula for nearly 50 years. Numerous teacher reference books and lesson plans using problem solving have been published since the 1960s. Government-authorized mathematics textbooks for elementary grades, published by six private companies, have had more and more problem solving over…

  7. Examples and problems in mathematical statistics

    CERN Document Server

    Zacks, Shelemyahu

    2013-01-01

    This book presents examples that illustrate the theory of mathematical statistics and details how to apply the methods for solving problems.  While other books on the topic contain problems and exercises, they do not focus on problem solving. This book fills an important niche in the statistical theory literature by providing a theory/example/problem approach.  Each chapter is divided into four parts: Part I provides the needed theory so readers can become familiar with the concepts, notations, and proven results; Part II presents examples from a variety of fields including engineering, mathem

  8. Analytical derivation: An epistemic game for solving mathematically based physics problems

    Science.gov (United States)

    Bajracharya, Rabindra R.; Thompson, John R.

    2016-06-01

    Problem solving, which often involves multiple steps, is an integral part of physics learning and teaching. Using the perspective of the epistemic game, we documented a specific game that is commonly pursued by students while solving mathematically based physics problems: the analytical derivation game. This game involves deriving an equation through symbolic manipulations and routine mathematical operations, usually without any physical interpretation of the processes. This game often creates cognitive obstacles in students, preventing them from using alternative resources or better approaches during problem solving. We conducted hour-long, semi-structured, individual interviews with fourteen introductory physics students. Students were asked to solve four "pseudophysics" problems containing algebraic and graphical representations. The problems required the application of the fundamental theorem of calculus (FTC), which is one of the most frequently used mathematical concepts in physics problem solving. We show that the analytical derivation game is necessary, but not sufficient, to solve mathematically based physics problems, specifically those involving graphical representations.

  9. Investigating and developing engineering students' mathematical modelling and problem-solving skills

    Science.gov (United States)

    Wedelin, Dag; Adawi, Tom; Jahan, Tabassum; Andersson, Sven

    2015-09-01

    How do engineering students approach mathematical modelling problems and how can they learn to deal with such problems? In the context of a course in mathematical modelling and problem solving, and using a qualitative case study approach, we found that the students had little prior experience of mathematical modelling. They were also inexperienced problem solvers, unaware of the importance of understanding the problem and exploring alternatives, and impeded by inappropriate beliefs, attitudes and expectations. Important impacts of the course belong to the metacognitive domain. The nature of the problems, the supervision and the follow-up lectures were emphasised as contributing to the impacts of the course, where students show major development. We discuss these empirical results in relation to a framework for mathematical thinking and the notion of cognitive apprenticeship. Based on the results, we argue that this kind of teaching should be considered in the education of all engineers.

  10. Pre-service teachers’ challenges in presenting mathematical problems

    Science.gov (United States)

    Desfitri, R.

    2018-01-01

    The purpose of this study was to analyzed how pre-service teachers prepare and assigned tasks or assignments in teaching practice situations. This study was also intended to discuss about kind of tasks or assignments they gave to students. Participants of this study were 15 selected pre-service mathematics teachers from mathematics education department who took part on microteaching class as part of teaching preparation program. Based on data obtained, it was occasionally found that there were hidden errors on questions or tasks assigned by pre-service teachers which might lead their students not to be able to reach a logical or correct answer. Although some answers might seem to be true, they were illogical or unfavourable. It is strongly recommended that pre-service teachers be more careful when posing mathematical problems so that students do not misunderstand the problems or the concepts, since both teachers and students were sometimes unaware of errors in problems being worked on.

  11. Is There a Causal Relation between Mathematical Creativity and Mathematical Problem-Solving Performance?

    Science.gov (United States)

    Tyagi, Tarun Kumar

    2016-01-01

    The relationship between mathematical creativity (MC) and mathematical problem-solving performance (MP) has often been studied but the causal relation between these two constructs has yet to be clearly reported. The main purpose of this study was to define the causal relationship between MC and MP. Data from a representative sample of 480…

  12. Mathematical problems in image processing

    International Nuclear Information System (INIS)

    Chidume, C.E.

    2000-01-01

    This is the second volume of a new series of lecture notes of the Abdus Salam International Centre for Theoretical Physics. This volume contains the lecture notes given by A. Chambolle during the School on Mathematical Problems in Image Processing. The school consisted of two weeks of lecture courses and one week of conference

  13. Language, arithmetic word problems, and deaf students: Linguistic strategies used to solve tasks

    Science.gov (United States)

    Zevenbergen, Robyn; Hyde, Merv; Power, Des

    2001-12-01

    There has been limited examination of the intersection between language and arithmetic in the performance of deaf students, although some previous research has shown that deaf and hearing-impaired1 students are delayed in both their language acquisition and arithmetic performance. This paper examines the performance of deaf and hearing-impaired students in South-East Queensland, Australia, in solving arithmetic word problems. It was found that the subjects' solutions of word problems confirmed trends for hearing students, but that their performance was delayed in comparison. The results confirm other studies where deaf and hearing-impaired students are delayed in their language acquisition and this impacts on their capacity to successfully undertake the resolution of word problems.

  14. A synthesis of mathematical and cognitive performances of students with mathematics learning disabilities.

    Science.gov (United States)

    Shin, Mikyung; Bryant, Diane Pedrotty

    2015-01-01

    The purpose of this study was to synthesize the findings from 23 articles that compared the mathematical and cognitive performances of students with mathematics learning disabilities (LD) to (a) students with LD in mathematics and reading, (b) age- or grade-matched students with no LD, and (c) mathematical-ability-matched younger students with no LD. Overall results revealed that students with mathematics LD exhibited higher word problem-solving abilities and no significant group differences on working memory, long-term memory, and metacognition measures compared to students with LD in mathematics and reading. Findings also revealed students with mathematics LD demonstrated significantly lower performance compared to age- or grade-matched students with no LD on both mathematical and cognitive measures. Comparison between students with mathematics LD and younger students with no LD revealed mixed outcomes on mathematical measures and generally no significant group differences on cognitive measures. © Hammill Institute on Disabilities 2013.

  15. Using realistic mathematics education and the DAPIC problem-solving process to enhance secondary school students' mathematical literacy

    Directory of Open Access Journals (Sweden)

    Sunisa Sumirattana

    2017-09-01

    This study was based on research and development design. The main purposes of this study were to develop an instructional process for enhancing mathematical literacy among students in secondary school and to study the effects of the developed instructional process on mathematical literacy. The instructional process was developed by analyzing and synthesizing realistic mathematics education and the DAPIC problem-solving process. The developed instructional process was verified by experts and was trialed. The designated pre-test/post-test control method was used to study the effectiveness of the developed instructional process on mathematical literacy. The sample consisted of 104 ninth grade students from a secondary school in Bangkok, Thailand. The developed instructional process consisted of five steps, namely (1 posing real life problems, (2 solving problems individually or in a group, (3 presenting and discussing, (4 developing formal mathematics, and (5 applying knowledge. The mathematical literacy of the experimental group was significantly higher after being taught through the instructional process. The same results were obtained when comparing the results of the experimental group with the control group.

  16. Mathematical model in economic environmental problems

    Energy Technology Data Exchange (ETDEWEB)

    Nahorski, Z. [Polish Academy of Sciences, Systems Research Inst. (Poland); Ravn, H.F. [Risoe National Lab. (Denmark)

    1996-12-31

    The report contains a review of basic models and mathematical tools used in economic regulation problems. It starts with presentation of basic models of capital accumulation, resource depletion, pollution accumulation, and population growth, as well as construction of utility functions. Then the one-state variable model is discussed in details. The basic mathematical methods used consist of application of the maximum principle and phase plane analysis of the differential equations obtained as the necessary conditions of optimality. A summary of basic results connected with these methods is given in appendices. (au) 13 ills.; 17 refs.

  17. Analysis of mathematical problem-solving ability based on metacognition on problem-based learning

    Science.gov (United States)

    Mulyono; Hadiyanti, R.

    2018-03-01

    Problem-solving is the primary purpose of the mathematics curriculum. Problem-solving abilities influenced beliefs and metacognition. Metacognition as superordinate capabilities can direct, regulate cognition and motivation and then problem-solving processes. This study aims to (1) test and analyzes the quality of problem-based learning and (2) investigate the problem-solving capabilities based on metacognition. This research uses mixed method study with The subject research are class XI students of Mathematics and Science at High School Kesatrian 2 Semarang which divided into tacit use, aware use, strategic use and reflective use level. The collecting data using scale, interviews, and tests. The data processed with the proportion of test, t-test, and paired samples t-test. The result shows that the students with levels tacit use were able to complete the whole matter given, but do not understand what and why a strategy is used. Students with aware use level were able to solve the problem, be able to build new knowledge through problem-solving to the indicators, understand the problem, determine the strategies used, although not right. Students on the Strategic ladder Use can be applied and adopt a wide variety of appropriate strategies to solve the issues and achieved re-examine indicators of process and outcome. The student with reflective use level is not found in this study. Based on the results suggested that study about the identification of metacognition in problem-solving so that the characteristics of each level of metacognition more clearly in a more significant sampling. Teachers need to know in depth about the student metacognitive activity and its relationship with mathematical problem solving and another problem resolution.

  18. The effect of Missouri mathematics project learning model on students’ mathematical problem solving ability

    Science.gov (United States)

    Handayani, I.; Januar, R. L.; Purwanto, S. E.

    2018-01-01

    This research aims to know the influence of Missouri Mathematics Project Learning Model to Mathematical Problem-solving Ability of Students at Junior High School. This research is a quantitative research and uses experimental research method of Quasi Experimental Design. The research population includes all student of grade VII of Junior High School who are enrolled in the even semester of the academic year 2016/2017. The Sample studied are 76 students from experimental and control groups. The sampling technique being used is cluster sampling method. The instrument is consisted of 7 essay questions whose validity, reliability, difficulty level and discriminating power have been tested. Before analyzing the data by using t-test, the data has fulfilled the requirement for normality and homogeneity. The result of data shows that there is the influence of Missouri mathematics project learning model to mathematical problem-solving ability of students at junior high school with medium effect.

  19. The effect of problem posing and problem solving with realistic mathematics education approach to the conceptual understanding and adaptive reasoning

    Science.gov (United States)

    Mahendra, Rengga; Slamet, Isnandar; Budiyono

    2017-12-01

    One of the difficulties of students in learning mathematics is on the subject of geometry that requires students to understand abstract things. The aim of this research is to determine the effect of learning model Problem Posing and Problem Solving with Realistic Mathematics Education Approach to conceptual understanding and students' adaptive reasoning in learning mathematics. This research uses a kind of quasi experimental research. The population of this research is all seventh grade students of Junior High School 1 Jaten, Indonesia. The sample was taken using stratified cluster random sampling technique. The test of the research hypothesis was analyzed by using t-test. The results of this study indicate that the model of Problem Posing learning with Realistic Mathematics Education Approach can improve students' conceptual understanding significantly in mathematics learning. In addition tu, the results also showed that the model of Problem Solving learning with Realistic Mathematics Education Approach can improve students' adaptive reasoning significantly in learning mathematics. Therefore, the model of Problem Posing and Problem Solving learning with Realistic Mathematics Education Approach is appropriately applied in mathematics learning especially on the subject of geometry so as to improve conceptual understanding and students' adaptive reasoning. Furthermore, the impact can improve student achievement.

  20. Use of the Fox derivatives in the solution of the word problem for groups

    International Nuclear Information System (INIS)

    Majumdar, S.

    1988-09-01

    Applying Fox's free partial derivative, the word problem of a finitely presented group has been reduced to the problem of finding an algorithm for determining the existence of a root of a system of linear equations over the integral group ring. The solubility of the word problem for torsion-free one-relator groups and torsion-free polycyclic-by-finite groups has been deduced. (author). 10 refs

  1. Lectures on mathematical theory of extremum problems

    CERN Document Server

    1972-01-01

    The author of this book, Igor' Vladimirovich Girsanov, was one of the first mathematicians to study general extremum problems and to realize the feasibility and desirability of a unified theory of extremal problems, based on a functional­ analytic approach. He actively advocated this view, and his special course, given at the Faculty of Mechanics and Mathematics of the Moscow State University in 1963 and 1964, was apparently the first systematic exposition of a unified approach to the theory of extremal problems. This approach was based on the ideas of Dubovitskii and Milyutin [1]. The general theory of extremal problems has developed so intensely during the past few years that its basic concepts may now be considered finalized. Nevertheless, as yet the basic results of this new field of mathematics have not been presented in a form accessible to a wide range of readers. (The profound paper of Dubovitskii and Milyutin [2] can hardly be recommended for a first study of the theory, since, in particular, it doe...

  2. Applying Lakatos' Theory to the Theory of Mathematical Problem Solving.

    Science.gov (United States)

    Nunokawa, Kazuhiko

    1996-01-01

    The relation between Lakatos' theory and issues in mathematics education, especially mathematical problem solving, is investigated by examining Lakatos' methodology of a scientific research program. (AIM)

  3. University Students' Problem Posing Abilities and Attitudes towards Mathematics.

    Science.gov (United States)

    Grundmeier, Todd A.

    2002-01-01

    Explores the problem posing abilities and attitudes towards mathematics of students in a university pre-calculus class and a university mathematical proof class. Reports a significant difference in numeric posing versus non-numeric posing ability in both classes. (Author/MM)

  4. Process-based Assignment-Setting Change for Support of Overcoming Bottlenecks in Learning by Problem-Posing in Arithmetic Word Problems

    Science.gov (United States)

    Supianto, A. A.; Hayashi, Y.; Hirashima, T.

    2017-02-01

    Problem-posing is well known as an effective activity to learn problem-solving methods. Monsakun is an interactive problem-posing learning environment to facilitate arithmetic word problems learning for one operation of addition and subtraction. The characteristic of Monsakun is problem-posing as sentence-integration that lets learners make a problem of three sentences. Monsakun provides learners with five or six sentences including dummies, which are designed through careful considerations by an expert teacher as a meaningful distraction to the learners in order to learn the structure of arithmetic word problems. The results of the practical use of Monsakun in elementary schools show that many learners have difficulties in arranging the proper answer at the high level of assignments. The analysis of the problem-posing process of such learners found that their misconception of arithmetic word problems causes impasses in their thinking and mislead them to use dummies. This study proposes a method of changing assignments as a support for overcoming bottlenecks of thinking. In Monsakun, the bottlenecks are often detected as a frequently repeated use of a specific dummy. If such dummy can be detected, it is the key factor to support learners to overcome their difficulty. This paper discusses how to detect the bottlenecks and to realize such support in learning by problem-posing.

  5. The Impact of Metacognitive Strategies and Self-Regulating Processes of Solving Math Word Problems

    Science.gov (United States)

    Vula, Eda; Avdyli, Rrezarta; Berisha, Valbona; Saqipi, Blerim; Elezi, Shpetim

    2017-01-01

    This empirical study investigates the impact of metacognitive strategies and self-regulating processes in learners' achievement on solving math word problems. It specifically analyzes the impact of the linguistic factor and the number of steps and arithmetical operations that learners need to apply during the process of solving math word problems.…

  6. Problem solving as a challenge for mathematics education in The Netherlands

    NARCIS (Netherlands)

    Doorman, M.; Drijvers, P.; Dekker, T.; Heuvel-Panhuizen, T. van; Lange, J. de; Wijers, M.

    2007-01-01

    This paper deals with the challenge to establish problem solving as a living domain in mathematics education in The Netherlands. While serious attempts are made to implement a problem-oriented curriculum based on principles of realistic mathematics education with room for modelling and with

  7. To what extent do student teachers develop their mathematical problem solving ability by self-study?

    OpenAIRE

    Kool, Marjolein; Keijzer, Ronald

    2017-01-01

    A primary teacher needs mathematical problem solving ability. That is why Dutch student teachers have to show this ability in a nationwide mathematics test that contains many non-routine problems. Most student teachers prepare for this test by working on their own solving test-like problems. To what extent does these individual problem solving activities really contribute to their mathematical problem solving ability? Developing mathematical problem solving ability requires reflective mathema...

  8. Error Analysis for Arithmetic Word Problems--A Case Study of Primary Three Students in One Singapore School

    Science.gov (United States)

    Cheng, Lu Pien

    2015-01-01

    In this study, ways in which 9-year old students from one Singapore school solved 1-step and 2-step word problems based on the three semantic structures were examined. The students' work and diagrams provided insights into the range of errors in word problem solving for 1- step and 2-step word problems. In particular, the errors provided some…

  9. The comprehension of mathematic problems in primary school

    Directory of Open Access Journals (Sweden)

    Karel Pérez Ariza

    2015-05-01

    Full Text Available The paper describes the result of the research project “A study of causes of difficulties in learning comprehension from an interdisciplinary perspective in Camagüey. The main objective of that study is to propose a methodology for the comprehension of mathematic problems in primary school. In designing the methodology, the characteristics of this text variety, basic principle of the theory of reading comprehension and problem solving were taking into account. In this research work several theoretical methods were used —analysis-synthesis, historical-logical, inductive-deductive— to elaborate the theoretical framework, while modeling and system approach in the methodology construction. Additionally, empirical methods were used in order to assess the knowledge about comprehension of mathematic problems; among them observation and analysis of the activity results.

  10. APPLYING PROFESSIONALLY ORIENTED PROBLEMS OF MATHEMATICAL MODELING IN TEACHING STUDENTS OF ENGINEERING DEPARTMENTS

    Directory of Open Access Journals (Sweden)

    Natal’ya Yur’evna Gorbunova

    2017-06-01

    Full Text Available We described several aspects of organizing student research work, as well as solving a number of mathematical modeling problems: professionally-oriented, multi-stage, etc. We underlined the importance of their economic content. Samples of using such problems in teaching Mathematics at agricultural university were given. Several questions connected with information material selection and peculiarities of research problems application were described. Purpose. The author aims to show the possibility and necessity of using professionally-oriented problems of mathematical modeling in teaching Mathematics at agricultural university. The subject of analysis is including such problems into educational process. Methodology. The main research method is dialectical method of obtaining knowledge of finding approaches to selection, writing and using mathematical modeling and professionally-oriented problems in educational process; the methodology is study of these methods of obtaining knowledge. Results. As a result of analysis of literature, students opinions, observation of students work, and taking into account personal teaching experience, it is possible to make conclusion about importance of using mathematical modeling problems, as it helps to systemize theoretical knowledge, apply it to practice, raise students study motivation in engineering sphere. Practical implications. Results of the research can be of interest for teachers of Mathematics in preparing Bachelor and Master students of engineering departments of agricultural university both for theoretical research and for modernization of study courses.

  11. Students Use Graphic Organizers to Improve Mathematical Problem-Solving Communications

    Science.gov (United States)

    Zollman, Alan

    2009-01-01

    Improving students' problem-solving abilities is a major, if not the major, goal of middle grades mathematics. To address this goal, the author, who is a university mathematics educator, and nine inner-city middle school teachers developed a math/science action research project. This article describes their unique approach to mathematical problem…

  12. Space Mathematics, A Resource for Teachers Outlining Supplementary Space-Related Problems in Mathematics.

    Science.gov (United States)

    Reynolds, Thomas D.; And Others

    This compilation of 138 problems illustrating applications of high school mathematics to various aspects of space science is intended as a resource from which the teacher may select questions to supplement his regular course. None of the problems require a knowledge of calculus or physics, and solutions are presented along with the problem…

  13. Bibliography on moving boundary problems with key word index

    International Nuclear Information System (INIS)

    Wilson, D.G.; Solomon, A.D.; Trent, J.S.

    1979-10-01

    This bibliography concentrates mainly on time-dependent moving-boundary problems of heat and mass transfer. The bibliography is in two parts, a list of the references ordered by last name of the first author and a key word index to the titles. Few references from before 1965 are included

  14. Students’ mathematical representations on secondary school in solving trigonometric problems

    Science.gov (United States)

    Istadi; Kusmayadi, T. A.; Sujadi, I.

    2017-06-01

    This research aimed to analyse students’ mathematical representations on secondary school in solving trigonometric problems. This research used qualitative method. The participants were 4 students who had high competence of knowledge taken from 20 students of 12th natural-science grade SMAN-1 Kota Besi, Central Kalimantan. Data validation was carried out using time triangulation. Data analysis used Huberman and Miles stages. The results showed that their answers were not only based on the given figure, but also used the definition of trigonometric ratio on verbal representations. On the other hand, they were able to determine the object positions to be observed. However, they failed to determine the position of the angle of depression at the sketches made on visual representations. Failure in determining the position of the angle of depression to cause an error in using the mathematical equation. Finally, they were unsuccessful to use the mathematical equation properly on symbolic representations. From this research, we could recommend the importance of translations between mathematical problems and mathematical representations as well as translations among mathematical representaions (verbal, visual, and symbolic) in learning mathematics in the classroom.

  15. Bibliography on moving boundary problems with key word index

    Energy Technology Data Exchange (ETDEWEB)

    Wilson, D.G.; Solomon, A.D.; Trent, J.S.

    1979-10-01

    This bibliography concentrates mainly on time-dependent moving-boundary problems of heat and mass transfer. The bibliography is in two parts, a list of the references ordered by last name of the first author and a key word index to the titles. Few references from before 1965 are included. (RWR)

  16. Obstacles Related to Structuring for Mathematization Encountered by Students when Solving Physics Problems

    DEFF Research Database (Denmark)

    Niss, Martin

    2017-01-01

    This paper studies the cognitive obstacles related to one aspect of mathematization in physics problem-solving, namely, what might be called structuring for mathematization, where the problem situation is structured in such a way that a translation to a mathematical universe can be done. We report...

  17. Using What Matters to Students in Bilingual Mathematics Problems

    Science.gov (United States)

    Dominguez, Higinio

    2011-01-01

    In this study, the author represented what matters to bilingual students in their everyday lives--namely bilingualism and everyday experiences--in school-based mathematical problems. Solving problems in pairs, students demonstrated different patterns of organizing and coordinating talk across problem contexts and across languages. Because these…

  18. Teachers Implementing Mathematical Problem Posing in the Classroom: Challenges and Strategies

    Science.gov (United States)

    Leung, Shuk-kwan S.

    2013-01-01

    This paper reports a study about how a teacher educator shared knowledge with teachers when they worked together to implement mathematical problem posing (MPP) in the classroom. It includes feasible methods for getting practitioners to use research-based tasks aligned to the curriculum in order to encourage children to pose mathematical problems.…

  19. Investigating a Proposed Problem Solving Theory in the Context of Mathematical Problem Solving: A Multi-Case Study

    Science.gov (United States)

    Mills, Nadia Monrose

    2015-01-01

    The ability to succeed in Science, Technology, Engineering, and Mathematics (STEM) careers is contingent on a student's ability to engage in mathematical problem solving. As a result, there has been increased focus on students' ability to think critically by providing them more with problem solving experiences in the classroom. Much research has…

  20. The Efficacy of Using Diagrams When Solving Probability Word Problems in College

    Science.gov (United States)

    Beitzel, Brian D.; Staley, Richard K.

    2015-01-01

    Previous experiments have shown a deleterious effect of visual representations on college students' ability to solve total- and joint-probability word problems. The present experiments used conditional-probability problems, known to be more difficult than total- and joint-probability problems. The diagram group was instructed in how to use tree…

  1. Protocol Analysis of Group Problem Solving in Mathematics: A Cognitive-Metacognitive Framework for Assessment.

    Science.gov (United States)

    Artzt, Alice F.; Armour-Thomas, Eleanor

    The roles of cognition and metacognition were examined in the mathematical problem-solving behaviors of students as they worked in small groups. As an outcome, a framework that links the literature of cognitive science and mathematical problem solving was developed for protocol analysis of mathematical problem solving. Within this framework, each…

  2. Diversity of problems of international mathematical olympiads (imo)

    OpenAIRE

    Kukuraitis, Nerijus

    2012-01-01

    Šiame darbe yra pateikta 16 Pasaulinių olimpiadų uždavinių ir jų sprendimų. Uždaviniai yra lyginami pagal jų sudėtingumo lygį. Sixteen problems and their solutions from International Mathematical Olympiads are presented in this work. Problems are compared by their difficulty.

  3. Mathematical problems in wave propagation theory

    CERN Document Server

    1970-01-01

    The papers comprising this collection are directly or indirectly related to an important branch of mathematical physics - the mathematical theory of wave propagation and diffraction. The paper by V. M. Babich is concerned with the application of the parabolic-equation method (of Academician V. A. Fok and M. A, Leontovich) to the problem of the asymptotic behavior of eigenfunc­ tions concentrated in a neighborhood of a closed geodesie in a Riemannian space. The techniques used in this paper have been föund useful in solving certain problems in the theory of open resonators. The topic of G. P. Astrakhantsev's paper is similar to that of the paper by V. M. Babich. Here also the parabolic-equation method is used to find the asymptotic solution of the elasticity equations which describes Love waves concentrated in a neighborhood of some surface ray. The paper of T. F. Pankratova is concerned with finding the asymptotic behavior of th~ eigenfunc­ tions of the Laplace operator from the exact solution for the surf...

  4. Enhancing students’ mathematical problem posing skill through writing in performance tasks strategy

    Science.gov (United States)

    Kadir; Adelina, R.; Fatma, M.

    2018-01-01

    Many researchers have studied the Writing in Performance Task (WiPT) strategy in learning, but only a few paid attention on its relation to the problem-posing skill in mathematics. The problem-posing skill in mathematics covers problem reformulation, reconstruction, and imitation. The purpose of the present study was to examine the effect of WiPT strategy on students’ mathematical problem-posing skill. The research was conducted at a Public Junior Secondary School in Tangerang Selatan. It used a quasi-experimental method with randomized control group post-test. The samples were 64 students consists of 32 students of the experiment group and 32 students of the control. A cluster random sampling technique was used for sampling. The research data were obtained by testing. The research shows that the problem-posing skill of students taught by WiPT strategy is higher than students taught by a conventional strategy. The research concludes that the WiPT strategy is more effective in enhancing the students’ mathematical problem-posing skill compared to the conventional strategy.

  5. Influence of Number Size, Problem Structure and Response Mode on Children's Solutions of Multiplication Word Problems.

    Science.gov (United States)

    De Corte, E.; And Others

    One important finding from recent research on multiplication word problems is that children's performances are strongly affected by the nature of the multiplier (whether it is an integer, decimal larger than 1 or a decimal smaller than 1). On the other hand, the size of the multiplicand has little or no effect on problem difficulty. The aim of the…

  6. Is Word-Problem Solving a Form of Text Comprehension?

    Science.gov (United States)

    Fuchs, Lynn S.; Fuchs, Douglas; Compton, Donald L.; Hamlett, Carol L.; Wang, Amber Y.

    2015-01-01

    This study's hypotheses were that (a) word-problem (WP) solving is a form of text comprehension that involves language comprehension processes, working memory, and reasoning, but (b) WP solving differs from other forms of text comprehension by requiring WP-specific language comprehension as well as general language comprehension. At the start of…

  7. Robotic Toys as a Catalyst for Mathematical Problem Solving

    Science.gov (United States)

    Highfield, Kate

    2010-01-01

    Robotic toys present unique opportunities for teachers of young children to integrate mathematics learning with engaging problem-solving tasks. This article describes a series of tasks using Bee-bots and Pro-bots, developed as part a larger project examining young children's use of robotic toys as tools in developing mathematical and metacognitive…

  8. Comparison of mathematical problem solving strategies of primary school pupils

    OpenAIRE

    Wasilewská, Eliška

    2016-01-01

    The aim of this dissertation is to describe the role of educational strategy especially in field of the teaching of mathematics and to compare the mathematical problem solving strategies of primary school pupils which are taught by using different educational strategies. In the theoretical part, the main focus is on divergent educational strategies and their characteristics, next on factors affected teaching/learning process and finally on solving the problems. The empirical part of the disse...

  9. Model Drawing Strategy for Fraction Word Problem Solving of Fourth-Grade Students with Learning Disabilities

    Science.gov (United States)

    Sharp, Emily; Shih Dennis, Minyi

    2017-01-01

    This study used a multiple probe across participants design to examine the effects of a model drawing strategy (MDS) intervention package on fraction comparing and ordering word problem-solving performance of three Grade 4 students. MDS is a form of cognitive strategy instruction for teaching word problem solving that includes explicit instruction…

  10. Mathematics Literacy on Problem Based Learning with Indonesian Realistic Mathematics Education Approach Assisted E-Learning Edmodo

    Science.gov (United States)

    Wardono; Waluya, S. B.; Mariani, Scolastika; Candra D, S.

    2016-02-01

    This study aims to find out that there are differences in mathematical literacy ability in content Change and Relationship class VII Junior High School 19, Semarang by Problem Based Learning (PBL) model with an Indonesian Realistic Mathematics Education (called Pendidikan Matematika Realistik Indonesia or PMRI in Indonesia) approach assisted Elearning Edmodo, PBL with a PMRI approach, and expository; to know whether the group of students with learning PBL models with PMRI approach and assisted E-learning Edmodo can improve mathematics literacy; to know that the quality of learning PBL models with a PMRI approach assisted E-learning Edmodo has a good category; to describe the difficulties of students in working the problems of mathematical literacy ability oriented PISA. This research is a mixed methods study. The population was seventh grade students of Junior High School 19, Semarang Indonesia. Sample selection is done by random sampling so that the selected experimental class 1, class 2 and the control experiment. Data collected by the methods of documentation, tests and interviews. From the results of this study showed average mathematics literacy ability of students in the group PBL models with a PMRI approach assisted E-learning Edmodo better than average mathematics literacy ability of students in the group PBL models with a PMRI approach and better than average mathematics literacy ability of students in the expository models; Mathematics literacy ability in the class using the PBL model with a PMRI approach assisted E-learning Edmodo have increased and the improvement of mathematics literacy ability is higher than the improvement of mathematics literacy ability of class that uses the model of PBL learning with PMRI approach and is higher than the improvement of mathematics literacy ability of class that uses the expository models; The quality of learning using PBL models with a PMRI approach assisted E-learning Edmodo have very good category.

  11. The impact of metacognitive strategies and self-regulating processes of solving math word problems

    OpenAIRE

    Eda Vula; Rrezarta Avdyli; Valbona Berisha; Blerim Saqipi; Shpetim Elezi

    2017-01-01

    This empirical study investigates the impact of metacognitive strategies and self-regulating processes in learners’ achievement on solving math word problems. It specifically analyzes the impact of the linguistic factor and the number of steps and arithmetical operations that learners need to apply during the process of solving math word problems. Two hundred sixty-three learners, of three classes of third graders (N=130) and four classes of fifth ...

  12. Mathematical expeditions chronicles by the explorers

    CERN Document Server

    Laubenbacher, Reinhard

    1999-01-01

    This book contains the stories of five mathematical journeys into new realms, told through the writings of the explorers themselves. Some were guided by mere curiosity and the thrill of adventure, while others had more practical motives. In each case the outcome was a vast expansion of the known mathematical world and the realization that still greater vistas remained to be explored. The authors tell these stories by guiding the reader through the very words of the mathematicians at the heart of these events, and thereby provide insight into the art of approaching mathematical problems. The book can be used in a variety of ways. The five chapters are completely independent, each with varying levels of mathematical sophistication. The book will be enticing to students, to instructors, and to the intellectually curious reader. By working through some of the original sources and supplemental exercises, which discuss and solve - or attempt to solve - a great problem, this book helps the reader discover the roots ...

  13. Students’ Algebraic Reasonsing In Solving Mathematical Problems With Adversity Quotient

    Science.gov (United States)

    Aryani, F.; Amin, S. M.; Sulaiman, R.

    2018-01-01

    Algebraic reasoning is a process in which students generalize mathematical ideas from a set of particular instances and express them in increasingly formal and age-appropriate ways. Using problem solving approach to develop algebraic reasoning of mathematics may enhace the long-term learning trajectory of the majority students. The purpose of this research was to describe the algebraic reasoning of quitter, camper, and climber junior high school students in solving mathematical problems. This research used qualitative descriptive method. Subjects were determined by purposive sampling. The technique of collecting data was done by task-based interviews.The results showed that the algebraic reasoning of three students in the process of pattern seeking by identifying the things that are known and asked in a similar way. But three students found the elements of pattern recognition in different ways or method. So, they are generalize the problem of pattern formation with different ways. The study of algebraic reasoning and problem solving can be a learning paradigm in the improve students’ knowledge and skills in algebra work. The goal is to help students’ improve academic competence, develop algebraic reasoning in problem solving.

  14. Glogs as Non-Routine Problem Solving Tools in Mathematics

    Science.gov (United States)

    Devine, Matthew T.

    2013-01-01

    In mathematical problem solving, American students are falling behind their global peers because of a lack of foundational and reasoning skills. A specific area of difficulty with problem solving is working non-routine, heuristic-based problems. Many students are not provided with effective instruction and often grow frustrated and dislike math.…

  15. Cognitive Backgrounds of Problem Solving: A Comparison of Open-Ended vs. Closed Mathematics Problems

    Science.gov (United States)

    Bahar, Abdulkadir; Maker, C. June

    2015-01-01

    Problem solving has been a core theme in education for several decades. Educators and policy makers agree on the importance of the role of problem solving skills for school and real life success. A primary purpose of this study was to investigate the influence of cognitive abilities on mathematical problem solving performance of elementary…

  16. Growing geometric reasoning in solving problems of analytical geometry through the mathematical communication problems to state Islamic university students

    Science.gov (United States)

    Mujiasih; Waluya, S. B.; Kartono; Mariani

    2018-03-01

    Skills in working on the geometry problems great needs of the competence of Geometric Reasoning. As a teacher candidate, State Islamic University (UIN) students need to have the competence of this Geometric Reasoning. When the geometric reasoning in solving of geometry problems has grown well, it is expected the students are able to write their ideas to be communicative for the reader. The ability of a student's mathematical communication is supposed to be used as a marker of the growth of their Geometric Reasoning. Thus, the search for the growth of geometric reasoning in solving of analytic geometry problems will be characterized by the growth of mathematical communication abilities whose work is complete, correct and sequential, especially in writing. Preceded with qualitative research, this article was the result of a study that explores the problem: Was the search for the growth of geometric reasoning in solving analytic geometry problems could be characterized by the growth of mathematical communication abilities? The main activities in this research were done through a series of activities: (1) Lecturer trains the students to work on analytic geometry problems that were not routine and algorithmic process but many problems that the process requires high reasoning and divergent/open ended. (2) Students were asked to do the problems independently, in detail, complete, order, and correct. (3) Student answers were then corrected each its stage. (4) Then taken 6 students as the subject of this research. (5) Research subjects were interviewed and researchers conducted triangulation. The results of this research, (1) Mathematics Education student of UIN Semarang, had adequate the mathematical communication ability, (2) the ability of this mathematical communication, could be a marker of the geometric reasoning in solving of problems, and (3) the geometric reasoning of UIN students had grown in a category that tends to be good.

  17. Problem-Solving: Scaling the "Brick Wall"

    Science.gov (United States)

    Benson, Dave

    2011-01-01

    Across the primary and secondary phases, pupils are encouraged to use and apply their knowledge, skills, and understanding of mathematics to solve problems in a variety of forms, ranging from single-stage word problems to the challenge of extended rich tasks. Amongst many others, Cockcroft (1982) emphasised the importance and relevance of…

  18. Unfinished Student Answer in PISA Mathematics Contextual Problem

    Science.gov (United States)

    Lutfianto, Moch.; Zulkardi; Hartono, Yusuf

    2013-01-01

    Solving mathematics contextual problems is one way that can be used to enable students to have the skills needed to live in the 21st century. Completion contextual problem requires a series of steps in order to properly answer the questions that are asked. The purpose of this study was to determine the steps performed students in solving…

  19. Context problems in realistic mathematics education: A calculus course as an example

    NARCIS (Netherlands)

    Gravemeijer, K.P.E.; Doorman, L.M.

    1999-01-01

    This article discusses the role of context problems, as they are used in the Dutch approach that is known as realistic mathematics education (RME). In RME, context problems are intended for supporting a reinvention process that enables students to come to grips with formal mathematics. This approach

  20. Using Mathematics and Engineering to Solve Problems in Secondary Level Biology

    Science.gov (United States)

    Cox, Charles; Reynolds, Birdy; Schunn, Christian; Schuchardt, Anita

    2016-01-01

    There are strong classroom ties between mathematics and the sciences of physics and chemistry, but those ties seem weaker between mathematics and biology. Practicing biologists realize both that there are interesting mathematics problems in biology, and that viewing classroom biology in the context of another discipline could support students'…

  1. Problems of Mathematical Finance by Stochastic Control Methods

    Science.gov (United States)

    Stettner, Łukasz

    The purpose of this paper is to present main ideas of mathematics of finance using the stochastic control methods. There is an interplay between stochastic control and mathematics of finance. On the one hand stochastic control is a powerful tool to study financial problems. On the other hand financial applications have stimulated development in several research subareas of stochastic control in the last two decades. We start with pricing of financial derivatives and modeling of asset prices, studying the conditions for the absence of arbitrage. Then we consider pricing of defaultable contingent claims. Investments in bonds lead us to the term structure modeling problems. Special attention is devoted to historical static portfolio analysis called Markowitz theory. We also briefly sketch dynamic portfolio problems using viscosity solutions to Hamilton-Jacobi-Bellman equation, martingale-convex analysis method or stochastic maximum principle together with backward stochastic differential equation. Finally, long time portfolio analysis for both risk neutral and risk sensitive functionals is introduced.

  2. Notes on the students’ solutions of Mathematical Olympiad problems

    OpenAIRE

    Veilande, Ingrida

    2015-01-01

    The quality of mathematics education in schools of Latvia can be evaluated by several criteria: on national level – by the results of centralized examination, by diagnostic tests, by students’ achievements in educational Olympiads, and in international comparison – by analysis of results of students’ assessment programs such as TIMSS and PISA. These statistics identify the major problems in mathematics education.

  3. Empowering Educationally Disadvantaged Mathematics Students through a Strategies-Based Problem Solving Approach

    Science.gov (United States)

    Ramnarain, Umesh

    2014-01-01

    A major impediment to problem solving in mathematics in the great majority of South African schools is that disadvantaged students from seriously impoverished learning environments are lacking in the necessary informal mathematical knowledge to develop their own strategies for solving non-routine problems. A randomized pretest-posttest control…

  4. On the Relationships between (Relatively) Advanced Mathematical Knowledge and (Relatively) Advanced Problem-Solving Behaviours

    Science.gov (United States)

    Koichu, Boris

    2010-01-01

    This article discusses an issue of inserting mathematical knowledge within the problem-solving processes. Relatively advanced mathematical knowledge is defined in terms of "three mathematical worlds"; relatively advanced problem-solving behaviours are defined in terms of taxonomies of "proof schemes" and "heuristic behaviours". The relationships…

  5. Mathematical mechanic using physical reasoning to solve problems

    CERN Document Server

    Levi, Mark

    2009-01-01

    Everybody knows that mathematics is indispensable to physics--imagine where we'd be today if Einstein and Newton didn't have the math to back up their ideas. But how many people realize that physics can be used to produce many astonishing and strikingly elegant solutions in mathematics? Mark Levi shows how in this delightful book, treating readers to a host of entertaining problems and mind-bending puzzlers that will amuse and inspire their inner physicist. Levi turns math and physics upside down, revealing how physics can simplify proofs and lead to quicker solutions and new theorems, and how physical solutions can illustrate why results are true in ways lengthy mathematical calculations never can

  6. Mathematical Abstraction in the Solving of Ill-Structured Problems by Elementary School Students in Korea

    Science.gov (United States)

    Hong, Jee Yun; Kim, Min Kyeong

    2016-01-01

    Ill-structured problems can be regarded as one of the measures that meet recent social needs emphasizing students' abilities to solve real-life problems. This study aimed to analyze the mathematical abstraction process in solving such problems, and to identify the mathematical abstraction level ([I] Recognition of mathematical structure through…

  7. Understanding Problem-Solving Errors by Students with Learning Disabilities in Standards-Based and Traditional Curricula

    Science.gov (United States)

    Bouck, Emily C.; Bouck, Mary K.; Joshi, Gauri S.; Johnson, Linley

    2016-01-01

    Students with learning disabilities struggle with word problems in mathematics classes. Understanding the type of errors students make when working through such mathematical problems can further describe student performance and highlight student difficulties. Through the use of error codes, researchers analyzed the type of errors made by 14 sixth…

  8. Improving of Junior High School Visual Thinking Representation Ability in Mathematical Problem Solving by CTL

    Directory of Open Access Journals (Sweden)

    Edy Surya

    2013-01-01

    Full Text Available The students’  difficulty which was found is in the problem of understanding, drawing diagrams, reading the charts correctly, conceptual formal  mathematical understanding, and  mathematical problem solving. The appropriate problem representation is the basic way in order to understand the problem itself and make a plan to solve it. This research was the experimental classroom design with a pretest-posttest control in order to increase the representation of visual thinking ability on mathematical problem solving approach  with  contextual learning. The research instrument was a test, observation and interviews. Contextual approach increases of mathematical representations ability increases in students with high initial category, medium, and low compared to conventional approaches. Keywords: Visual Thinking Representation, Mathematical  Problem Solving, Contextual Teaching Learning Approach DOI: http://dx.doi.org/10.22342/jme.4.1.568.113-126

  9. Developing a pedagogical problem solving view for mathematics teachers with two reflection programs

    Directory of Open Access Journals (Sweden)

    Bracha KRAMARSKI

    2009-10-01

    Full Text Available The study investigated the effects of two reflection support programs on elementary school mathematics teachers’ pedagogical problem solving view. Sixty-two teachers participated in a professional development program. Thirty teachers were assigned to the self-questioning (S_Q training and thirty two teachers were assigned to the reflection discourse (R_D training. The S_Q program was based on the IMPROVE self-questioning approach which emphasizes systematic discussion along the phases of mathematical or pedagogical problem solving as student and teacher. The R_D program emphasized discussion of standard based teaching and learning principles. Findings indicated that systematic reflection support (S_Q is effective for developing mathematics PCK, and strengthening metacognitive knowledge of mathematics teachers, more than reflection discourse (R_D. No differences were found between the groups in developing beliefs about teaching mathematics in using problem solving view.

  10. Error analysis of mathematical problems on TIMSS: A case of Indonesian secondary students

    Science.gov (United States)

    Priyani, H. A.; Ekawati, R.

    2018-01-01

    Indonesian students’ competence in solving mathematical problems is still considered as weak. It was pointed out by the results of international assessment such as TIMSS. This might be caused by various types of errors made. Hence, this study aimed at identifying students’ errors in solving mathematical problems in TIMSS in the topic of numbers that considered as the fundamental concept in Mathematics. This study applied descriptive qualitative analysis. The subject was three students with most errors in the test indicators who were taken from 34 students of 8th graders. Data was obtained through paper and pencil test and student’s’ interview. The error analysis indicated that in solving Applying level problem, the type of error that students made was operational errors. In addition, for reasoning level problem, there are three types of errors made such as conceptual errors, operational errors and principal errors. Meanwhile, analysis of the causes of students’ errors showed that students did not comprehend the mathematical problems given.

  11. The Place of Problem Solving in Contemporary Mathematics Curriculum Documents

    Science.gov (United States)

    Stacey, Kaye

    2005-01-01

    This paper reviews the presentation of problem solving and process aspects of mathematics in curriculum documents from Australia, UK, USA and Singapore. The place of problem solving in the documents is reviewed and contrasted, and illustrative problems from teachers' support materials are used to demonstrate how problem solving is now more often…

  12. The 'thousand words' problem: Summarizing multi-dimensional data

    International Nuclear Information System (INIS)

    Scott, David M.

    2011-01-01

    Research highlights: → Sophisticated process sensors produce large multi-dimensional data sets. → Plant control systems cannot handle images or large amounts of data. → Various techniques reduce the dimensionality, extracting information from raw data. → Simple 1D and 2D methods can often be extended to 3D and 4D applications. - Abstract: An inherent difficulty in the application of multi-dimensional sensing to process monitoring and control is the extraction and interpretation of useful information. Ultimately the measured data must be collapsed into a relatively small number of values that capture the salient characteristics of the process. Although multiple dimensions are frequently necessary to isolate a particular physical attribute (such as the distribution of a particular chemical species in a reactor), plant control systems are not equipped to use such data directly. The production of a multi-dimensional data set (often displayed as an image) is not the final step of the measurement process, because information must still be extracted from the raw data. In the metaphor of one picture being equal to a thousand words, the problem becomes one of paraphrasing a lengthy description of the image with one or two well-chosen words. Various approaches to solving this problem are discussed using examples from the fields of particle characterization, image processing, and process tomography.

  13. Algebraic Reasoning in Solving Mathematical Problem Based on Learning Style

    Science.gov (United States)

    Indraswari, N. F.; Budayasa, I. K.; Ekawati, R.

    2018-01-01

    This study aimed to describe algebraic reasoning of secondary school’s pupils with different learning styles in solving mathematical problem. This study begins by giving the questionnaire to find out the learning styles and followed by mathematical ability test to get three subjects of 8th-grade whereas the learning styles of each pupil is visual, auditory, kinesthetic and had similar mathematical abilities. Then it continued with given algebraic problems and interviews. The data is validated using triangulation of time. The result showed that in the pattern of seeking indicator, subjects identified the things that were known and asked based on them observations. The visual and kinesthetic learners represented the known information in a chart, whereas the auditory learner in a table. In addition, they found the elements which makes the pattern and made a relationship between two quantities. In the pattern recognition indicator, they created conjectures on the relationship between two quantities and proved it. In the generalization indicator, they were determining the general rule of pattern found on each element of pattern using algebraic symbols and created a mathematical model. Visual and kinesthetic learners determined the general rule of equations which was used to solve problems using algebraic symbols, but auditory learner in a sentence.

  14. Refractive Thinking Profile In Solving Mathematical Problem Reviewed from Students Math Capability

    Science.gov (United States)

    Maslukha, M.; Lukito, A.; Ekawati, R.

    2018-01-01

    Refraction is a mental activity experienced by a person to make a decision through reflective thinking and critical thinking. Differences in mathematical capability have an influence on the difference of student’s refractive thinking processes in solving math problems. This descriptive research aims to generate a picture of refractive thinking of students in solving mathematical problems in terms of students’ math skill. Subjects in this study consisted of three students, namely students with high, medium, and low math skills based on mathematics capability test. Data collection methods used are test-based methods and interviews. After collected data is analyzed through three stages that are, condensing and displaying data, data display, and drawing and verifying conclusion. Results showed refractive thinking profiles of three subjects is different. This difference occurs at the planning and execution stage of the problem. This difference is influenced by mathematical capability and experience of each subject.

  15. Profile of male-field dependent (FD) prospective teacher's reflective thinking in solving contextual mathematical problem

    Science.gov (United States)

    Agustan, S.; Juniati, Dwi; Siswono, Tatag Yuli Eko

    2017-08-01

    Reflective thinking is an important component in the world of education, especially in professional education of teachers. In learning mathematics, reflective thinking is one way to solve mathematical problem because it can improve student's curiosity when student faces a mathematical problem. Reflective thinking is also a future competence that should be taught to students to face the challenges and to respond of demands of the 21st century. There are many factors which give impact toward the student's reflective thinking when student solves mathematical problem. One of them is cognitive style. For this reason, reflective thinking and cognitive style are important things in solving contextual mathematical problem. This research paper describes aspect of reflective thinking in solving contextual mathematical problem involved solution by using some mathematical concept, namely linear program, algebra arithmetic operation, and linear equations of two variables. The participant, in this research paper, is a male-prospective teacher who has Field Dependent. The purpose of this paper is to describe aspect of prospective teachers' reflective thinking in solving contextual mathematical problem. This research paper is a descriptive by using qualitative approach. To analyze the data, the researchers focus in four main categories which describe prospective teacher's activities using reflective thinking, namely; (a) formulation and synthesis of experience, (b) orderliness of experience, (c) evaluating the experience and (d) testing the selected solution based on the experience.

  16. Tense and aspect in word problems about motion: diagram, gesture, and the felt experience of time

    Science.gov (United States)

    de Freitas, Elizabeth; Zolkower, Betina

    2015-09-01

    Word problems about motion contain various conjugated verb forms. As students and teachers grapple with such word problems, they jointly operationalize diagrams, gestures, and language. Drawing on findings from a 3-year research project examining the social semiotics of classroom interaction, we show how teachers and students use gesture and diagram to make sense of complex verb forms in such word problems. We focus on the grammatical category of "aspect" for how it broadens the concept of verb tense. Aspect conveys duration and completion or frequency of an event. The aspect of a verb defines its temporal flow (or lack thereof) and the location of a vantage point for making sense of this durational process.

  17. Mathematical modelling and numerical simulation of oil pollution problems

    CERN Document Server

    2015-01-01

    Written by outstanding experts in the fields of marine engineering, atmospheric physics and chemistry, fluid dynamics and applied mathematics, the contributions in this book cover a wide range of subjects, from pure mathematics to real-world applications in the oil spill engineering business. Offering a truly interdisciplinary approach, the authors present both mathematical models and state-of-the-art numerical methods for adequately solving the partial differential equations involved, as well as highly practical experiments involving actual cases of ocean oil pollution. It is indispensable that different disciplines of mathematics, like analysis and numerics,  together with physics, biology, fluid dynamics, environmental engineering and marine science, join forces to solve today’s oil pollution problems.   The book will be of great interest to researchers and graduate students in the environmental sciences, mathematics and physics, showing the broad range of techniques needed in order to solve these poll...

  18. Creativity of Field-dependent and Field-independent Students in Posing Mathematical Problems

    Science.gov (United States)

    Azlina, N.; Amin, S. M.; Lukito, A.

    2018-01-01

    This study aims at describing the creativity of elementary school students with different cognitive styles in mathematical problem-posing. The posed problems were assessed based on three components of creativity, namely fluency, flexibility, and novelty. The free-type problem posing was used in this study. This study is a descriptive research with qualitative approach. Data collections were conducted through written task and task-based interviews. The subjects were two elementary students. One of them is Field Dependent (FD) and the other is Field Independent (FI) which were measured by GEFT (Group Embedded Figures Test). Further, the data were analyzed based on creativity components. The results show thatFD student’s posed problems have fulfilled the two components of creativity namely fluency, in which the subject posed at least 3 mathematical problems, and flexibility, in whichthe subject posed problems with at least 3 different categories/ideas. Meanwhile,FI student’s posed problems have fulfilled all three components of creativity, namely fluency, in which thesubject posed at least 3 mathematical problems, flexibility, in which thesubject posed problems with at least 3 different categories/ideas, and novelty, in which the subject posed problems that are purely the result of her own ideas and different from problems they have known.

  19. The enhancement of students' mathematical problem solving ability through teaching with metacognitive scaffolding approach

    Science.gov (United States)

    Prabawanto, Sufyani

    2017-05-01

    This research aims to investigate the enhancement of students' mathematical problem solving through teaching with metacognitive scaffolding approach. This research used a quasi-experimental design with pretest-posttest control. The subjects were pre-service elementary school teachers in a state university in Bandung. In this study, there were two groups: experimental and control groups. The experimental group consists of 60 studentswho acquire teaching mathematicsunder metacognitive scaffolding approach, while the control group consists of 58 studentswho acquire teaching mathematicsunder direct approach. Students were classified into three categories based on the mathematical prior ability, namely high, middle, and low. Data collection instruments consist of mathematical problem solving test instruments. By usingmean difference test, two conclusions of the research:(1) there is a significant difference in the enhancement of mathematical problem solving between the students who attended the course under metacognitive scaffolding approach and students who attended the course under direct approach, and(2) thereis no significant interaction effect of teaching approaches and ability level based on the mathematical prior ability toward enhancement of students' mathematical problem solving.

  20. Investigating middle school students’ difficulties in mathematical literacy problems level 1 and 2

    Science.gov (United States)

    Setiawati, S.; Herman, T.; Jupri, A.

    2017-11-01

    The background of this study is the lack of mathematical literacy skills of students. The proficiency of students’ mathematical literacy skills based on the results of the PISA 2015 study shows that Indonesian students at the proficiency level 1. This fact gave rise to this study which aims to investigate middle school students’ difficulties in mathematical literacy problems level 1 and 2. Qualitative research was used in this study. An individual written test on mathematical literacy problems was administered, followed by interviews. The subjects of the study were 61 students grade VII in Bandung and 26 of them were interviewed afterward. Data analysis revealed that students’ error in performing arithmetic most frequently observed. Other observed difficulties concerned understanding about algebra concept, applying arithmetic operation in algebraic expressions, and interpreting symbols to represent the unknown. In solving mathematical literacy problems, students use their prior knowledge, although sometimes not relevant to the questions. Based on the results, we suggest that mathematics learning in contextual learning and which invites students to participate in the processes of understanding the concepts.

  1. Strategies That Help Learning-Disabled Students Solve Verbal Mathematical Problems.

    Science.gov (United States)

    Giordano, Gerard

    1990-01-01

    Strategies are presented for dealing with factors that can be responsible for failure in mathematical problem solving. The suggestions include personalization of verbal problems, thematic strands based on student interests, visual representation, a laboratory approach, and paraphrasing. (JDD)

  2. Science Modelling in Pre-Calculus: How to Make Mathematics Problems Contextually Meaningful

    Science.gov (United States)

    Sokolowski, Andrzej; Yalvac, Bugrahan; Loving, Cathleen

    2011-01-01

    "Use of mathematical representations to model and interpret physical phenomena and solve problems is one of the major teaching objectives in high school math curriculum" [National Council of Teachers of Mathematics (NCTM), "Principles and Standards for School Mathematics", NCTM, Reston, VA, 2000]. Commonly used pre-calculus textbooks provide a…

  3. The impact of metacognitive strategies and self-regulating processes of solving math word problems

    Directory of Open Access Journals (Sweden)

    Eda Vula

    2017-09-01

    Full Text Available This empirical study investigates the impact of metacognitive strategies and self-regulating processes in learners’ achievement on solving math word problems. It specifically analyzes the impact of the linguistic factor and the number of steps and arithmetical operations that learners need to apply during the process of solving math word problems. Two hundred sixty-three learners, of three classes of third graders (N=130 and four classes of fifth graders (N=133 of the elementary cycle from two urban schools of Kosovo, participated in the study. Almost half of the total number of the third and fifth-graderswere exposed to metacognitive instruction. The rest of the learners were included in control classes in which they performed tasks without having been given any specific guidance, based exclusively on traditional methods and respective textbooks. All the learners were tested in math word problems twice, before the intervention and after it. Research findings have shown that metacognitive strategies and self-regulating processes that learners use to control their actions, to reason, and to reflect, are one of the main resources that influence their success in solving a math word problem. Although the difference between the pre-test and the post-test resultswas statistically significant solely with the fifth-grade experimental classes, yet an improved performance was observed in third-grade experimental learners’ classes compared to control classes. Theoretical and practical implications of the research are discussed in the end of the study.

  4. Mathematical grammar of biology

    CERN Document Server

    Yamagishi, Michel Eduardo Beleza

    2017-01-01

    This seminal, multidisciplinary book shows how mathematics can be used to study the first principles of DNA. Most importantly, it enriches the so-called “Chargaff’s grammar of biology” by providing the conceptual theoretical framework necessary to generalize Chargaff’s rules. Starting with a simple example of DNA mathematical modeling where human nucleotide frequencies are associated to the Fibonacci sequence and the Golden Ratio through an optimization problem, its breakthrough is showing that the reverse, complement and reverse-complement operators defined over oligonucleotides induce a natural set partition of DNA words of fixed-size. These equivalence classes, when organized into a matrix form, reveal hidden patterns within the DNA sequence of every living organism. Intended for undergraduate and graduate students both in mathematics and in life sciences, it is also a valuable resource for researchers interested in studying invariant genomic properties.

  5. Development of syntax of intuition-based learning model in solving mathematics problems

    Science.gov (United States)

    Yeni Heryaningsih, Nok; Khusna, Hikmatul

    2018-01-01

    The aim of the research was to produce syntax of Intuition Based Learning (IBL) model in solving mathematics problem for improving mathematics students’ achievement that valid, practical and effective. The subject of the research were 2 classes in grade XI students of SMAN 2 Sragen, Central Java. The type of the research was a Research and Development (R&D). Development process adopted Plomp and Borg & Gall development model, they were preliminary investigation step, design step, realization step, evaluation and revision step. Development steps were as follow: (1) Collected the information and studied of theories in Preliminary Investigation step, studied about intuition, learning model development, students condition, and topic analysis, (2) Designed syntax that could bring up intuition in solving mathematics problem and then designed research instruments. They were several phases that could bring up intuition, Preparation phase, Incubation phase, Illumination phase and Verification phase, (3) Realized syntax of Intuition Based Learning model that has been designed to be the first draft, (4) Did validation of the first draft to the validator, (5) Tested the syntax of Intuition Based Learning model in the classrooms to know the effectiveness of the syntax, (6) Conducted Focus Group Discussion (FGD) to evaluate the result of syntax model testing in the classrooms, and then did the revision on syntax IBL model. The results of the research were produced syntax of IBL model in solving mathematics problems that valid, practical and effective. The syntax of IBL model in the classroom were, (1) Opening with apperception, motivations and build students’ positive perceptions, (2) Teacher explains the material generally, (3) Group discussion about the material, (4) Teacher gives students mathematics problems, (5) Doing exercises individually to solve mathematics problems with steps that could bring up students’ intuition: Preparations, Incubation, Illumination, and

  6. Matematica 1. Livro do Aluno (Mathematics 1. Student Workbook).

    Science.gov (United States)

    D'Alu, Maria Jose

    Matematica 1 is the first book of a mathematics program in Portuguese "designed for first graders." The book contains 15 chapters dealing with: sets, numeration, place value, numbers from 0 through 99, addition and subtraction, geometric shapes, measurements (money, time, length), fractions, word problems, and commutative and associative…

  7. Don’t words come easy?A psychophysical exploration of word superiority

    Directory of Open Access Journals (Sweden)

    Randi eStarrfelt

    2013-09-01

    Full Text Available Words are made of letters, and yet sometimes it is easier to identify a word than a single letter. This word superiority effect (WSE has been observed when written stimuli are presented very briefly or degraded by visual noise. We compare performance with letters and words in three experiments, to explore the extents and limits of the WSE. Using a carefully controlled list of three letter words, we show that a word superiority effect can be revealed in vocal reaction times even to undegraded stimuli. With a novel combination of psychophysics and mathematical modelling, we further show that the typical WSE is specifically reflected in perceptual processing speed: single words are simply processed faster than single letters. Intriguingly, when multiple stimuli are presented simultaneously, letters are perceived more easily than words, and this is reflected both in perceptual processing speed and visual short term memory capacity. So, even if single words come easy, there is a limit to the word superiority effect.

  8. The Influence of Cognitive Abilities on Mathematical Problem Solving Performance

    Science.gov (United States)

    Bahar, Abdulkadir

    2013-01-01

    Problem solving has been a core theme in education for several decades. Educators and policy makers agree on the importance of the role of problem solving skills for school and real life success. A primary purpose of this study was to investigate the influence of cognitive abilities on mathematical problem solving performance of students. The…

  9. The Effect of External Representations on Compare Word Problems: Supporting Mental Model Construction

    Science.gov (United States)

    Munez, David; Orrantia, Josetxu; Rosales, Javier

    2013-01-01

    This study explored the effectiveness of external representations presented together with compare word problems, and whether such effectiveness was moderated by working memory. Participants were 49 secondary school students. Each participant solved 48 problems presented in 4 presentation types that included 2 difficulty treatments (number of steps…

  10. Enhancing Arithmetic and Word-Problem Solving Skills Efficiently by Individualized Computer-Assisted Practice

    Science.gov (United States)

    Schoppek, Wolfgang; Tulis, Maria

    2010-01-01

    The fluency of basic arithmetical operations is a precondition for mathematical problem solving. However, the training of skills plays a minor role in contemporary mathematics instruction. The authors proposed individualization of practice as a means to improve its efficiency, so that the time spent with the training of skills is minimized. As a…

  11. Improving of Junior High School Visual Thinking Representation Ability in Mathematical Problem Solving by CTL

    Science.gov (United States)

    Surya, Edy; Sabandar, Jozua; Kusumah, Yaya S.; Darhim

    2013-01-01

    The students' difficulty which was found is in the problem of understanding, drawing diagrams, reading the charts correctly, conceptual formal mathematical understanding, and mathematical problem solving. The appropriate problem representation is the basic way in order to understand the problem itself and make a plan to solve it. This research was…

  12. Text Comprehension and Oral Language as Predictors of Word-Problem Solving: Insights into Word-Problem Solving as a Form of Text Comprehension.

    Science.gov (United States)

    Fuchs, Lynn S; Gilbert, Jennifer K; Fuchs, Douglas; Seethaler, Pamela M; Martin, BrittanyLee N

    2018-01-01

    This study was designed to deepen insights on whether word-problem (WP) solving is a form of text comprehension (TC) and on the role of language in WPs. A sample of 325 second graders, representing high, average, and low reading and math performance, was assessed on (a) start-of-year TC, WP skill, language, nonlinguistic reasoning, working memory, and foundational skill (word identification, arithmetic) and (b) year-end WP solving, WP-language processing (understanding WP statements, without calculation demands), and calculations. Multivariate, multilevel path analysis, accounting for classroom and school effects, indicated that TC was a significant and comparably strong predictor of all outcomes. Start-of-year language was a significantly stronger predictor of both year-end WP outcomes than of calculations, whereas start-of-year arithmetic was a significantly stronger predictor of calculations than of either WP measure. Implications are discussed in terms of WP solving as a form of TC and a theoretically coordinated approach, focused on language, for addressing TC and WP-solving instruction.

  13. Mathematics Interventions for Students with High Functioning Autism/Asperger's Syndrome

    Science.gov (United States)

    Donaldson, Jeffrey B.; Zager, Dianne

    2010-01-01

    Teachers are often at a loss when considering how to address mathematics difficulties for students with high functioning autism/Asperger's syndrome (HFA/AS). Students may show difficulty remembering operations throughout an equation, organizing information on the page, and comprehending the language in instructions of word problems. These…

  14. DESIGN OF EDUCATIONAL PROBLEMS ON LINEAR PROGRAMMING USING SYSTEMS OF COMPUTER MATHEMATICS

    Directory of Open Access Journals (Sweden)

    Volodymyr M. Mykhalevych

    2013-11-01

    Full Text Available From a perspective of the theory of educational problems a problem of substitution in the conditions of ICT use of one discipline by an educational problem of another discipline is represented. Through the example of mathematical problems of linear programming it is showed that a student’s method of operation in the course of an educational problem solving is determinant in the identification of an educational problem in relation to a specific discipline: linear programming, informatics, mathematical modeling, methods of optimization, automatic control theory, calculus etc. It is substantiated the necessity of linear programming educational problems renovation with the purpose of making students free of bulky similar arithmetic calculations and notes which often becomes a barrier to a deeper understanding of key ideas taken as a basis of algorithms used by them.

  15. Open mathematical problems regarding non-Newtonian fluids

    International Nuclear Information System (INIS)

    Wilson, Helen J

    2012-01-01

    We present three open problems in the mathematical modelling of the flow of non-Newtonian fluids. The first problem is rather long standing: a discontinuity in the dependence of the rise velocity of a gas bubble on its volume. This is very well characterized experimentally but not, so far, fully reproduced either numerically or analytically. The other two are both instabilities. The first is observed experimentally but never predicted analytically or numerically. In the second instability, numerical studies reproduce the experimental observations but there is as yet no analytical or semi-analytical prediction of the linear instability which must be present. (invited article)

  16. Screening for word reading and spelling problems in elementary school: An item response theory perspective

    NARCIS (Netherlands)

    Keuning, J.; Verhoeven, L.T.W.

    2008-01-01

    The purpose of the present study was to explore whether the Item Response Theory (IRT) provides a suitable framework to screen for word reading and spelling problems during the elementary school period. The following issues were addressed from an IRT perspective: (a) the dimensionality of word

  17. Scientific word, Version 1.0

    Directory of Open Access Journals (Sweden)

    Semen Köksal

    1993-01-01

    Full Text Available Scientific Word is the first fully integrated mathematical word processor in the Windows 3.1 environment, which uses the TEX typesetting language for output. It runs as a Microsoft Windows application program and has two-way interface to TEX. The Scientific Word is an object-oriented WYSIWYG word processor for virtually all users who need typesetting scientific books, manuals and papers. It includes automatic equation numbering, spell checking, and LATEX and DVI previewer.

  18. Mathematical models and heuristic solutions for container positioning problems in port terminals

    DEFF Research Database (Denmark)

    Kallehauge, Louise Sibbesen

    2008-01-01

    presents an efficient solution algorithm for the CPP. Based on a number of new concepts, an event-based construction heuristic is developed and its ability to solve real-life problem instances is established. The backbone of the algorithm is a list of events, corresponding to a sequence of operations...... by constructing mathematical programming formulations of the problem and developing an efficient heuristic algorithm for its solution. The thesis consists of an introduction, two main chapters concerning new mathematical formulations and a new heuristic for the CPP, technical issues, computational results...... concerning the subject is reviewed. The research presented in this thesis is divided into two main parts: Construction and investigation of new mathematical programming formulations of the CPP and development and implementation of a new event-based heuristic for the problem. The first part presents three...

  19. Great Problems of Mathematics: A Course Based on Original Sources.

    Science.gov (United States)

    Laubenbacher, Reinhard C.; Pengelley, David J.

    1992-01-01

    Describes the history of five selected problems from mathematics that are included in an undergraduate honors course designed to utilize original sources for demonstrating the evolution of ideas developed in solving these problems: area and the definite integral, the beginnings of set theory, solutions of algebraic equations, Fermat's last…

  20. To what extent do student teachers develop their mathematical problem solving ability by self-study?

    NARCIS (Netherlands)

    Marjolein Kool; Ronald Keijzer

    2017-01-01

    A primary teacher needs mathematical problem solving ability. That is why Dutch student teachers have to show this ability in a nationwide mathematics test that contains many non-routine problems. Most student teachers prepare for this test by working on their own solving test-like problems. To what

  1. An empirical approach to the mathematical values of problem choice and argumentation

    DEFF Research Database (Denmark)

    Johansen, Mikkel Willum; Misfeldt, Morten

    2016-01-01

    In this paper we describe and discuss how mathematical values influence researchers’ choices when practicing mathematics. Our paper is based on a qualitative investigation of mathematicians’ practices, and its goal is to gain an empirically grounded understanding of mathematical values. More...... specifically, we will analyze the values connected to mathematicians’ choice of problems and their choice of argumentative style when communicating their results. We suggest that these two situations can be understood as relating to the three mathematical values: recognizability, formalizability...

  2. Promoting students’ mathematical problem-solving skills through 7e learning cycle and hypnoteaching model

    Science.gov (United States)

    Saleh, H.; Suryadi, D.; Dahlan, J. A.

    2018-01-01

    The aim of this research was to find out whether 7E learning cycle under hypnoteaching model can enhance students’ mathematical problem-solving skill. This research was quasi-experimental study. The design of this study was pretest-posttest control group design. There were two groups of sample used in the study. The experimental group was given 7E learning cycle under hypnoteaching model, while the control group was given conventional model. The population of this study was the student of mathematics education program at one university in Tangerang. The statistical analysis used to test the hypothesis of this study were t-test and Mann-Whitney U. The result of this study show that: (1) The students’ achievement of mathematical problem solving skill who obtained 7E learning cycle under hypnoteaching model are higher than the students who obtained conventional model; (2) There are differences in the students’ enhancement of mathematical problem-solving skill based on students’ prior mathematical knowledge (PMK) category (high, middle, and low).

  3. Is Word-Problem Solving a Form of Text Comprehension?

    Science.gov (United States)

    Fuchs, Lynn S.; Fuchs, Douglas; Compton, Donald L.; Hamlett, Carol L.; Wang, Amber Y.

    2015-01-01

    This study’s hypotheses were that (a) word-problem (WP) solving is a form of text comprehension that involves language comprehension processes, working memory, and reasoning, but (b) WP solving differs from other forms of text comprehension by requiring WP-specific language comprehension as well as general language comprehension. At the start of the 2nd grade, children (n = 206; on average, 7 years, 6 months) were assessed on general language comprehension, working memory, nonlinguistic reasoning, processing speed (a control variable), and foundational skill (arithmetic for WPs; word reading for text comprehension). In spring, they were assessed on WP-specific language comprehension, WPs, and text comprehension. Path analytic mediation analysis indicated that effects of general language comprehension on text comprehension were entirely direct, whereas effects of general language comprehension on WPs were partially mediated by WP-specific language. By contrast, effects of working memory and reasoning operated in parallel ways for both outcomes. PMID:25866461

  4. The Investigation of Elementary Mathematics Teacher Candidates' Problem Solving Skills According to Various Variables

    Science.gov (United States)

    Kaya, Deniz; Izgiol, Dilek; Kesan, Cenk

    2014-01-01

    The aim was to determine elementary mathematics teacher candidates' problem solving skills and analyze problem solving skills according to various variables. The data were obtained from total 306 different grade teacher candidates receiving education in Department of Elementary Mathematics Education, Buca Faculty of Education, Dokuz Eylul…

  5. A Critical Discourse Analysis of Practical Problems in a Foundation Mathematics Course at a South African University

    Science.gov (United States)

    le Roux, Kate; Adler, Jill

    2016-01-01

    Mathematical problems that make links to the everyday and to disciplines other than mathematics--variously referred to as practical, realistic, real-world or applied problems in the literature--feature in school and undergraduate mathematics reforms aimed at increasing mathematics participation in contexts of inequity and diversity. In this…

  6. Are Middle School Mathematics Teachers Able to Solve Word Problems without Using Variable?

    Science.gov (United States)

    Gökkurt Özdemir, Burçin; Erdem, Emrullah; Örnek, Tugba; Soylu, Yasin

    2018-01-01

    Many people consider problem solving as a complex process in which variables such as "x," "y" are used. Problems may not be solved by only using "variable." Problem solving can be rationalized and made easier using practical strategies. When especially the development of children at younger ages is considered, it is…

  7. Towards the Construction of a Framework to Deal with Routine Problems to Foster Mathematical Inquiry

    Science.gov (United States)

    Santos-Trigo, Manuel; Camacho-Machin, Matias

    2009-01-01

    To what extent does the process of solving textbook problems help students develop a way of thinking that is consistent with mathematical practice? Can routine problems be transformed into problem solving activities that promote students' mathematical reflection? These questions are used to outline and discuss features of an inquiry framework…

  8. Assessing the Relation between Seventh-Grade Students' Engagement and Mathematical Problem Solving Performance

    Science.gov (United States)

    Lein, Amy E.; Jitendra, Asha K.; Starosta, Kristin M.; Dupuis, Danielle N.; Hughes-Reid, Cheyenne L.; Star, Jon R.

    2016-01-01

    In this study, the authors assessed the contribution of engagement (on-task behavior) to the mathematics problem-solving performance of seventh-grade students after accounting for prior mathematics achievement. A subsample of seventh-grade students in four mathematics classrooms (one high-, two average-, and one low-achieving) from a larger…

  9. Working Memory Components and Problem-Solving Accuracy: Are There Multiple Pathways?

    Science.gov (United States)

    Swanson, H. Lee; Fung, Wenson

    2016-01-01

    This study determined the working memory (WM) components (executive, phonological short-term memory [STM], and visual-spatial sketchpad) that best predicted mathematical word problem-solving accuracy in elementary schoolchildren (N = 392). The battery of tests administered to assess mediators between WM and problem-solving included measures of…

  10. Incorporating the Common Core's Problem Solving Standard for Mathematical Practice into an Early Elementary Inclusive Classroom

    Science.gov (United States)

    Fletcher, Nicole

    2014-01-01

    Mathematics curriculum designers and policy decision makers are beginning to recognize the importance of problem solving, even at the earliest stages of mathematics learning. The Common Core includes sense making and perseverance in solving problems in its standards for mathematical practice for students at all grade levels. Incorporating problem…

  11. Reflective thinking in solving an algebra problem: a case study of field independent-prospective teacher

    Science.gov (United States)

    Agustan, S.; Juniati, Dwi; Yuli Eko Siswono, Tatag

    2017-10-01

    Nowadays, reflective thinking is one of the important things which become a concern in learning mathematics, especially in solving a mathematical problem. The purpose of this paper is to describe how the student used reflective thinking when solved an algebra problem. The subject of this research is one female student who has field independent cognitive style. This research is a descriptive exploratory study with data analysis using qualitative approach to describe in depth reflective thinking of prospective teacher in solving an algebra problem. Four main categories are used to analyse the reflective thinking in solving an algebra problem: (1) formulation and synthesis of experience, (2) orderliness of experience, (3) evaluating the experience and (4) testing the selected solution based on the experience. The results showed that the subject described the problem by using another word and the subject also found the difficulties in making mathematical modelling. The subject analysed two concepts used in solving problem. For instance, geometry related to point and line while algebra is related to algebra arithmetic operation. The subject stated that solution must have four aspect to get effective solution, specifically the ability to (a) understand the meaning of every words; (b) make mathematical modelling; (c) calculate mathematically; (d) interpret solution obtained logically. To test the internal consistency or error in solution, the subject checked and looked back related procedures and operations used. Moreover, the subject tried to resolve the problem in a different way to compare the answers which had been obtained before. The findings supported the assertion that reflective thinking provides an opportunity for the students in improving their weakness in mathematical problem solving. It can make a grow accuracy and concentration in solving a mathematical problem. Consequently, the students will get the right and logic answer by reflective thinking.

  12. Text Comprehension and Oral Language as Predictors of Word-Problem Solving: Insights into Word-Problem Solving as a Form of Text Comprehension

    Science.gov (United States)

    Fuchs, Lynn S.; Gilbert, Jennifer K.; Fuchs, Douglas; Seethaler, Pamela M.; Martin, BrittanyLee N.

    2018-01-01

    This study was designed to deepen insights on whether word-problem (WP) solving is a form of text comprehension (TC) and on the role of language in WPs. A sample of 325 second graders, representing high, average, and low reading and math performance, was assessed on (a) start-of-year TC, WP skill, language, nonlinguistic reasoning, working memory, and foundational skill (word identification, arithmetic) and (b) year-end WP solving, WP-language processing (understanding WP statements, without calculation demands), and calculations. Multivariate, multilevel path analysis, accounting for classroom and school effects, indicated that TC was a significant and comparably strong predictor of all outcomes. Start-of-year language was a significantly stronger predictor of both year-end WP outcomes than of calculations, whereas start-of-year arithmetic was a significantly stronger predictor of calculations than of either WP measure. Implications are discussed in terms of WP solving as a form of TC and a theoretically coordinated approach, focused on language, for addressing TC and WP-solving instruction. PMID:29643723

  13. Towards efficient measurement of metacognition in mathematical problem solving

    NARCIS (Netherlands)

    Jacobse, Annemieke E.; Harskamp, Egbert G.

    Metacognitive monitoring and regulation play an essential role in mathematical problem solving. Therefore, it is important for researchers and practitioners to assess students' metacognition. One proven valid, but time consuming, method to assess metacognition is by using think-aloud protocols.

  14. Assessing the Internal Dynamics of Mathematical Problem Solving in Small Groups.

    Science.gov (United States)

    Artzt, Alice F.; Armour-Thomas, Eleanor

    The purpose of this exploratory study was to examine the problem-solving behaviors and perceptions of (n=27) seventh-grade students as they worked on solving a mathematical problem within a small-group setting. An assessment system was developed that allowed for this analysis. To assess problem-solving behaviors within a small group a Group…

  15. Long-Range Memory in Literary Texts: On the Universal Clustering of the Rare Words.

    Science.gov (United States)

    Tanaka-Ishii, Kumiko; Bunde, Armin

    2016-01-01

    A fundamental problem in linguistics is how literary texts can be quantified mathematically. It is well known that the frequency of a (rare) word in a text is roughly inverse proportional to its rank (Zipf's law). Here we address the complementary question, if also the rhythm of the text, characterized by the arrangement of the rare words in the text, can be quantified mathematically in a similar basic way. To this end, we consider representative classic single-authored texts from England/Ireland, France, Germany, China, and Japan. In each text, we classify each word by its rank. We focus on the rare words with ranks above some threshold Q and study the lengths of the (return) intervals between them. We find that for all texts considered, the probability SQ(r) that the length of an interval exceeds r, follows a perfect Weibull-function, SQ(r) = exp(-b(β)rβ), with β around 0.7. The return intervals themselves are arranged in a long-range correlated self-similar fashion, where the autocorrelation function CQ(s) of the intervals follows a power law, CQ(s) ∼ s-γ, with an exponent γ between 0.14 and 0.48. We show that these features lead to a pronounced clustering of the rare words in the text.

  16. USING TASK LIKE PISA’S PROBLEM TO SUPPORT STUDENT’S CREATIVITY IN MATHEMATICS

    Directory of Open Access Journals (Sweden)

    Rita Novita

    2016-01-01

    Full Text Available Creativity is one of keys to success in the evolving global economy and also be a fundamental skill that is absolutely necessary in the 21st century. Also In mathematics, creativity or thinking creatively is important to be developed because creativity is an integral part of mathematics. However, limiting the use of creativity in the classroom reduces mathematics to a set of skills to master and rules to memorize. Doing so causes many children’s natural curiosity and enthusiasm for mathematics to disappear as they get older, creating a tremendous problem for mathematics educators who are trying to instil these very qualities. In order to investigate the increase in awareness of elementary school students’ creativity in solving mathematics’ problems by using task like PISA’s Question, a qualitative research emphasizing on holistic description was conducted. We used a formative evaluation type of development research as a mean to develop mathematical tasks like PISA’s question that have potential effect to support students’ creativity in mathematics. Ten elementary school students of grade 6 in Palembang were involved in this research. They judged the task given for them is very challenging and provokes their curiosity. The result showed that task like PISA’s question can encourage students to more creatively in mathematics.

  17. Foreign Words as a Problem in Standardisation / Lexicography: English and Afrikaans Loan-words in isiXhosa*

    Directory of Open Access Journals (Sweden)

    Anja Drame

    2011-10-01

    Full Text Available

    Abstract: Languages are not static systems. They develop and change, add new items while others become outdated. These changes can be clearly observed in the lexicon especially. No language can afford to ignore or neglect foreign influence. Due to globalisation, especially English gains more and more influence on other (also European languages. In developing countries, the languages of the former colonisers also still have an enormous impact on the indigenous languages. Some of these nations are slowly heading towards endogenous language policies which demands the modernisation of the technical vocabulary. This is however a costly and time-consuming process. In this regard language planners often prefer borrowing from foreign sources as a quick and therefore cheap method. The first part of this paper deals with the discussion amongst linguists, sociolinguists and lexicographers about the extent to which foreign words should be allowed in an indigenous language. The second part looks at the example of isiXhosa, one of South Africa's eleven official languages, which is strongly influenced by foreign words, especially English and Afrikaans, and shows problems and methods of the integration of foreign words into the isiXhosa grammatical structure.

    Keywords: FOREIGN WORDS, ISIXHOSA, ENGLISH, AFRIKAANS, BAHASA INDONESIA, RUSSIAN, ESTONIAN, GERMAN, LANGUAGE POLICY, LANGUAGE PURISM, LSP, MORPHOLOGY, SEMANTICS, PHONOLOGY

    Zusammenfassung: Fremdwörter als Problem in der Standardisierung/Lexikographie: Englische und afrikaanse Lehnwörter in isiXhosa. Sprachen sindkeine statischen Systeme. Sie entwickeln und verändern sich, fügen neue Bestandteile hinzu,während andere veralten. Diese Vorgänge lassen sich besonders deutlich im Lexikon einer Sprachebeobachten. Keine Sprache kann es sich leisten fremde Einflüsse zu ignorieren oder zurückzuweisen.Aufgrund von Globalisation gewinnt vor allem das Englisch immer mehr Einfluss aufandere (auch

  18. Problem representation and mathematical problem solving of students of varying math ability.

    Science.gov (United States)

    Krawec, Jennifer L

    2014-01-01

    The purpose of this study was to examine differences in math problem solving among students with learning disabilities (LD, n = 25), low-achieving students (LA, n = 30), and average-achieving students (AA, n = 29). The primary interest was to analyze the processes students use to translate and integrate problem information while solving problems. Paraphrasing, visual representation, and problem-solving accuracy were measured in eighth grade students using a researcher-modified version of the Mathematical Processing Instrument. Results indicated that both students with LD and LA students struggled with processing but that students with LD were significantly weaker than their LA peers in paraphrasing relevant information. Paraphrasing and visual representation accuracy each accounted for a statistically significant amount of variance in problem-solving accuracy. Finally, the effect of visual representation of relevant information on problem-solving accuracy was dependent on ability; specifically, for students with LD, generating accurate visual representations was more strongly related to problem-solving accuracy than for AA students. Implications for instruction for students with and without LD are discussed.

  19. The Strategies of Mathematics Teachers When Solving Number Sense Problems

    Directory of Open Access Journals (Sweden)

    Sare Şengül

    2014-04-01

    Full Text Available Number sense involves efficient strategies and the ability to think flexibly with numbers and number operations and flexible thinking ability and the inclination getting for making sound mathematical judgements. The aim of this study was to investigate the strategies used by mathematics teachers while solving number sense problems. Eleven mathematics teachers from a graduate program in education were the participants. A number sense test which has a total of 12 problems is used as the data gathering tool. Teachers’ responses and strategies were analyzed both qualitatively and quantitatively.First, participants’ responses were evaluated for correctness. Then the strategies teachers used were analyzed. The strategies were categorized as based on the use of number sense or rule based strategies. When the correct and incorrect responses were considered together, in the 46% of the responses number sense strategies were used and in 54% the rule-based strategies were used. The results of this study showed that even though teachers can use number sense strategies at some level, there is still room for development in teachers’ number sense.

  20. Write Is Right: Using Graphic Organizers to Improve Student Mathematical Problem Solving

    Science.gov (United States)

    Zollman, Alan

    2012-01-01

    Teachers have used graphic organizers successfully in teaching the writing process. This paper describes graphic organizers and their potential mathematics benefits for both students and teachers, elucidates a specific graphic organizer adaptation for mathematical problem solving, and discusses results using the "four-corners-and-a-diamond"…

  1. Intuitive physics knowledge, physics problem solving and the role of mathematical equations

    Directory of Open Access Journals (Sweden)

    Laura Buteler

    2012-09-01

    Full Text Available The present work explores the role that mathematical equations play in modifying students’ physical intuition (diSessa, 1993. The work is carried out assuming that students achieve a great deal of the refinement in their physical intuitions during problem solving (Sherin, 2006. The study is guided by the question of how the use of mathematical equations contributes to this refinement. The authors aim at expanding on Sherin´s (2006 hypothesis, suggesting a more bounding relation between physical intuitions and mathematics. In this scenario, intuitions play a more compelling role in “deciding” which equations are acceptable and which are not. Our hypothesis is constructed on the basis of three cases: the first published by Sherin (2006 and two more from registries of our own. The three cases are compared and analyzed in relation to the role of mathematical equations in refining – or not – the intuitive knowledge students bring to play during problem solving.

  2. Examination Of Gifted Students’ Probability Problem Solving Process In Terms Of Mathematical Thinking

    Directory of Open Access Journals (Sweden)

    Serdal BALTACI

    2016-10-01

    Full Text Available It is a widely known fact that gifted students have different skills compared to their peers. However, to what extent gifted students use mathematical thinking skills during probability problem solving process emerges as a significant question. Thence, the main aim of the present study is to examine 8th grade gifted students’ probability problem-solving process related to daily life in terms of mathematical thinking skills. In this regard, a case study was used in the study. The participants of the study were six students at 8th grade (four girls and two boys from the Science and Art Center. One of the purposeful sampling methods, maximum variation sampling was used for selecting the participants. Clinical interview and problems were used as a data collection tool. As a results of the study, it was determined that gifted students use reasoning and strategies skill, which is one of the mathematical thinking skills, mostly on the process of probability problem solving, and communication skills at least.

  3. Working memory components that predict word problem solving: Is it merely a function of reading, calculation, and fluid intelligence?

    Science.gov (United States)

    Fung, Wenson; Swanson, H Lee

    2017-07-01

    The purpose of this study was to assess whether the differential effects of working memory (WM) components (the central executive, phonological loop, and visual-spatial sketchpad) on math word problem-solving accuracy in children (N = 413, ages 6-10) are completely mediated by reading, calculation, and fluid intelligence. The results indicated that all three WM components predicted word problem solving in the nonmediated model, but only the storage component of WM yielded a significant direct path to word problem-solving accuracy in the fully mediated model. Fluid intelligence was found to moderate the relationship between WM and word problem solving, whereas reading, calculation, and related skills (naming speed, domain-specific knowledge) completely mediated the influence of the executive system on problem-solving accuracy. Our results are consistent with findings suggesting that storage eliminates the predictive contribution of executive WM to various measures Colom, Rebollo, Abad, & Shih (Memory & Cognition, 34: 158-171, 2006). The findings suggest that the storage component of WM, rather than the executive component, has a direct path to higher-order processing in children.

  4. Students’ Self-Monitoring on Mathematics Ability: Cube and Cuboid Problem Solving

    Science.gov (United States)

    Lusiana, N. T.; Lukito, A.; Khabibah, S.

    2018-01-01

    This study aims at describing students’ activity to understand the behaviors processes called self-monitoring in a cube and cuboid problem solving viewed from mathematics ability. The subjects were eight graders of junior high school who studied surface area and volume of cube and cuboid clussified into high, average and low mathematics abilities. Mathematics ability test to select the subjects the study. Data were collected through self-monitoring task and interviews. Data triangulation was used to verify the credibillity findings. Data analysis was done by data condensation, data display and conclusion drawing and verification. Results showed that students’ self-monitoring with high math ability is more fullfilled self-monitoring components. Students with average and low math abilities not fullfilled the component that covers verifying the results during solving the problem. It is expected that teachers must provide different learning treatments to improve students’ self-monitoring for better learning outcomes.

  5. Mathematical Approaches to Problems in Resource Management and Epidemiology

    CERN Document Server

    Levin, Simon; Shoemaker, Christine

    1989-01-01

    Increasingly, mathematical methods are being used to advantage in addressing the problems facing humanity in managing its environment. Problems in resource management and epidemiology especially have demonstrated the utility of quantitative modeling. To explore these approaches, the Center of Applied Mathematics at Cornell University organized a conference in Fall, 1987, with the objective of surveying and assessing the state of the art. This volume records the proceedings of that conference. Underlying virtually all of these studies are models of population growth, from individual cells to large vertebrates. Cell population growth presents the simplest of systems for study, and is of fundamental importance in its own right for a variety of medical and environmental applications. In Part I of this volume, Michael Shuler describes computer models of individual cells and cell populations, and Frank Hoppensteadt discusses the synchronization of bacterial culture growth. Together, these provide a valuable introdu...

  6. INVESTIGATING AND COMMUNICATING TECHNOLOGY MATHEMATICS PROBLEM SOLVING EXPERIENCE OF TWO PRESERVICE TEACHERS

    Directory of Open Access Journals (Sweden)

    Ana Kuzle

    2012-04-01

    Full Text Available In this paper, I report on preservice teachers’ reflections and perceptions on theirproblem-solving process in a technological context. The purpose of the study was to to investigatehow preservice teachers experience working individually in a dynamic geometry environment andhow these experiences affect their own mathematical activity when integrating content (nonroutineproblems and context (technology environment. Careful analysis of participants’ perceptionsregarding their thinking while engaged in problem solving, provided an opportunity to explorehow they explain the emergence of problem solving when working in a dynamic geometryenvironment. The two participants communicated their experience both through the lenses ofthemselves as problem solvers and as future mathematics educators. Moreover, the results of thestudy indicated that problem solving in a technology environment does not necessarily allow focuson decision-making, reflection, and problem solving processes as reported by previous research.

  7. The Interference of Stereotype Threat with Women's Generation of Mathematical Problem-Solving Strategies.

    Science.gov (United States)

    Quinn, Diane M.; Spencer, Steven J.

    2001-01-01

    Investigated whether stereotype threat would depress college women's math performance. In one test, men outperformed women when solving word problems, though women performed equally when problems were converted into numerical equivalents. In another test, participants solved difficult problems in high or reduced stereotype threat conditions. Women…

  8. Mathematical intelligence developed in math learning with classical backsound music of the classical era

    Science.gov (United States)

    Karlimah

    2018-05-01

    This study examines the application of classical music backsound in mathematics learning. The method used is quasi experimental design nonequivalent pretest-posttest control group in elementary school students in Tasikmalaya city, Indonesia. The results showed that classical music contributed significantly to the mathematical intelligence of elementary school students. The mathematical intelligence shown is in the cognitive ability ranging from the level of knowledge to evaluation. High level mathematical intelligence is shown by students in reading and writing integers with words and numbers. The low level of mathematical intelligence exists in projecting the story into a mathematical problem. The implication of this research is the use of classical music backsound on learning mathematics should pay attention to the level of difficulty of mathematics material being studied.

  9. Exploring a Structure for Mathematics Lessons That Foster Problem Solving and Reasoning

    Science.gov (United States)

    Sullivan, Peter; Walker, Nadia; Borcek, Chris; Rennie, Mick

    2015-01-01

    While there is widespread agreement on the importance of incorporating problem solving and reasoning into mathematics classrooms, there is limited specific advice on how this can best happen. This is a report of an aspect of a project that is examining the opportunities and constraints in initiating learning by posing challenging mathematics tasks…

  10. Numerical methods for solution of some nonlinear problems of mathematical physics

    International Nuclear Information System (INIS)

    Zhidkov, E.P.

    1981-01-01

    The continuous analog of the Newton method and its application to some nonlinear problems of mathematical physics using a computer is considered. It is shown that the application of this method in JINR to the wide range of nonlinear problems has shown its universality and high efficiency [ru

  11. Combining fuzzy mathematics with fuzzy logic to solve business management problems

    Science.gov (United States)

    Vrba, Joseph A.

    1993-12-01

    Fuzzy logic technology has been applied to control problems with great success. Because of this, many observers fell that fuzzy logic is applicable only in the control arena. However, business management problems almost never deal with crisp values. Fuzzy systems technology--a combination of fuzzy logic, fuzzy mathematics and a graphical user interface--is a natural fit for developing software to assist in typical business activities such as planning, modeling and estimating. This presentation discusses how fuzzy logic systems can be extended through the application of fuzzy mathematics and the use of a graphical user interface to make the information contained in fuzzy numbers accessible to business managers. As demonstrated through examples from actual deployed systems, this fuzzy systems technology has been employed successfully to provide solutions to the complex real-world problems found in the business environment.

  12. A broad look at the literature on math word problem-solving interventions for third graders

    Directory of Open Access Journals (Sweden)

    Sheri Kingsdorf

    2016-12-01

    Full Text Available Though research on effective instruction in math word problem solving is prominent at the middle and secondary levels, much less work has been done in elementary grades. In this article, we review the research on varied problem-solving instructional interventions at the third-grade level for students across ability levels. Third grade was chosen as the focus due to the fact that word problem-solving requirements are first introduced into the curriculum and standardized assessment at this point in time. Drawing on quantitative studies using single subject, quasi-experimental, and randomized controlled trial designs, we examine the instructional components and instructional content identified as effective across the 13 studies that met search criteria. Conclusions focus on current understanding of best practices, limitations of the existing research, and important considerations for future research.

  13. LEVELING STUDENTS’ CREATIVE THINKING IN SOLVING AND POSING MATHEMATICAL PROBLEM

    Directory of Open Access Journals (Sweden)

    Tatag Yuli Eko Siswono

    2010-07-01

    Full Text Available Many researchers assume that people are creative, but their degree ofcreativity is different. The notion of creative thinking level has beendiscussed .by experts. The perspective of mathematics creative thinkingrefers to a combination of logical and divergent thinking which is basedon intuition but has a conscious aim. The divergent thinking is focusedon flexibility, fluency, and novelty in mathematical problem solving andproblem posing. As students have various backgrounds and differentabilities, they possess different potential in thinking patterns,imagination, fantasy and performance; therefore, students have differentlevels of creative thinking. A research study was conducted in order todevelop a framework for students’ levels of creative thinking inmathematics. This research used a qualitative approach to describe thecharacteristics of the levels of creative thinking. Task-based interviewswere conducted to collect data with ten 8thgrade junior secondary schoolstudents. The results distinguished five levels of creative thinking,namely level 0 to level 4 with different characteristics in each level.These differences are based on fluency, flexibility, and novelty inmathematical problem solving and problem posing.Keywords: student’s creative thinking, problem posing, flexibility,fluency, novelty DOI: http://dx.doi.org/10.22342/jme.1.1.794.17-40

  14. Mathematics Instructional Model Based on Realistic Mathematics Education to Promote Problem Solving Ability at Junior High School Padang

    OpenAIRE

    Edwin Musdi

    2016-01-01

    This research aims to develop a mathematics instructional model based realistic mathematics education (RME) to promote students' problem-solving abilities. The design research used Plomp models, which consists of preliminary phase, development or proto-typing phase and assessment phase.  At this study, only the first two phases conducted. The first phase, a preliminary investigation, carried out with a literature study to examine the theory-based instructional learning RME model, characterist...

  15. Clinical and Cognitive Characteristics Associated with Mathematics Problem Solving in Adolescents with Autism Spectrum Disorder.

    Science.gov (United States)

    Oswald, Tasha M; Beck, Jonathan S; Iosif, Ana-Maria; McCauley, James B; Gilhooly, Leslie J; Matter, John C; Solomon, Marjorie

    2016-04-01

    Mathematics achievement in autism spectrum disorder (ASD) has been understudied. However, the ability to solve applied math problems is associated with academic achievement, everyday problem-solving abilities, and vocational outcomes. The paucity of research on math achievement in ASD may be partly explained by the widely-held belief that most individuals with ASD are mathematically gifted, despite emerging evidence to the contrary. The purpose of the study was twofold: to assess the relative proportions of youth with ASD who demonstrate giftedness versus disability on applied math problems, and to examine which cognitive (i.e., perceptual reasoning, verbal ability, working memory) and clinical (i.e., test anxiety) characteristics best predict achievement on applied math problems in ASD relative to typically developing peers. Twenty-seven high-functioning adolescents with ASD and 27 age- and Full Scale IQ-matched typically developing controls were assessed on standardized measures of math problem solving, perceptual reasoning, verbal ability, and test anxiety. Results indicated that 22% of the ASD sample evidenced a mathematics learning disability, while only 4% exhibited mathematical giftedness. The parsimonious linear regression model revealed that the strongest predictor of math problem solving was perceptual reasoning, followed by verbal ability and test anxiety, then diagnosis of ASD. These results inform our theories of math ability in ASD and highlight possible targets of intervention for students with ASD struggling with mathematics. © 2015 International Society for Autism Research, Wiley Periodicals, Inc.

  16. INTERCULTURAL FEATURES AND THE THEME OF TRAVELLING IN BILINGUAL MATHEMATICS LESSONS

    Directory of Open Access Journals (Sweden)

    Zuzana Naštická

    2016-08-01

    Full Text Available The present qualitative research is focused on bilingual mathematics education. The research presents findings of a case study of one bilingual Slovak and English mathematics 40-minute lesson within an after school elective bilingual mathematics course running weekly since October, 2015. The lesson took place in March, 2016, and was attended by nine learners aged 12-13, eight boys and one girl. The learners are cases of successive school additive bilingual education. The elective course as a whole is a case of immerse bilingual educational programme. In terms of sociolinguistic settings, the course lessons are cases of bilingual education with external second language. The researcher designed and realized the course lessons in terms of CLIL approach, i.e. Content and Language Integrated Learning. The main aim of the case study was to examine if bilingual mathematics instruction does or does not prevent learners from solving math word problems. Secondly, the analysis of transcription of the lesson audio-record served for identification of intercultural features which might hinder the learning process. The analysis of the transcribed audio-record indicates that the bilingual context did not prevent students from solving math word problems, although each of the students worked at their individual rate. On the other hand, some students were confused by the comma as a thousands-separator in multi-digit numbers, and this actually hindered their learning and problem solving process. This fact has been identified as an intercultural difference which had to be explicitly explained to the students. In order to lessen the possible negative influences of bilingual context on mathematics education, teachers need to predict students’ responses to various intercultural differences which students are unfamiliar with.

  17. Use of open-ended problems as the basis for the mathematical creativity growth disclosure of student

    Science.gov (United States)

    Suyitno, A.; Suyitno, H.; Rochmad; Dwijanto

    2018-03-01

    Mathematical creativity is the essence of learning in mathematics. However, mathematical creativity had not yet grown among students. Means there was a gap between needs and reality. This gap must be bridged through by scientific studies, and there were novelty findings, namely the discovery of stages to cultivate of Mathematical Creativity. The problem formulation: How to use of open-ended problems as the basis for the mathematical creativity growth disclosure of student? The goal was to use of open issues as the basis for the mathematical creativity growth disclosure of student. Research method with a qualitative approach. After data was collected then activity in data analysis, include data reduction, data presentation, data interpretation, and conclusion/verification. The results of the research: After the learning by applying the modification of RTTW learning model, then the students were trained to do the open-ended problems and by looking at the UTS and UAS values then qualitatively the results: (1) There was a significant increase of the student's final score. (2) The category of the growth of mathematical creativity of students, the Very Good there were three students, the Good there were six students, There were 17 students, and there were six students. The validation of these results was reinforced by interviews and triangulation. (3) Stage to cultivate mathematical creativity: lecturers should need to provide inputs on student work; Apply an appropriate learning model, and train students to work on the continuing problems.

  18. Structuring Word Problems for Diagnostic Teaching: Helping Teachers Meet the Needs of Children with Mild Disabilities.

    Science.gov (United States)

    Parmar, Rene S.; Cawley, John F.

    1994-01-01

    Matrix organization can be used to construct math word problems for children with mild disabilities. Matrix organization specifies the characteristics of problems, such as problem theme or setting, operations, level of computation complexity, reading vocabulary level, and need for classification. A sample scope and sequence and 16 sample word…

  19. Mathematical Literacy: A new literacy or a new mathematics?

    Directory of Open Access Journals (Sweden)

    Renuka Vithal

    2006-10-01

    Full Text Available Mathematical Literacy is a ‘hot’ topic at present in most countries, whether it is referred to by that name, or in some cases as Numeracy, or Quantitative Literacy, or Matheracy, or as some part of Ethnomathematics, or related to Mathematics in Society. Questions continue to be asked about what is meant by mathematics in any concept of Mathematical Literacy and the use of the very word ‘Literacy’ in its association with Mathematics has been challenged. Its importance, however, lies in changing our perspective on mathematics teaching, away from the elitism so often associated with much mathematics education, and towards a more equitable, accessible and genuinely educational ideal.

  20. Mathematical bridges

    CERN Document Server

    Andreescu, Titu; Tetiva, Marian

    2017-01-01

    Building bridges between classical results and contemporary nonstandard problems, Mathematical Bridges embraces important topics in analysis and algebra from a problem-solving perspective. Blending old and new techniques, tactics and strategies used in solving challenging mathematical problems, readers will discover numerous genuine mathematical gems throughout that will heighten their appreciation of the inherent beauty of mathematics. Most of the problems are original to the authors and are intertwined in a well-motivated exposition driven by representative examples. The book is structured to assist the reader in formulating and proving conjectures, as well as devising solutions to important mathematical problems by making connections between various concepts and ideas from different areas of mathematics. Instructors and educators teaching problem-solving courses or organizing mathematics clubs, as well as motivated mathematics students from high school juniors to college seniors, will find Mathematical Bri...

  1. Long-Range Memory in Literary Texts: On the Universal Clustering of the Rare Words.

    Directory of Open Access Journals (Sweden)

    Kumiko Tanaka-Ishii

    Full Text Available A fundamental problem in linguistics is how literary texts can be quantified mathematically. It is well known that the frequency of a (rare word in a text is roughly inverse proportional to its rank (Zipf's law. Here we address the complementary question, if also the rhythm of the text, characterized by the arrangement of the rare words in the text, can be quantified mathematically in a similar basic way. To this end, we consider representative classic single-authored texts from England/Ireland, France, Germany, China, and Japan. In each text, we classify each word by its rank. We focus on the rare words with ranks above some threshold Q and study the lengths of the (return intervals between them. We find that for all texts considered, the probability SQ(r that the length of an interval exceeds r, follows a perfect Weibull-function, SQ(r = exp(-b(βrβ, with β around 0.7. The return intervals themselves are arranged in a long-range correlated self-similar fashion, where the autocorrelation function CQ(s of the intervals follows a power law, CQ(s ∼ s-γ, with an exponent γ between 0.14 and 0.48. We show that these features lead to a pronounced clustering of the rare words in the text.

  2. Factors involved in making post-performance judgments in mathematics problem-solving.

    Science.gov (United States)

    García Fernández, Trinidad; Kroesbergen, Evelyn; Rodríguez Pérez, Celestino; González-Castro, Paloma; González-Pienda, Julio A

    2015-01-01

    This study examines the impact of executive functions, affective-motivational variables related to mathematics, mathematics achievement and task characteristics on fifth and sixth graders’ calibration accuracy after completing two mathematical problems. A sample of 188 students took part in the study. They were divided into two groups as function of their judgment accuracy after completing the two tasks (accurate= 79, inaccurate= 109). Differences between these groups were examined. The discriminative value of these variables to predict group membership was analyzed, as well as the effect of age, gender, and grade level. The results indicated that accurate students showed better levels of executive functioning, and more positive feelings, beliefs, and motivation related to mathematics. They also spent more time on the tasks. Mathematics achievement, perceived usefulness of mathematics, and time spent on Task 1 significantly predicted group membership, classifying 71.3% of the sample correctly. These results support the relationship between academic achievement and calibration accuracy, suggesting the need to consider a wide range of factors when explaining performance judgments.

  3. Mathematical problems in modeling artificial heart

    Directory of Open Access Journals (Sweden)

    Ahmed N. U.

    1995-01-01

    Full Text Available In this paper we discuss some problems arising in mathematical modeling of artificial hearts. The hydrodynamics of blood flow in an artificial heart chamber is governed by the Navier-Stokes equation, coupled with an equation of hyperbolic type subject to moving boundary conditions. The flow is induced by the motion of a diaphragm (membrane inside the heart chamber attached to a part of the boundary and driven by a compressor (pusher plate. On one side of the diaphragm is the blood and on the other side is the compressor fluid. For a complete mathematical model it is necessary to write the equation of motion of the diaphragm and all the dynamic couplings that exist between its position, velocity and the blood flow in the heart chamber. This gives rise to a system of coupled nonlinear partial differential equations; the Navier-Stokes equation being of parabolic type and the equation for the membrane being of hyperbolic type. The system is completed by introducing all the necessary static and dynamic boundary conditions. The ultimate objective is to control the flow pattern so as to minimize hemolysis (damage to red blood cells by optimal choice of geometry, and by optimal control of the membrane for a given geometry. The other clinical problems, such as compatibility of the material used in the construction of the heart chamber, and the membrane, are not considered in this paper. Also the dynamics of the valve is not considered here, though it is also an important element in the overall design of an artificial heart. We hope to model the valve dynamics in later paper.

  4. The Cognitive Correlates of Third-Grade Skill in Arithmetic, Algorithmic Computation, and Arithmetic Word Problems

    Science.gov (United States)

    Fuchs, Lynn S.; Fuchs, Douglas; Compton, Donald L.; Powell, Sarah R.; Seethaler, Pamela M.; Capizzi, Andrea M.; Schatschneider, Christopher; Fletcher, Jack M.

    2006-01-01

    The purpose of this study was to examine the cognitive correlates of RD-grade skill in arithmetic, algorithmic computation, and arithmetic word problems. Third graders (N = 312) were measured on language, nonverbal problem solving, concept formation, processing speed, long-term memory, working memory, phonological decoding, and sight word…

  5. Mathematics Teaching as Problem Solving: A Framework for Studying Teacher Metacognition Underlying Instructional Practice in Mathematics.

    Science.gov (United States)

    Artzt, Alice F.; Armour-Thomas, Eleanor

    1998-01-01

    Uses a "teaching as problem solving" perspective to examine the components of metacognition underlying the instructional practice of seven experienced and seven beginning secondary-school mathematics teachers. Data analysis of observations, lesson plans, videotapes, and audiotapes of structured interviews suggests that the metacognition of…

  6. A comparison between strategies applied by mathematicians and mathematics teachers to solve a problem

    OpenAIRE

    Guerrero-Ortiz, Carolina; Mena-Lorca, Jaime

    2015-01-01

    International audience; This study analyses the results obtained from comparing the paths shown by expert mathematicians on the one hand and mathematics teachers on the other, when addressing a hypothetical problem that requires the construction of a mathematical model. The research was conducted with a qualitative approach, applying a case study which involved a group of mathematics teachers and three experts from different mathematical areas. The results show that the process of constructin...

  7. Errors of Students Learning With React Strategy in Solving the Problems of Mathematical Representation Ability

    Directory of Open Access Journals (Sweden)

    Delsika Pramata Sari

    2017-06-01

    Full Text Available The purpose of this study was to investigate the errors experienced by students learning with REACT strategy and traditional learning in solving problems of mathematical representation ability. This study used quasi experimental pattern with static-group comparison design. The subjects of this study were 47 eighth grade students of junior high school in Bandung consisting of two samples. The instrument used was a test to measure students' mathematical representation ability. The reliability coefficient about the mathematical representation ability was 0.56. The most prominent errors of mathematical representation ability of students learning with REACT strategy and traditional learning, was on indicator that solving problem involving arithmetic symbols (symbolic representation. In addition, errors were also experienced by many students with traditional learning on the indicator of making the image of a real world situation to clarify the problem and facilitate its completion (visual representation.

  8. Abelian primitive words

    OpenAIRE

    Domaratzki, Michael; Rampersad, Narad

    2011-01-01

    We investigate Abelian primitive words, which are words that are not Abelian powers. We show that unlike classical primitive words, the set of Abelian primitive words is not context-free. We can determine whether a word is Abelian primitive in linear time. Also different from classical primitive words, we find that a word may have more than one Abelian root. We also consider enumeration problems and the relation to the theory of codes. Peer reviewed

  9. The IMO Compendium A Collection of Problems Suggested for the International Mathematical Olympiads 1959-2004

    CERN Document Server

    Djukic, Dusan; Matic, Ivan

    2006-01-01

    The International Mathematical Olympiad (IMO) is a prestigious competition for high-school students interested in mathematics. It offers high school students a chance to measure up with students from the rest of the world. This book contains problems and solutions that appeared on the IMO over the years. It presents a grand total of 1900 problems.

  10. Mathematical Problems in Synthetic Aperture Radar

    Science.gov (United States)

    Klein, Jens

    2010-10-01

    This thesis is concerned with problems related to Synthetic Aperture Radar (SAR). The thesis is structured as follows: The first chapter explains what SAR is, and the physical and mathematical background is illuminated. The following chapter points out a problem with a divergent integral in a common approach and proposes an improvement. Numerical comparisons are shown that indicate that the improvements allow for a superior image quality. Thereafter the problem of limited data is analyzed. In a realistic SAR-measurement the data gathered from the electromagnetic waves reflected from the surface can only be collected from a limited area. However the reconstruction formula requires data from an infinite distance. The chapter gives an analysis of the artifacts which can obscure the reconstructed images due to this problem. Additionally, some numerical examples are shown that point to the severity of the problem. In chapter 4 the fact that data is available only from a limited area is used to propose a new inversion formula. This inversion formula has the potential to make it easier to suppress artifacts due to limited data and, depending on the application, can be refined to a fast reconstruction formula. In the penultimate chapter a solution to the problem of left-right ambiguity is presented. This problem exists since the invention of SAR and is caused by the geometry of the measurements. This leads to the fact that only symmetric images can be obtained. With the solution from this chapter it is possible to reconstruct not only the even part of the reflectivity function, but also the odd part, thus making it possible to reconstruct asymmetric images. Numerical simulations are shown to demonstrate that this solution is not affected by stability problems as other approaches have been. The final chapter develops some continuative ideas that could be pursued in the future.

  11. Students’ Mathematical Literacy in Solving PISA Problems Based on Keirsey Personality Theory

    Science.gov (United States)

    Masriyah; Firmansyah, M. H.

    2018-01-01

    This research is descriptive-qualitative research. The purpose is to describe students’ mathematical literacy in solving PISA on space and shape content based on Keirsey personality theory. The subjects are four junior high school students grade eight with guardian, artisan, rational or idealist personality. Data collecting methods used test and interview. Data of Keirsey Personality test, PISA test, and interview were analysed. Profile of mathematical literacy of each subject are described as follows. In formulating, guardian subject identified mathematical aspects are formula of rectangle area and sides length; significant variables are terms/conditions in problem and formula of ever encountered question; translated into mathematical language those are measurement and arithmetic operations. In employing, he devised and implemented strategies using ease of calculation on area-subtraction principle; declared truth of result but the reason was less correct; didn’t use and switch between different representations. In interpreting, he declared result as area of house floor; declared reasonableness according measurement estimation. In formulating, artisan subject identified mathematical aspects are plane and sides length; significant variables are solution procedure on both of daily problem and ever encountered question; translated into mathematical language those are measurement, variables, and arithmetic operations as well as symbol representation. In employing, he devised and implemented strategies using two design comparison; declared truth of result without reason; used symbol representation only. In interpreting, he expressed result as floor area of house; declared reasonableness according measurement estimation. In formulating, rational subject identified mathematical aspects are scale and sides length; significant variables are solution strategy on ever encountered question; translated into mathematical language those are measurement, variable, arithmetic

  12. Metacognition, Motivation, and Emotions: Contribution of Self-Regulated Learning to Solving Mathematical Problems

    Science.gov (United States)

    Tzohar-Rozen, Meirav; Kramarski, Bracha

    2014-01-01

    Mathematical problem solving is one of the most valuable aspects of mathematics education. It is also the most difficult for elementary-school students (Verschaffel, Greer, & De Corte, 2000). Students experience cognitive and metacognitive difficulties in this area and develop negative emotions and poor motivation, which hamper their efforts…

  13. Critical Thinking and Problem Solving Skills in Mathematics of Grade-7 Public Secondary Students

    Directory of Open Access Journals (Sweden)

    Emil C. Alcantara

    2017-11-01

    Full Text Available The study aimed to assess the academic performance, critical thinking skills, and problem solving skills in mathematics of Grade-7 students in the five central public secondary schools of Area 2, Division of Batangas, Philippines. This study utilized descriptive method of research. Three hundred forty one (341 students of the public secondary schools out of the total of 2,324 Grade-7 students were selected through systematic random sampling as the subjects of the study. It was found out that the level of performance in Mathematics of the Grade-7 students is proficient. The level of critical thinking skills of students from the different schools is above average as well as their level of problem solving skills. The mathematics performance of the students is positively correlated to their level of critical thinking skills and problem solving skills. Students considered the following learning competencies in the different content areas of Grade-7 Mathematics as difficult to master: solving problems involving sets, describing the development of measurement from the primitive to the present international system of units, finding a solution of an equation or inequality involving one variable, using compass and straightedge to bisect line segments and angles, and analyzing, interpreting accurately and drawing conclusions from graphic and tabular presentations of statistical data.

  14. Longitudinal mediators of achievement in mathematics and reading in typical and atypical development.

    Science.gov (United States)

    Barnes, Marcia A; Raghubar, Kimberly P; English, Lianne; Williams, Jeffrey M; Taylor, Heather; Landry, Susan

    2014-03-01

    Longitudinal studies of neurodevelopmental disorders that are diagnosed at or before birth and are associated with specific learning difficulties at school-age provide one method for investigating developmental precursors of later-emerging academic disabilities. Spina bifida myelomeningocele (SBM) is a neurodevelopmental disorder associated with particular problems in mathematics, in contrast to well-developed word reading. Children with SBM (n=30) and typically developing children (n=35) were used to determine whether cognitive abilities measured at 36 and 60 months of age mediated the effect of group on mathematical and reading achievement outcomes at 8.5 and 9.5 years of age. A series of multiple mediator models showed that: visual-spatial working memory at 36 months and phonological awareness at 60 months partially mediated the effect of group on math calculations, phonological awareness partially mediated the effect of group on small addition and subtraction problems on a test of math fluency, and visual-spatial working memory mediated the effect of group on a test of math problem solving. Groups did not differ on word reading, and phonological awareness was the only mediator for reading fluency and reading comprehension. The findings are discussed with reference to theories of mathematical development and disability and with respect to both common and differing cognitive correlates of math and reading. Copyright © 2013 Elsevier Inc. All rights reserved.

  15. The Impact of Problem-Based Learning Approach to Senior High School Students’ Mathematics Critical Thinking Ability

    Directory of Open Access Journals (Sweden)

    Reviandari Widyatiningtyas

    2015-07-01

    Full Text Available The study was report the findings of an only post-test control group research design and aims to analyze the influence of problem-based learning approach, school level, and students’ prior mathematical ability to student’s mathematics critical thinking ability. The research subjects were 140 grade ten senior high school students coming from excellent and moderate school level. The research instruments a set of mathematical critical thinking ability test, and the data were analyzed by using two ways ANOVA and t-test. The research found that the problem based learning approach has significant impact to the ability of students’ mathematics critical thinking in terms of school level and students’ prior mathematical abilities. Furthermore. This research also found that there is no interaction between learning approach and school level, and learning approach and students’ prior mathematics ability to students’ mathematics critical thinking ability.

  16. Flexibility in Mathematics Problem Solving Based on Adversity Quotient

    Science.gov (United States)

    Dina, N. A.; Amin, S. M.; Masriyah

    2018-01-01

    Flexibility is an ability which is needed in problem solving. One of the ways in problem solving is influenced by Adversity Quotient (AQ). AQ is the power of facing difficulties. There are three categories of AQ namely climber, camper, and quitter. This research is a descriptive research using qualitative approach. The aim of this research is to describe flexibility in mathematics problem solving based on Adversity Quotient. The subjects of this research are climber student, camper student, and quitter student. This research was started by giving Adversity Response Profile (ARP) questioner continued by giving problem solving task and interviews. The validity of data measurement was using time triangulation. The results of this research shows that climber student uses two strategies in solving problem and doesn’t have difficulty. The camper student uses two strategies in solving problem but has difficulty to finish the second strategies. The quitter student uses one strategy in solving problem and has difficulty to finish it.

  17. Investigating Pre-service Mathematics Teachers’ Geometric Problem Solving Process in Dynamic Geometry Environment

    Directory of Open Access Journals (Sweden)

    Deniz Özen

    2013-03-01

    Full Text Available The aim of this study is to investigate pre-service elementary mathematics teachers’ open geometric problem solving process in a Dynamic Geometry Environment. With its qualitative inquiry based research design employed, the participants of the study are three pre-service teachers from 4th graders of the Department of Elementary Mathematics Teaching. In this study, clinical interviews, screencaptures of the problem solving process in the Cabri Geomery Environment, and worksheets included 2 open geometry problems have been used to collect the data. It has been investigated that all the participants passed through similar recursive phases as construction, exploration, conjecture, validate, and justification in the problem solving process. It has been thought that this study provide a new point of view to curriculum developers, teachers and researchers

  18. Effectiveness of an Online Social Constructivist Mathematical Problem Solving Course for Malaysian Pre-Service Teachers

    Directory of Open Access Journals (Sweden)

    Kim-Leong Lai

    2009-07-01

    Full Text Available This study assessed the effectiveness of an online mathematical problem solving course designed using a social constructivist approach for pre-service teachers. Thirty-seven pre-service teachers at the Batu Lintang Teacher Institute, Sarawak, Malaysia were randomly selected to participate in the study. The participants were required to complete the course online without the typical face-to-face classes and they were also required to solve authentic mathematical problems in small groups of 4-5 participants based on the Polya’s Problem Solving Model via asynchronous online discussions. Quantitative and qualitative methods such as questionnaires and interviews were used to evaluate the effects of the online learning course. Findings showed that a majority of the participants were satisfied with their learning experiences in the course. There were no significant changes in the participants’ attitudes toward mathematics, while the participants’ skills in problem solving for “understand the problem” and “devise a plan” steps based on the Polya Model were significantly enhanced, though no improvement was apparent for “carry out the plan” and “review”. The results also showed that there were significant improvements in the participants’ critical thinking skills. Furthermore, participants with higher initial computer skills were also found to show higher performance in mathematical problem solving as compared to those with lower computer skills. However, there were no significant differences in the participants’ achievements in the course based on gender. Generally, the online social constructivist mathematical problem solving course is beneficial to the participants and ought to be given the attention it deserves as an alternative to traditional classes. Nonetheless, careful considerations need to be made in the designing and implementing of online courses to minimize problems that participants might encounter while

  19. The effect of shift-problem lessons in the mathematics classsroom

    NARCIS (Netherlands)

    Palha, S.; Dekker, R.; Gravemeijer, K.

    2015-01-01

    It remains difficult to foster problem-solving and mathematical-reasoning capabilities in classrooms where students and teachers are accustomed to the more traditional forms of education. Several studies suggest that this difficulty might be related to the kind of knowledge students acquire in such

  20. The effect of shift-problem lessons in the mathematics classroom

    NARCIS (Netherlands)

    Palha, S.; Dekker, Rijkje; Gravemeijer, K.P.E.

    2015-01-01

    It remains difficult to foster problem-solving and mathematical-reasoning capabilities in classrooms where students and teachers are accustomed to the more traditional forms of education. Several studies suggest that this difficulty might be related to the kind of knowledge students acquire in such

  1. The relationship between mathematical problem-solving skills and self-regulated learning through homework behaviours, motivation, and metacognition

    Science.gov (United States)

    Çiğdem Özcan, Zeynep

    2016-04-01

    Studies highlight that using appropriate strategies during problem solving is important to improve problem-solving skills and draw attention to the fact that using these skills is an important part of students' self-regulated learning ability. Studies on this matter view the self-regulated learning ability as key to improving problem-solving skills. The aim of this study is to investigate the relationship between mathematical problem-solving skills and the three dimensions of self-regulated learning (motivation, metacognition, and behaviour), and whether this relationship is of a predictive nature. The sample of this study consists of 323 students from two public secondary schools in Istanbul. In this study, the mathematics homework behaviour scale was administered to measure students' homework behaviours. For metacognition measurements, the mathematics metacognition skills test for students was administered to measure offline mathematical metacognitive skills, and the metacognitive experience scale was used to measure the online mathematical metacognitive experience. The internal and external motivational scales used in the Programme for International Student Assessment (PISA) test were administered to measure motivation. A hierarchic regression analysis was conducted to determine the relationship between the dependent and independent variables in the study. Based on the findings, a model was formed in which 24% of the total variance in students' mathematical problem-solving skills is explained by the three sub-dimensions of the self-regulated learning model: internal motivation (13%), willingness to do homework (7%), and post-problem retrospective metacognitive experience (4%).

  2. Interpretation of Word Problems and The Effect of Web Based Educational Simulations on Their Solutions

    Directory of Open Access Journals (Sweden)

    Hasan KARAL

    2010-06-01

    Full Text Available The purpose of this study is to make the students involve with the simulation environment with the developed practice and to develop their problem solving abilities by making easy their understanding of word problems. For this goal, a web based simulation environment which could be manipulated related to the defined movement and pool problems was designed in the light of the defined questions in the curriculum. The research was designed according to semi-experimental pattern which has equalized control group. It was applied in two different 8th grade classes on 44 students in total in the city center of Trabzon in 2008-2009 spring semester. In the research it was benefited from both quantitative and qualitative data collection methods, in the study as the data collection instrument to measure students’ cognitive achievements, it was benefited from word problems achievement test which had 20 items and its KR-20 coefficient was 0,86, observations and from the interviews which were made with the students. The study involved 19 students in experiment group and 25 students in the controlled group. It was used web based education in the experiment group, however, in controlled group, traditional education was used. For the analysis of the data collected in the research, t-test as used for the independent groups. At the end of the research, it was seen that in understanding and solving the word problems, the students in the experiment group who used web based education environment which included simulation environment was more successful than the controlled group who used the traditional method. After the interviews it was concluded that the students in the experiment group had positive thoughts about the web based simulations environment. It is defined that students are more motivated to the lesson and they have an increasing self-confident in problem solving in simulation environment

  3. Mathematical writing

    CERN Document Server

    Vivaldi, Franco

    2014-01-01

    This book teaches the art of writing mathematics, an essential -and difficult- skill for any mathematics student.   The book begins with an informal introduction on basic writing principles and a review of the essential dictionary for mathematics. Writing techniques are developed gradually, from the small to the large: words, phrases, sentences, paragraphs, to end with short compositions. These may represent the introduction of a concept, the abstract of a presentation or the proof of a theorem. Along the way the student will learn how to establish a coherent notation, mix words and symbols effectively, write neat formulae, and structure a definition.   Some elements of logic and all common methods of proofs are featured, including various versions of induction and existence proofs. The book concludes with advice on specific aspects of thesis writing (choosing of a title, composing an abstract, compiling a bibliography) illustrated by large number of real-life examples. Many exercises are included; over 150...

  4. Discrete Mathematics

    DEFF Research Database (Denmark)

    Sørensen, John Aasted

    2011-01-01

    The objectives of Discrete Mathematics (IDISM2) are: The introduction of the mathematics needed for analysis, design and verification of discrete systems, including the application within programming languages for computer systems. Having passed the IDISM2 course, the student will be able...... to accomplish the following: -Understand and apply formal representations in discrete mathematics. -Understand and apply formal representations in problems within discrete mathematics. -Understand methods for solving problems in discrete mathematics. -Apply methods for solving problems in discrete mathematics......; construct a finite state machine for a given application. Apply these concepts to new problems. The teaching in Discrete Mathematics is a combination of sessions with lectures and students solving problems, either manually or by using Matlab. Furthermore a selection of projects must be solved and handed...

  5. Mathematical programming methods for large-scale topology optimization problems

    DEFF Research Database (Denmark)

    Rojas Labanda, Susana

    for mechanical problems, but has rapidly extended to many other disciplines, such as fluid dynamics and biomechanical problems. However, the novelty and improvements of optimization methods has been very limited. It is, indeed, necessary to develop of new optimization methods to improve the final designs......, and at the same time, reduce the number of function evaluations. Nonlinear optimization methods, such as sequential quadratic programming and interior point solvers, have almost not been embraced by the topology optimization community. Thus, this work is focused on the introduction of this kind of second...... for the classical minimum compliance problem. Two of the state-of-the-art optimization algorithms are investigated and implemented for this structural topology optimization problem. A Sequential Quadratic Programming (TopSQP) and an interior point method (TopIP) are developed exploiting the specific mathematical...

  6. Problem-solving rubrics revisited: Attending to the blending of informal conceptual and formal mathematical reasoning

    Science.gov (United States)

    Hull, Michael M.; Kuo, Eric; Gupta, Ayush; Elby, Andrew

    2013-06-01

    Much research in engineering and physics education has focused on improving students’ problem-solving skills. This research has led to the development of step-by-step problem-solving strategies and grading rubrics to assess a student’s expertise in solving problems using these strategies. These rubrics value “communication” between the student’s qualitative description of the physical situation and the student’s formal mathematical descriptions (usually equations) at two points: when initially setting up the equations, and when evaluating the final mathematical answer for meaning and plausibility. We argue that (i) neither the rubrics nor the associated problem-solving strategies explicitly value this kind of communication during mathematical manipulations of the chosen equations, and (ii) such communication is an aspect of problem-solving expertise. To make this argument, we present a case study of two students, Alex and Pat, solving the same kinematics problem in clinical interviews. We argue that Pat’s solution, which connects manipulation of equations to their physical interpretation, is more expertlike than Alex’s solution, which uses equations more algorithmically. We then show that the types of problem-solving rubrics currently available do not discriminate between these two types of solutions. We conclude that problem-solving rubrics should be revised or repurposed to more accurately assess problem-solving expertise.

  7. The development of a professional development intervention for mathematical problem-solving pedagogy in a localised context

    Directory of Open Access Journals (Sweden)

    Brantina Chirinda

    2017-06-01

    Full Text Available This article reports on the design and findings of the first iteration of a classroom-based design research project which endeavours to design a professional development intervention for teachers’ mathematical problem-solving pedagogy. The major outcome of this study is the generation of design principles that can be used by other researchers developing a professional development (PD intervention for mathematical problem-solving pedagogy. This study contributes to the mathematical problem-solving pedagogy and PD body of knowledge by working with teachers in an under-researched environment (an informal settlement in Gauteng, South Africa. In this iteration, two experienced Grade 9 mathematics teachers and their learners at a public secondary school in Gauteng, South Africa, participated in a 6-month intervention. Findings from the data are discussed in light of their implications for the next cycle and other PD studies.

  8. The implementation of multiple intelligences based teaching model to improve mathematical problem solving ability for student of junior high school

    Science.gov (United States)

    Fasni, Nurli; Fatimah, Siti; Yulanda, Syerli

    2017-05-01

    This research aims to achieve some purposes such as: to know whether mathematical problem solving ability of students who have learned mathematics using Multiple Intelligences based teaching model is higher than the student who have learned mathematics using cooperative learning; to know the improvement of the mathematical problem solving ability of the student who have learned mathematics using Multiple Intelligences based teaching model., to know the improvement of the mathematical problem solving ability of the student who have learned mathematics using cooperative learning; to know the attitude of the students to Multiple Intelligences based teaching model. The method employed here is quasi-experiment which is controlled by pre-test and post-test. The population of this research is all of VII grade in SMP Negeri 14 Bandung even-term 2013/2014, later on two classes of it were taken for the samples of this research. A class was taught using Multiple Intelligences based teaching model and the other one was taught using cooperative learning. The data of this research were gotten from the test in mathematical problem solving, scale questionnaire of the student attitudes, and observation. The results show the mathematical problem solving of the students who have learned mathematics using Multiple Intelligences based teaching model learning is higher than the student who have learned mathematics using cooperative learning, the mathematical problem solving ability of the student who have learned mathematics using cooperative learning and Multiple Intelligences based teaching model are in intermediate level, and the students showed the positive attitude in learning mathematics using Multiple Intelligences based teaching model. As for the recommendation for next author, Multiple Intelligences based teaching model can be tested on other subject and other ability.

  9. GeoGebra Assist Discovery Learning Model for Problem Solving Ability and Attitude toward Mathematics

    Science.gov (United States)

    Murni, V.; Sariyasa, S.; Ardana, I. M.

    2017-09-01

    This study aims to describe the effet of GeoGebra utilization in the discovery learning model on mathematical problem solving ability and students’ attitude toward mathematics. This research was quasi experimental and post-test only control group design was used in this study. The population in this study was 181 of students. The sampling technique used was cluster random sampling, so the sample in this study was 120 students divided into 4 classes, 2 classes for the experimental class and 2 classes for the control class. Data were analyzed by using one way MANOVA. The results of data analysis showed that the utilization of GeoGebra in discovery learning can lead to solving problems and attitudes towards mathematics are better. This is because the presentation of problems using geogebra can assist students in identifying and solving problems and attracting students’ interest because geogebra provides an immediate response process to students. The results of the research are the utilization of geogebra in the discovery learning can be applied in learning and teaching wider subject matter, beside subject matter in this study.

  10. Does chess instruction improve mathematical problem-solving ability? Two experimental studies with an active control group.

    Science.gov (United States)

    Sala, Giovanni; Gobet, Fernand

    2017-12-01

    It has been proposed that playing chess enables children to improve their ability in mathematics. These claims have been recently evaluated in a meta-analysis (Sala & Gobet, 2016, Educational Research Review, 18, 46-57), which indicated a significant effect in favor of the groups playing chess. However, the meta-analysis also showed that most of the reviewed studies used a poor experimental design (in particular, they lacked an active control group). We ran two experiments that used a three-group design including both an active and a passive control group, with a focus on mathematical ability. In the first experiment (N = 233), a group of third and fourth graders was taught chess for 25 hours and tested on mathematical problem-solving tasks. Participants also filled in a questionnaire assessing their meta-cognitive ability for mathematics problems. The group playing chess was compared to an active control group (playing checkers) and a passive control group. The three groups showed no statistically significant difference in mathematical problem-solving or metacognitive abilities in the posttest. The second experiment (N = 52) broadly used the same design, but the Oriental game of Go replaced checkers in the active control group. While the chess-treated group and the passive control group slightly outperformed the active control group with mathematical problem solving, the differences were not statistically significant. No differences were found with respect to metacognitive ability. These results suggest that the effects (if any) of chess instruction, when rigorously tested, are modest and that such interventions should not replace the traditional curriculum in mathematics.

  11. An Examination of High School Students' Online Engagement in Mathematics Problems

    Science.gov (United States)

    Lim, Woong; Son, Ji-Won; Gregson, Susan; Kim, Jihye

    2018-01-01

    This article examines high school students' engagement in a set of trigonometry problems. Students completed this task independently in an online environment with access to Internet search engines, online textbooks, and YouTube videos. The findings imply that students have the resourcefulness to solve procedure-based mathematics problems in an…

  12. Mathematical enculturation from the students' perspective: shifts in problem-solving beliefs and behaviour during the bachelor programme

    NARCIS (Netherlands)

    Perrenet, J.C.; Taconis, R.

    2009-01-01

    This study investigates the changes in mathematical problem-solving beliefs and behaviour of mathematics students during the years after entering university. Novice bachelor students fill in a questionnaire about their problem-solving beliefs and behaviour. At the end of their bachelor programme, as

  13. Coping Strategies Applied to Comprehend Multistep Arithmetic Word Problems by Students with Above-Average Numeracy Skills and Below-Average Reading Skills

    Science.gov (United States)

    Nortvedt, Guri A.

    2011-01-01

    This article discusses how 13-year-old students with above-average numeracy skills and below-average reading skills cope with comprehending word problems. Compared to other students who are proficient in numeracy and are skilled readers, these students are more disadvantaged when solving single-step and multistep arithmetic word problems. The…

  14. Mathematics Instructional Model Based on Realistic Mathematics Education to Promote Problem Solving Ability at Junior High School Padang

    Directory of Open Access Journals (Sweden)

    Edwin Musdi

    2016-02-01

    Full Text Available This research aims to develop a mathematics instructional model based realistic mathematics education (RME to promote students' problem-solving abilities. The design research used Plomp models, which consists of preliminary phase, development or proto-typing phase and assessment phase.  At this study, only the first two phases conducted. The first phase, a preliminary investigation, carried out with a literature study to examine the theory-based instructional learning RME model, characteristics of learners, learning management descriptions by junior high school mathematics teacher and relevant research. The development phase is done by developing a draft model (an early prototype model that consists of the syntax, the social system, the principle of reaction, support systems, and the impact and effects of instructional support. Early prototype model contain a draft model, lesson plans, worksheets, and assessments. Tesssmer formative evaluation model used to revise the model. In this study only phase of one to one evaluation conducted. In the ppreliminary phase has produced a theory-based learning RME model, a description of the characteristics of learners in grade VIII Junior High School Padang and the description of teacher teaching in the classroom. The result showed that most students were still not be able to solve the non-routine problem. Teachers did not optimally facilitate students to develop problem-solving skills of students. It was recommended that the model can be applied in the classroom.

  15. Mathematics of quantum mechanics. Foundations, examples, problems, solutions; Mathematik der Quantenmechanik. Grundlagen, Beispiele, Aufgaben, Loesungen

    Energy Technology Data Exchange (ETDEWEB)

    Korsch, Hans Juergen

    2013-07-01

    This book mediates the fundamental terms and methods, which are necessary for an understanding of quantum mechanics. It shows, how mathematics can contribute to the understanding of quantum mechanics. The presented quantum-mechanical problems aim at the illustration and exercise of the most important mathematical methods. Because of the clear and understandable presentation and the numerous completely calculated examples and problems this book is suited for the self-study, for the accompanying of courses on quantum physics, for the accomplishment of exercise problems, and for the preparation on examinations.

  16. Enhancing Learners' Problem Solving Performance in Mathematics: A Cognitive Load Perspective

    Science.gov (United States)

    Dhlamini, Joseph J.

    2016-01-01

    This paper reports on a pilot study that investigated the effect of implementing a context-based problem solving instruction (CBPSI) to enhance the problem solving performance of high school mathematics learners. Primarily, the pilot study aimed: (1) to evaluate the efficiency of data collection instruments; and, (2) to test the efficacy of CBPSI…

  17. Review of Street-Fighting Mathematics: The Art of Educated Guessing and Opportunistic Problem Solving by Sanjoy Mahajan

    OpenAIRE

    Thomas J. Pfaff

    2015-01-01

    Mahajan, Sanjoy. Street-Fighting Mathematics: The Art of Educated Guessing and Opportunistic Problem Solving (The MIT Press, Cambridge, Massachusetts, 2010). 152 pp. ISBN 978--0--262--51429--3 Street-Fighting Mathematics is an engaging collection of problem-solving techniques. The book is not for a general audience, as it requires a significant level of mathematical and scientific background knowledge. In particular, most of the book requires knowledge of Calculus I and there are examples ...

  18. Effects of "Handep" Cooperative Learning Based on Indigenous Knowledge on Mathematical Problem Solving Skill

    Science.gov (United States)

    Demitra; Sarjoko

    2018-01-01

    Indigenous people of Dayak tribe in Kalimantan, Indonesia have traditionally relied on a system of mutual cooperation called "handep." The cultural context has an influence on students mathematics learning. The "handep" system might be suitable for modern learning situations to develop mathematical problem-solving skill. The…

  19. Math Teachers' Attitudes towards Photo Math Application in Solving Mathematical Problem Using Mobile Camera

    Science.gov (United States)

    Hamadneh, Iyad M.; Al-Masaeed, Aslan

    2015-01-01

    This study aimed at finding out mathematics teachers' attitudes towards photo math application in solving mathematical problems using mobile camera; it also aim to identify significant differences in their attitudes according to their stage of teaching, educational qualifications, and teaching experience. The study used judgmental/purposive…

  20. Molecular Phylogenetics: Mathematical Framework and Unsolved Problems

    Science.gov (United States)

    Xia, Xuhua

    Phylogenetic relationship is essential in dating evolutionary events, reconstructing ancestral genes, predicting sites that are important to natural selection, and, ultimately, understanding genomic evolution. Three categories of phylogenetic methods are currently used: the distance-based, the maximum parsimony, and the maximum likelihood method. Here, I present the mathematical framework of these methods and their rationales, provide computational details for each of them, illustrate analytically and numerically the potential biases inherent in these methods, and outline computational challenges and unresolved problems. This is followed by a brief discussion of the Bayesian approach that has been recently used in molecular phylogenetics.

  1. The contribution of general cognitive abilities and number abilities to different aspects of mathematics in children.

    Science.gov (United States)

    Träff, Ulf

    2013-10-01

    This study examined the relative contributions of general cognitive abilities and number abilities to word problem solving, calculation, and arithmetic fact retrieval in a sample of 134 children aged 10 to 13 years. The following tasks were administered: listening span, visual matrix span, verbal fluency, color naming, Raven's Progressive Matrices, enumeration, number line estimation, and digit comparison. Hierarchical multiple regressions demonstrated that number abilities provided an independent contribution to fact retrieval and word problem solving. General cognitive abilities contributed to problem solving and calculation. All three number tasks accounted for a similar amount of variance in fact retrieval, whereas only the number line estimation task contributed unique variance in word problem solving. Verbal fluency and Raven's matrices accounted for an equal amount of variance in problem solving and calculation. The current findings demonstrate, in accordance with Fuchs and colleagues' developmental model of mathematical learning (Developmental Psychology, 2010, Vol. 46, pp. 1731-1746), that both number abilities and general cognitive abilities underlie 10- to 13-year-olds' proficiency in problem solving, whereas only number abilities underlie arithmetic fact retrieval. Thus, the amount and type of cognitive contribution to arithmetic proficiency varies between the different aspects of arithmetic. Furthermore, how closely linked a specific aspect of arithmetic is to the whole number representation systems is not the only factor determining the amount and type of cognitive contribution in 10- to 13-year-olds. In addition, the mathematical complexity of the task appears to influence the amount and type of cognitive support. Copyright © 2013 Elsevier Inc. All rights reserved.

  2. How to make university students solve physics problems requiring mathematical skills: The "Adventurous Problem Solving" approach

    NARCIS (Netherlands)

    de Mul, F.F.M.; Martin Batlle, C.; Martin i Batlle, Cristina; de Bruijn, Imme; Rinzema, K.; Rinzema, Kees

    2003-01-01

    Teaching physics to first-year university students (in the USA: junior/senior level) is often hampered by their lack of skills in the underlying mathematics, and that in turn may block their understanding of the physics and their ability to solve problems. Examples are vector algebra, differential

  3. Problem Solving Abilities and Perceptions in Alternative Certification Mathematics Teachers

    Science.gov (United States)

    Evans, Brian R.

    2012-01-01

    It is important for teacher educators to understand new alternative certification middle and high school teachers' mathematical problem solving abilities and perceptions. Teachers in an alternative certification program in New York were enrolled in a proof-based algebra course. At the beginning and end of a semester participants were given a…

  4. Classification of SBS Mathematics Questions between 2008-2013 years with Respect to PISA Competency Levels

    Directory of Open Access Journals (Sweden)

    Tuğba Aydoğdu İskenderoğlu

    2013-11-01

    Full Text Available One of the main objectives of the education is students are ready to make the conditions of the age in which they live. From this perspective approach of mathematics education changed in our age looking for answers to the question as instead of "what we teach them?", "How they use our teaching which their life?". In other words, the main purpose of education is our students collected under the mathematical literacy. Then teach them which students' knowledge which use in everyday life, to make logical inferences, interpret and solve problems related to the various situations. The purpose of this study was implemented in our country between the years of 2008-2013 SBS questions examined and categorized according to the scale which PISA mathematics proficiency. In this study, data was collected using qualitative research techniques, document analysis methods of data collection. The results of the study, the questions examined in this study all levels of math exams in 2008-2013 SBS was not appropriate questions. Questions in general 2, 3 and 4 levels of which they are located, exams include just one question which is the highest level of 5 and there have not been any questions level 6. SBS administered by the Ministry of National Education thought-provoking the absence of the upper levels questions. For this reason, math questions of SBS should be every level and be prepared questions reconsideration measurement is recommended.Key Words: PISA, Mathematics competency levels, Mathematics competency scale, Mathematics problems of SBS

  5. Geomechanical problems of an underground storage of spent nuclear fuel and their mathematic modelling

    Directory of Open Access Journals (Sweden)

    Antonín Hájek

    2007-01-01

    Full Text Available The paper is devoted to the use of mathematical modelling for analysis of the thermo-mechanical (T-M processes, which are relevant for the assessment of underground repositories of the spent nuclear fuel. Wes shall discuss mathematical formulation, numerical methods and parallel alghorithms, which are capable to solve large-scale complicated and coupled 3D problems. Particularly, we show an application of the described methods and parallel computer simulations for analysis of model problems concerning the Swedish KBS3 concept of underground repository.

  6. The Effects of Problem Posing on Student Mathematical Learning: A Meta-Analysis

    Science.gov (United States)

    Rosli, Roslinda; Capraro, Mary Margaret; Capraro, Robert M.

    2014-01-01

    The purpose of the study was to meta-synthesize research findings on the effectiveness of problem posing and to investigate the factors that might affect the incorporation of problem posing in the teaching and learning of mathematics. The eligibility criteria for inclusion of literature in the meta-analysis was: published between 1989 and 2011,…

  7. The heat treatment of steel. A mathematical control problem

    Energy Technology Data Exchange (ETDEWEB)

    Hoemberg, Dietmar; Kern, Daniela

    2009-07-21

    The goal of this paper is to show how the heat treatment of steel can be modelled in terms of a mathematical optimal control problem. The approach is applied to laser surface hardening and the cooling of a steel slab including mechanical effects. Finally, it is shown how the results can be utilized in industrial practice by a coupling with machine-based control. (orig.)

  8. Academic Motivation Maintenance for Students While Solving Mathematical Problems in the Middle School

    OpenAIRE

    M. Rodionov; Z. Dedovets

    2015-01-01

    The level and type of student academic motivation are the key factors in their development and determine the effectiveness of their education. Improving motivation is very important with regard to courses on middle school mathematics. This article examines the general position regarding the practice of academic motivation. It also examines the particular features of mathematical problem solving in a school setting.

  9. Schema-Based Instruction with Concrete and Virtual Manipulatives to Teach Problem Solving to Students with Autism

    Science.gov (United States)

    Root, Jenny R.; Browder, Diane M.; Saunders, Alicia F.; Lo, Ya-yu

    2017-01-01

    The current study evaluated the effects of modified schema-based instruction on the mathematical word problem solving skills of three elementary students with autism spectrum disorders and moderate intellectual disability. Participants learned to solve compare problem type with themes that related to their interests and daily experiences. In…

  10. Discrete Mathematics

    DEFF Research Database (Denmark)

    Sørensen, John Aasted

    2011-01-01

    ; construct a finite state machine for a given application. Apply these concepts to new problems. The teaching in Discrete Mathematics is a combination of sessions with lectures and students solving problems, either manually or by using Matlab. Furthermore a selection of projects must be solved and handed...... to accomplish the following: -Understand and apply formal representations in discrete mathematics. -Understand and apply formal representations in problems within discrete mathematics. -Understand methods for solving problems in discrete mathematics. -Apply methods for solving problems in discrete mathematics...... to new problems. Relations and functions: Define a product set; define and apply equivalence relations; construct and apply functions. Apply these concepts to new problems. Natural numbers and induction: Define the natural numbers; apply the principle of induction to verify a selection of properties...

  11. Mathematical Enculturation from the Students' Perspective: Shifts in Problem-Solving Beliefs and Behaviour during the Bachelor Programme

    Science.gov (United States)

    Perrenet, Jacob; Taconis, Ruurd

    2009-01-01

    This study investigates the changes in mathematical problem-solving beliefs and behaviour of mathematics students during the years after entering university. Novice bachelor students fill in a questionnaire about their problem-solving beliefs and behaviour. At the end of their bachelor programme, as experienced bachelor students, they again fill…

  12. Behaviour of mathematics and physics students in solving problem of Vector-Physics context

    Science.gov (United States)

    Sardi; Rizal, M.; Mansyur, J.

    2018-04-01

    This research aimed to describe behaviors of mathematics and physics students in solving problem of the vector concept in physics context. The subjects of the research were students who enrolled in Mathematics Education Study Program and Physics Education Study Program of FKIP Universitas Tadulako. The selected participants were students who received the highest score in vector fundamental concept test in each study program. The data were collected through thinking-aloud activity followed by an interview. The steps of data analysis included data reduction, display, and conclusion drawing. The credibility of the data was tested using a triangulation method. Based on the data analysis, it can be concluded that the two groups of students did not show fundamental differences in problem-solving behavior, especially in the steps of understanding the problem (identifying, collecting and analyzing facts and information), planning (looking for alternative strategies) and conducting the alternative strategy. The two groups were differ only in the evaluation aspect. In contrast to Physics students who evaluated their answer, mathematics students did not conducted an evaluation activity on their work. However, the difference was not caused by the differences in background knowledge.

  13. The Prevalent Rate of Problem-Solving Approach in Teaching Mathematics in Ghanaian Basic Schools

    Science.gov (United States)

    Nyala, Joseph; Assuah, Charles; Ayebo, Abraham; Tse, Newel

    2016-01-01

    Stakeholders of mathematics education decry the rate at which students' performance are falling below expectation; they call for a shift to practical methods of teaching the subject in Ghanaian basic schools. The study explores the extent to which Ghanaian basic school mathematics teachers use problem-solving approach in their lessons. The…

  14. Mathematical and numerical analysis of PN models for photons transport problems

    International Nuclear Information System (INIS)

    Valentin, Xavier

    2015-01-01

    Computational costs for direct numerical simulations of photon transport problems are very high in terms of CPU time and memory. One way to tackle this issue is to develop reduced models that a cheaper to solve numerically. There exists number of these models: moments models, discrete ordinates models (S N ), diffusion-like models... In this thesis, we focus on P N models in which the transport operator is approached by mean of a truncated development on the spherical harmonics basis. These models are arbitrary accurate in the angular dimension and are rotationally invariants (in multiple space dimensions). The latter point is fundamental when one wants to simulate inertial confinement fusion (ICF) experiments where the spherical symmetry plays an important part in the accuracy of the numerical solutions. We study the mathematical structure of the PN models and construct a new numerical method in the special case of a one dimensional space dimension with spherical symmetry photon transport problems. We first focus on a linear transport problem in the vacuum. Even in this simple case, it appears in the P N equations geometrical source terms that are stiff in the neighborhood of r = 0 and thus hard to discretize. Existing numerical methods are not satisfactory for multiple reasons: (1) inaccuracy in the neighborhood of r = 0 ('flux-dip'), (2) do not capture steady states (well-balanced scheme), (3) no stability proof. Following recent works, we develop a new well-balanced scheme for which we show the L 2 stability. We then extend the scheme for photon transport problems within a no moving media, the linear Boltzmann equation, and interest ourselves on its behavior in the diffusion limit (asymptotic-preserving property). In a second part, we consider radiation hydrodynamics problems. Since modelization of these problems is still under discussion in the literature, we compare a set of existing models by mean of mathematical analysis and establish a hierarchy

  15. Solving a bi-objective mathematical programming model for bloodmobiles location routing problem

    Directory of Open Access Journals (Sweden)

    Masoud Rabbani

    2017-01-01

    Full Text Available Perishability of platelets, uncertainty of donors’ arrival and conflicting views in platelet supply chain have made platelet supply chain planning a problematic issue. In this paper, mobile blood collection system for platelet production is investigated. Two mathematical models are presented to cover the bloodmobile collection planning problem. The first model is a multi-objective fuzzy mathematical programming in which the bloodmobiles locations are considered with the aim of maximizing potential amount of blood collection and minimizing the operational cost. The second model is a vehicle routing problem with time windows which studies the shuttles routing problem. To tackle the first model, it is reformulated as a crisp multi objective linear programming model and then solved through a fuzzy multi objective programming approach. Several sensitivity analysis are conducted on important parameters to demonstrate the applicability of the proposed model. The proposed model is then solved by using a tailored Simulated Annealing (SA algorithm. The numerical results demonstrate promising efficiency of the proposed solution method.

  16. Collection of proceedings of the international conference on programming and mathematical methods for solution of physical problems

    International Nuclear Information System (INIS)

    1994-01-01

    Traditional International Conference on programming and mathematical methods for solution of physical problems took place in Dubna in Jun, 14-19, 1993. More than 160 scientists from 14 countries participated in the Conference. They presented about 120 reports, the range of problems including computerized information complexes, experimental data acquisition and processing systems, mathematical simulation and calculation experiment in physics, analytical and numerical methods for solution of physical problems

  17. PROBLEMS RELATED TO THE ADAPTATION OF SOME ENGLISH LOAN WORDS IN CROATIAN AND POLISH LANGUAGE

    Directory of Open Access Journals (Sweden)

    Maciek Czerwiński

    2000-01-01

    Full Text Available Many languages in the new era are open to the influence of English language. There is no doubt about it but process of adaptation in various languages can run quite different. I focused in the article on such problems. Adaptation itself of every single loan word (and thus English loan word as well runs on the three basic levels: phonological, morphological and semantic. On every level we find some particular tendencies in particular language. To research them properly we should look at another language and compare them.

  18. Creativity in Unique Problem-Solving in Mathematics and Its Influence on Motivation for Learning

    Science.gov (United States)

    Bishara, Saied

    2016-01-01

    This research study investigates the ability of students to tackle the solving of unique mathematical problems in the domain of numerical series, verbal and formal, and its influence on the motivation of junior high students with learning disabilities in the Arab sector. Two instruments were used to collect the data: mathematical series were…

  19. Examining the design features of a communication-rich, problem-centred mathematics professional development

    Science.gov (United States)

    de Araujo, Zandra; Orrill, Chandra Hawley; Jacobson, Erik

    2018-04-01

    While there is considerable scholarship describing principles for effective professional development, there have been few attempts to examine these principles in practice. In this paper, we identify and examine the particular design features of a mathematics professional development experience provided for middle grades teachers over 14 weeks. The professional development was grounded in a set of mathematical tasks that each had one right answer, but multiple solution paths. The facilitator engaged participants in problem solving and encouraged participants to work collaboratively to explore different solution paths. Through analysis of this collaborative learning environment, we identified five design features for supporting teacher learning of important mathematics and pedagogy in a problem-solving setting. We discuss these design features in depth and illustrate them by presenting an elaborated example from the professional development. This study extends the existing guidance for the design of professional development by examining and operationalizing the relationships among research-based features of effective professional development and the enacted features of a particular design.

  20. Modeling Students' Problem Solving Performance in the Computer-Based Mathematics Learning Environment

    Science.gov (United States)

    Lee, Young-Jin

    2017-01-01

    Purpose: The purpose of this paper is to develop a quantitative model of problem solving performance of students in the computer-based mathematics learning environment. Design/methodology/approach: Regularized logistic regression was used to create a quantitative model of problem solving performance of students that predicts whether students can…

  1. Mathematics Prerequisites for Introductory Geoscience Courses: Using Technology to Help Solve the Problem

    Science.gov (United States)

    Burn, H. E.; Wenner, J. M.; Baer, E. M.

    2011-12-01

    The quantitative components of introductory geoscience courses can pose significant barriers to students. Many academic departments respond by stripping courses of their quantitative components or by attaching prerequisite mathematics courses [PMC]. PMCs cause students to incur additional costs and credits and may deter enrollment in introductory courses; yet, stripping quantitative content from geoscience courses masks the data-rich, quantitative nature of geoscience. Furthermore, the diversity of math skills required in geoscience and students' difficulty with transferring mathematical knowledge across domains suggest that PMCs may be ineffective. Instead, this study explores an alternative strategy -- to remediate students' mathematical skills using online modules that provide students with opportunities to build contextual quantitative reasoning skills. The Math You Need, When You Need It [TMYN] is a set of modular online student resources that address mathematical concepts in the context of the geosciences. TMYN modules are online resources that employ a "just-in-time" approach - giving students access to skills and then immediately providing opportunities to apply them. Each module places the mathematical concept in multiple geoscience contexts. Such an approach illustrates the immediate application of a principle and provides repeated exposure to a mathematical skill, enhancing long-term retention. At the same time, placing mathematics directly in several geoscience contexts better promotes transfer of learning by using similar discourse (words, tools, representations) and context that students will encounter when applying mathematics in the future. This study uses quantitative and qualitative data to explore the effectiveness of TMYN modules in remediating students' mathematical skills. Quantitative data derive from ten geoscience courses that used TMYN modules during the fall 2010 and spring 2011 semesters; none of the courses had a PMC. In all courses

  2. Evaluating the Use of Problem-Based Video Podcasts to Teach Mathematics in Higher Education

    Science.gov (United States)

    Kay, Robin; Kletskin, Ilona

    2012-01-01

    Problem-based video podcasts provide short, web-based, audio-visual explanations of how to solve specific procedural problems in subject areas such as mathematics or science. A series of 59 problem-based video podcasts covering five key areas (operations with functions, solving equations, linear functions, exponential and logarithmic functions,…

  3. Metacognition, Motivation and Emotions: Contribution of Self-Regulated Learning to Solving Mathematical Problems

    Directory of Open Access Journals (Sweden)

    Meirav Tzohar-Rozen

    2014-11-01

    Full Text Available Mathematical problem solving is among the most valuable aspects of mathematics education. It is also the hardest for elementary school students (Verschaffel, Greer & De Corte, 2000. Students experience cognitive and metacognitive difficulties in this area and develop negative emotions and poor motivation which hamper their efforts (Kramarski, Weiss, & Kololshi-Minsker, 2010. 9–11 seems the critical stage for developing attitudes and emotional reactions towards mathematics (Artino, 2009. These metacognitive and motivational-emotional factors are fundamental components of Self-Regulated Learning (SRL, a non-innate process requiring systematic, explicit student training (Pintrich, 2000; Zimmerman, 2000. Most self-regulation studies relating to problem-solving focus on metacognition. Few explore the motivational-emotional component. This study aimed to develop, examine, and compare two SRL interventions dealing with two additional components of self-regulation: metacognitive regulation (MC and motivational-emotional regulation (ME. It also sought to examine the significance of these components and their contribution to learners' problem-solving achievements and self-regulation. The study examined 118 fifth grade students, randomly assigned to two groups. Pre- and post-intervention, the two groups completed self-regulation questionnaires relating to metacognition, motivation, and emotion. They also solved arithmetic series problems presented in two ways (verbal form and numeric form. After intervention we also examined a novel transfer problem. The intervention consisted of 10 hours for 5 weeks. Following the intervention the groups exhibited similar improvements across all the problems. The MC group performed best in metacognitive self-regulation and the ME group performed best in certain motivational-emotional aspects of self-regulation. Research implications are discussed.

  4. Mathematical problems of the dynamics of incompressible fluid on a rotating sphere

    CERN Document Server

    Skiba, Yuri N

    2017-01-01

    This book presents selected mathematical problems involving the dynamics of a two-dimensional viscous and ideal incompressible fluid on a rotating sphere. In this case, the fluid motion is completely governed by the barotropic vorticity equation (BVE), and the viscosity term in the vorticity equation is taken in its general form, which contains the derivative of real degree of the spherical Laplace operator. This work builds a bridge between basic concepts and concrete outcomes by pursuing a rich combination of theoretical, analytical and numerical approaches, and is recommended for specialists developing mathematical methods for application to problems in physics, hydrodynamics, meteorology and geophysics, as well for upper undergraduate or graduate students in the areas of dynamics of incompressible fluid on a rotating sphere, theory of functions on a sphere, and flow stability.

  5. Learning to Solve Addition and Subtraction Word Problems in English as an Imported Language

    Science.gov (United States)

    Verzosa, Debbie Bautista; Mulligan, Joanne

    2013-01-01

    This paper reports an intervention phase of a design study aimed to assist second-grade Filipino children in solving addition word problems in English, a language they primarily encounter only in school. With Filipino as the medium of instruction, an out-of-school pedagogical intervention providing linguistic and representational scaffolds was…

  6. Multimodal Languaging as a Pedagogical Model—A Case Study of the Concept of Division in School Mathematics

    Directory of Open Access Journals (Sweden)

    Jorma Joutsenlahti

    2017-01-01

    Full Text Available The purpose of this study is to present a multimodal languaging model for mathematics education. The model consists of mathematical symbolic language, a pictorial language, and a natural language. By applying this model, the objective was to study how 4th grade pupils (N = 21 understand the concept of division. The data was collected over six hours of teaching sessions, during which the pupils expressed their mathematical thinking mainly by writing and drawing. Their productions, as well as questionnaire after the process, were analyzed qualitatively. The results show that, in expressing the mathematical problem in verbal form, most of the students saw it as a division into parts. It was evident from the pupils’ texts and drawings that the mathematical expression of subtraction could be interpreted in three different ways. It was found that the pupils enjoyed using writing in the solution of word problems, and it is suggested that the use of different modes in expressing mathematical thinking may both strengthen the learning of mathematical concepts and support the evaluation of learning.

  7. A Further Study of Productive Failure in Mathematical Problem Solving: Unpacking the Design Components

    Science.gov (United States)

    Kapur, Manu

    2011-01-01

    This paper replicates and extends my earlier work on productive failure in mathematical problem solving (Kapur, doi:10.1007/s11251-009-9093-x, 2009). One hundred and nine, seventh-grade mathematics students taught by the same teacher from a Singapore school experienced one of three learning designs: (a) traditional lecture and practice (LP), (b)…

  8. Improving attitudes toward mathematics learning with problem posing in class VIII

    Science.gov (United States)

    Vionita, Alfha; Purboningsih, Dyah

    2017-08-01

    This research is classroom action research which is collaborated to improve student's behavior toward math and mathematics learning at class VIII by using problem posing approach. The subject of research is all of students grade VIIIA which consist of 32 students. This research has been held on two period, first period is about 3 times meeting, and second period is about 4 times meeting. The instrument of this research is implementation of learning observation's guidance by using problem posing approach. Cycle test has been used to measure cognitive competence, and questionnaire to measure the students' behavior in mathematics learning process. The result of research shows the students' behavior has been improving after using problem posing approach. It is showed by the behavior's criteria of students that has increasing result from the average in first period to high in second period. Furthermore, the percentage of test result is also improve from 68,75% in first period to 78,13% in second period. On the other hand, the implementation of learning observation by using problem posing approach has also improving and it is showed by the average percentage of teacher's achievement in first period is 89,2% and student's achievement 85,8%. These results get increase in second period for both teacher and students' achievement which are 94,4% and 91,11%. As a result, students' behavior toward math learning process in class VIII has been improving by using problem posing approach.

  9. The effect of creative problem solving on students’ mathematical adaptive reasoning

    Science.gov (United States)

    Muin, A.; Hanifah, S. H.; Diwidian, F.

    2018-01-01

    This research was conducted to analyse the effect of creative problem solving (CPS) learning model on the students’ mathematical adaptive reasoning. The method used in this study was a quasi-experimental with randomized post-test only control group design. Samples were taken as many as two classes by cluster random sampling technique consisting of experimental class (CPS) as many as 40 students and control class (conventional) as many as 40 students. Based on the result of hypothesis testing with the t-test at the significance level of 5%, it was obtained that significance level of 0.0000 is less than α = 0.05. This shows that the students’ mathematical adaptive reasoning skills who were taught by CPS model were higher than the students’ mathematical adaptive reasoning skills of those who were taught by conventional model. The result of this research showed that the most prominent aspect of adaptive reasoning that could be developed through a CPS was inductive intuitive. Two aspects of adaptive reasoning, which were inductive intuitive and deductive intuitive, were mostly balanced. The different between inductive intuitive and deductive intuitive aspect was not too big. CPS model can develop student mathematical adaptive reasoning skills. CPS model can facilitate development of mathematical adaptive reasoning skills thoroughly.

  10. Social class and mathematics school knowledge of two private schools in Banten Province

    Science.gov (United States)

    Siahaan, M. F.

    2018-05-01

    The purpose of this study was to identify school mathematics topics and mathematics learning experiences of two elementary schools in contrasting social class settings under an umbrella of one institution. A case study research methodology was used to examine data collected from those two Elementary schools. The data revealed that there were similarities in curriculum framework, curriculum materials but there were also significant differences in what was taught and what was experienced in those two schools. The data suggested that word problem and a pedagogy of critical thinking were implemented in one of the schools. The differences were assessed in terms of theoretical and social implications. It was concluded that social stratification of mathematical knowledge occurred

  11. Reflective Learning and Prospective Teachers' Conceptual Understanding, Critical Thinking, Problem Solving, and Mathematical Communication Skills

    Science.gov (United States)

    Junsay, Merle L.

    2016-01-01

    This is a quasi-experimental study that explored the effects of reflective learning on prospective teachers' conceptual understanding, critical thinking, problem solving, and mathematical communication skills and the relationship of these variables. It involved 60 prospective teachers from two basic mathematics classes of an institution of higher…

  12. Quantum mechanics problems in observer's mathematics

    Energy Technology Data Exchange (ETDEWEB)

    Khots, Boris; Khots, Dmitriy [Compressor Controls Corp, Des Moines, Iowa (United States); iMath Consulting LLC, Omaha, Nebraska (United States)

    2012-11-06

    This work considers the ontology, guiding equation, Schrodinger's equation, relation to the Born Rule, the conditional wave function of a subsystem in a setting of arithmetic, algebra and topology provided by Observer's Mathematics (see www.mathrelativity.com). Observer's Mathematics creates new arithmetic, algebra, geometry, topology, analysis and logic which do not contain the concept of continuum, but locally coincide with the standard fields. Certain results and communications pertaining to solutions of these problems are provided. In particular, we prove the following theorems: Theorem I (Two-slit interference). Let {Psi}{sub 1} be a wave from slit 1, {Psi}{sub 2} - from slit 2, and {Psi} = {Psi}{sub 1}+{Psi}{sub 2}. Then the probability of {Psi} being a wave equals to 0.5. Theorem II (k-bodies solution). For W{sub n} from m-observer point of view with m>log{sub 10}((2 Multiplication-Sign 10{sup 2n}-1){sup 2k}+1), the probability of standard expression of Hamiltonian variation is less than 1 and depends on n,m,k.

  13. Bringing Reality into Calculus Classrooms: Mathematizing a Real-life Problem Simulated in a Virtual Environment

    Directory of Open Access Journals (Sweden)

    Olga V. Shipulina

    2013-01-01

    Full Text Available The study explores how students, who had completed the AP calculus course, mathematized the optimal navigation real-life problem simulated in the Second Life Virtual Environment. The particular research interest was to investigate whether/how students’ empirical activity in VE influences the way of their mathematizing.

  14. Mathematical Problems in Creating Large Astronomical Catalogs

    Directory of Open Access Journals (Sweden)

    Prokhorov M. E.

    2016-12-01

    Full Text Available The next stage after performing observations and their primary reduction is to transform the set of observations into a catalog. To this end, objects that are irrelevant to the catalog should be excluded from observations and gross errors should be discarded. To transform such a prepared data set into a high-precision catalog, we need to identify and correct systematic errors. Therefore, each object of the survey should be observed several, preferably many, times. The problem formally reduces to solving an overdetermined set of equations. However, in the case of catalogs this system of equations has a very specific form: it is extremely sparse, and its sparseness increases rapidly with the number of objects in the catalog. Such equation systems require special methods for storing data on disks and in RAM, and for the choice of the techniques for their solving. Another specific feature of such systems is their high “stiffiness”, which also increases with the volume of a catalog. Special stable mathematical methods should be used in order not to lose precision when solving such systems of equations. We illustrate the problem by the example of photometric star catalogs, although similar problems arise in the case of positional, radial-velocity, and parallax catalogs.

  15. Interference thinking in constructing students’ knowledge to solve mathematical problems

    Science.gov (United States)

    Jayanti, W. E.; Usodo, B.; Subanti, S.

    2018-04-01

    This research aims to describe interference thinking in constructing students’ knowledge to solve mathematical problems. Interference thinking in solving problems occurs when students have two concepts that interfere with each other’s concept. Construction of problem-solving can be traced using Piaget’s assimilation and accommodation framework, helping to know the students’ thinking structures in solving the problems. The method of this research was a qualitative method with case research strategy. The data in this research involving problem-solving result and transcripts of interviews about students’ errors in solving the problem. The results of this research focus only on the student who experience proactive interference, where student in solving a problem using old information to interfere with the ability to recall new information. The student who experience interference thinking in constructing their knowledge occurs when the students’ thinking structures in the assimilation and accommodation process are incomplete. However, after being given reflection to the student, then the students’ thinking process has reached equilibrium condition even though the result obtained remains wrong.

  16. Efforts to Improve Mathematics Teacher Competency Through Training Program on Design Olympiad Mathematics Problems Based on Higher Order Thinking Skills in The Junior High School

    Science.gov (United States)

    Arnellis, A.; Jamaan, E. Z.; Amalita, N.

    2018-04-01

    The goal to analyse a improvement of teacher competence after being trained in preparing high-order math olympicad based on high order thinking skills in junior high school teachers in Pesisir Selatan Regency. The sample of these activities are teachers at the MGMP junior high school in Pesisir Selatan District. Evaluation of the implementation is done by giving a pre test and post test, which will measure the success rate of the implementation of this activities. The existence of the devotion activities is expected to understand the enrichment of mathematics olympiad material and training in the preparation of math olympiad questions for the teachers of South Pesisir district junior high school, motivating and raising the interest of the participants in order to follow the mathematics olympiad with the enrichment of mathematics materials and the training of problem solving about mathematics olympiad for junior high school teachers, the participants gain experience and gain insight, as well as the ins and outs of junior mathematics olympiad and implement to teachers and students in olympic competitions. The result of that the post-test is better than the result of pretest in the training of mathematics teacher competence improvement in composing the mathematics olympiad problem based on high order thinking skills of junior high school (SMP) in Pesisir Selatan District, West Sumatra, Indonesia.

  17. Investigating a Link between Pre-Calculus Students' Uses of Graphing Calculators and Their Understanding of Mathematical Symbols

    Science.gov (United States)

    Kenney, Rachael H.

    2014-01-01

    This study examined ways in which students make use of a graphing calculator and how use relates to comfort and understanding with mathematical symbols. Analysis involved examining students' words and actions in problem solving to identify evidence of algebraic insight. Findings suggest that some symbols and symbolic structures had strong…

  18. Using Metacognitive Strategies to Improve Reading Comprehension and Solve a Word Problem

    Directory of Open Access Journals (Sweden)

    Tomo Djudin

    2017-03-01

    Full Text Available This article describes briefly the theories of metacognition and the impacts of metacognitive skills on learning. The differences between cognitive strategy and metacognitive strategy were mentioned. Some strategies to improve students’ meta cognition skills in the classroom explored as well. Based on the theories, two models of metacognitive strategies instruction for deeply understanding in reading textbook and for finding a solution of words physics problem solving were developed. These models will enable students to be independent and strategic learners.

  19. Semantic Contamination and Mathematical Proof: Can a Non-Proof Prove?

    Science.gov (United States)

    Mejia-Ramos, Juan Pablo; Inglis, Matthew

    2011-01-01

    The way words are used in natural language can influence how the same words are understood by students in formal educational contexts. Here we argue that this so-called semantic contamination effect plays a role in determining how students engage with mathematical proof, a fundamental aspect of learning mathematics. Analyses of responses to…

  20. Spoken Word Recognition and Serial Recall of Words from Components in the Phonological Network

    Science.gov (United States)

    Siew, Cynthia S. Q.; Vitevitch, Michael S.

    2016-01-01

    Network science uses mathematical techniques to study complex systems such as the phonological lexicon (Vitevitch, 2008). The phonological network consists of a "giant component" (the largest connected component of the network) and "lexical islands" (smaller groups of words that are connected to each other, but not to the giant…

  1. Mathematical marriages: intercourse between mathematics and Semiotic choice.

    Science.gov (United States)

    Wagner, Roy

    2009-04-01

    This paper examines the interaction between Semiotic choices and the presentation and solution of a family of contemporary mathematical problems centred around the so-called 'stable marriage problem'. I investigate how a socially restrictive choice of signs impacts mathematical production both in terms of problem formation and of solutions. I further note how the choice of gendered language ends up constructing a reality, which duplicates the very structural framework that it imported into mathematical analysis in the first place. I go on to point out some semiotic lines of flight from this interlocking grip of mathematics and gendered language.

  2. A Metacognitive Profile of Vocational High School Student’s Field Independent in Mathematical Problem Solving

    Science.gov (United States)

    Nugraheni, L.; Budayasa, I. K.; Suwarsono, S. T.

    2018-01-01

    The study was designed to discover examine the profile of metacognition of vocational high school student of the Machine Technology program that had high ability and field independent cognitive style in mathematical problem solving. The design of this study was exploratory research with a qualitative approach. This research was conducted at the Machine Technology program of the vocational senior high school. The result revealed that the high-ability student with field independent cognitive style conducted metacognition practices well. That involved the three types of metacognition activities, consisting of planning, monitoring, and evaluating at metacognition level 2 or aware use, 3 or strategic use, 4 or reflective use in mathematical problem solving. The applicability of the metacognition practices conducted by the subject was never at metacognition level 1 or tacit use. This indicated that the participant were already aware, capable of choosing strategies, and able to reflect on their own thinking before, after, or during the process at the time of solving mathematical problems.That was very necessary for the vocational high school student of Machine Technology program.

  3. Teaching general problem-solving skills is not a substitute for, or a viable addition to, teaching mathematics

    NARCIS (Netherlands)

    Sweller, John; Clark, Richard; Kirschner, Paul A.

    2010-01-01

    Sweller, J., Clark, R., & Kirschner, P. A. (2010). Teaching general problem-solving skills is not a substitute for, or a viable addition to, teaching mathematics. Notices of the American Mathematical Society, 57, 1303-1304.

  4. Developing calculus textbook model that supported with GeoGebra to enhancing students’ mathematical problem solving and mathematical representation

    Science.gov (United States)

    Dewi, N. R.; Arini, F. Y.

    2018-03-01

    The main purpose of this research is developing and produces a Calculus textbook model that supported with GeoGebra. This book was designed to enhancing students’ mathematical problem solving and mathematical representation. There were three stages in this research i.e. define, design, and develop. The textbooks consisted of 6 chapters which each chapter contains introduction, core materials and include examples and exercises. The textbook developed phase begins with the early stages of designed the book (draft 1) which then validated by experts. Revision of draft 1 produced draft 2. The data were analyzed with descriptive statistics. The analysis showed that the Calculus textbook model that supported with GeoGebra, valid and fill up the criteria of practicality.

  5. Critical Thinking Skills Of Junior High School Female Students With High Mathematical Skills In Solving Contextual And Formal Mathematical Problems

    Science.gov (United States)

    Ismail; Suwarsono, St.; Lukito, A.

    2018-01-01

    Critical thinking is one of the most important skills of the 21st century in addition to other learning skills such as creative thinking, communication skills and collaborative skills. This is what makes researchers feel the need to conduct research on critical thinking skills in junior high school students. The purpose of this study is to describe the critical thinking skills of junior high school female students with high mathematical skills in solving contextual and formal mathematical problems. To achieve this is used qualitative research. The subject of the study was a female student of eight grade junior high school. The students’ critical thinking skills are derived from in-depth problem-based interviews using interview guidelines. Interviews conducted in this study are problem-based interviews, which are done by the subject given a written assignment and given time to complete. The results show that critical thinking skills of female high school students with high math skills are as follows: In solving the problem at the stage of understanding the problem used interpretation skills with sub-indicators: categorization, decode, and clarify meaning. At the planning stage of the problem-solving strategy is used analytical skills with sub-indicators: idea checking, argument identification and argument analysis and evaluation skills with sub indicators: assessing the argument. In the implementation phase of problem solving, inference skills are used with subindicators: drawing conclusions, and problem solving and explanatory skills with sub-indicators: problem presentation, justification procedures, and argument articulation. At the re-checking stage all steps have been employed self-regulatory skills with sub-indicators: self-correction and selfstudy.

  6. Understanding and quantifying cognitive complexity level in mathematical problem solving items

    Directory of Open Access Journals (Sweden)

    SUSAN E. EMBRETSON

    2008-09-01

    Full Text Available The linear logistic test model (LLTM; Fischer, 1973 has been applied to a wide variety of new tests. When the LLTM application involves item complexity variables that are both theoretically interesting and empirically supported, several advantages can result. These advantages include elaborating construct validity at the item level, defining variables for test design, predicting parameters of new items, item banking by sources of complexity and providing a basis for item design and item generation. However, despite the many advantages of applying LLTM to test items, it has been applied less often to understand the sources of complexity for large-scale operational test items. Instead, previously calibrated item parameters are modeled using regression techniques because raw item response data often cannot be made available. In the current study, both LLTM and regression modeling are applied to mathematical problem solving items from a widely used test. The findings from the two methods are compared and contrasted for their implications for continued development of ability and achievement tests based on mathematical problem solving items.

  7. Teaching Personal Finance Mathematical Problem Solving to Individuals with Moderate Intellectual Disability

    Science.gov (United States)

    Root, Jenny; Saunders, Alicia; Spooner, Fred; Brosh, Chelsi

    2017-01-01

    The ability to solve mathematical problems related to purchasing and personal finance is important in promoting skill generalization and increasing independence for individuals with moderate intellectual disabilities (IDs). Using a multiple probe across participant design, this study investigated the effects of modified schema-based instruction…

  8. Assessing metacognition of grade 2 and grade 4 students using an adaptation of multi-method interview approach during mathematics problem-solving

    Science.gov (United States)

    Kuzle, A.

    2018-06-01

    The important role that metacognition plays as a predictor for student mathematical learning and for mathematical problem-solving, has been extensively documented. But only recently has attention turned to primary grades, and more research is needed at this level. The goals of this paper are threefold: (1) to present metacognitive framework during mathematics problem-solving, (2) to describe their multi-method interview approach developed to study student mathematical metacognition, and (3) to empirically evaluate the utility of their model and the adaptation of their approach in the context of grade 2 and grade 4 mathematics problem-solving. The results are discussed not only with regard to further development of the adapted multi-method interview approach, but also with regard to their theoretical and practical implications.

  9. Some applications of fractal mathematics in the evaluation of environmental problems

    Energy Technology Data Exchange (ETDEWEB)

    Thimm, H. F.; Poon, D. C.; McCormack, M.

    1997-11-01

    Application of fractal mathematics to commonly occurring environmental problems in the petroleum industry is discussed. Examples are provided to illustrate application of the technique. The specific examples cited involve the interpretation of mercury contamination data at a gas plant and the determination of the optimal volume of soil excavation at a contaminated site. 10 refs., 4 figs.

  10. A review of mathematical models in economic environmental problems

    DEFF Research Database (Denmark)

    Nahorski, Z.; Ravn, H.F.

    2000-01-01

    The paper presents a review of mathematical models used,in economic analysis of environmental problems. This area of research combines macroeconomic models of growth, as dependent on capital, labour, resources, etc., with environmental models describing such phenomena like natural resources...... exhaustion or pollution accumulation and degradation. In simpler cases the models can be treated analytically and the utility function can be optimized using, e.g., such tools as the maximum principle. In more complicated cases calculation of the optimal environmental policies requires a computer solution....

  11. The profile of conceptual comprehension of pre-service teacher in the mathematical problem solving with low emotional intelligence

    Science.gov (United States)

    Prayitno, S. H.; Suwarsono, St.; Siswono, T. Y. E.

    2018-03-01

    Conceptual comprehension in this research is the ability to use the procedures that are owned by pre-service teachers to solve problems by finding the relation of the concept to another, or can be done by identifying the type of problem and associating it with a troubleshooting procedures, or connect the mathematical symbols with mathematical ideas and incorporate them into a series of logical reasoning, or by using prior knowledge that occurred directly, through its conceptual knowledge. The goal of this research is to describe the profile of conceptual comprehensin of pre-service teachers with low emotional intelligence in mathematical problems solving. Through observation and in-depth interview with the research subject the conclusion was that: pre-service teachers with low emotional intelligence pertained to the level of formal understanding in understanding the issues, relatively to the level of intuitive understanding in planning problem solving, to the level of relational understanding in implementing the relational problem solving plan, and pertained to the level of formal understanding in looking back to solve the problem.

  12. An inverse problem for a mathematical model of aquaponic agriculture

    Science.gov (United States)

    Bobak, Carly; Kunze, Herb

    2017-01-01

    Aquaponic agriculture is a sustainable ecosystem that relies on a symbiotic relationship between fish and macrophytes. While the practice has been growing in popularity, relatively little mathematical models exist which aim to study the system processes. In this paper, we present a system of ODEs which aims to mathematically model the population and concetrations dynamics present in an aquaponic environment. Values of the parameters in the system are estimated from the literature so that simulated results can be presented to illustrate the nature of the solutions to the system. As well, a brief sensitivity analysis is performed in order to identify redundant parameters and highlight those which may need more reliable estimates. Specifically, an inverse problem with manufactured data for fish and plants is presented to demonstrate the ability of the collage theorem to recover parameter estimates.

  13. Occupational Therapy Interventions Effect on Mathematical Problems in Students with Special Learning Disorders

    Directory of Open Access Journals (Sweden)

    Rogaieh Mohammadi

    2009-10-01

    Full Text Available Objectives: Dyscalculia is specific learning disabilities affecting the acquisition of mathematic skills in an otherwise normal child. The aim of this study was investigation of occupational therapy interventions effect on mathematical problems in students with special learning disorders. Methods: 40 students with dyscalculia (2-5 grades were selected and divided through randomized permuted blocks method into two groups 20 persons as intervention group and the others as the control group. Initially both of groups were administered by the "Iran Key math Test". Then intervention group received occupational therapy interventions for 20 sessions individually and two groups were administered by the Test again. Data was analyzed by using Paired and Independent t-tests. Results: By the paired sample t-test the mean of total marks of Iran Key math Test demonstrated statistically significant difference in both of groups (P<0.05, but the measure of difference in intervention group was more than control group. The mean of marks of Basic Concepts, Operations and Applications demonstrated statistically significant difference at intervention group. Discussion: Occupational therapy interventions had clinical effect on mathematical problems in students with special learning disorders.

  14. Working Memory, Attention, and Mathematical Problem Solving: A Longitudinal Study of Elementary School Children

    Science.gov (United States)

    Swanson, H. Lee

    2011-01-01

    The role of working memory (WM) in children's growth in mathematical problem solving was examined in a longitudinal study of children (N = 127). A battery of tests was administered that assessed problem solving, achievement, WM, and cognitive processing (inhibition, speed, phonological coding) in Grade 1 children, with follow-up testing in Grades…

  15. Perceptual Learning in Early Mathematics: Interacting with Problem Structure Improves Mapping, Solving and Fluency

    Science.gov (United States)

    Thai, Khanh-Phuong; Son, Ji Y.; Hoffman, Jessica; Devers, Christopher; Kellman, Philip J.

    2014-01-01

    Mathematics is the study of structure but students think of math as solving problems according to rules. Students can learn procedures, but they often have trouble knowing when to apply learned procedures, especially to problems unlike those they trained with. In this study, the authors rely on the psychological mechanism of perceptual learning…

  16. METHOD OF GREEN FUNCTIONS IN MATHEMATICAL MODELLING FOR TWO-POINT BOUNDARY-VALUE PROBLEMS

    Directory of Open Access Journals (Sweden)

    E. V. Dikareva

    2015-01-01

    Full Text Available Summary. In many applied problems of control, optimization, system theory, theoretical and construction mechanics, for problems with strings and nods structures, oscillation theory, theory of elasticity and plasticity, mechanical problems connected with fracture dynamics and shock waves, the main instrument for study these problems is a theory of high order ordinary differential equations. This methodology is also applied for studying mathematical models in graph theory with different partitioning based on differential equations. Such equations are used for theoretical foundation of mathematical models but also for constructing numerical methods and computer algorithms. These models are studied with use of Green function method. In the paper first necessary theoretical information is included on Green function method for multi point boundary-value problems. The main equation is discussed, notions of multi-point boundary conditions, boundary functionals, degenerate and non-degenerate problems, fundamental matrix of solutions are introduced. In the main part the problem to study is formulated in terms of shocks and deformations in boundary conditions. After that the main results are formulated. In theorem 1 conditions for existence and uniqueness of solutions are proved. In theorem 2 conditions are proved for strict positivity and equal measureness for a pair of solutions. In theorem 3 existence and estimates are proved for the least eigenvalue, spectral properties and positivity of eigenfunctions. In theorem 4 the weighted positivity is proved for the Green function. Some possible applications are considered for a signal theory and transmutation operators.

  17. Application of a Mathematical Model to an Advertisement Reservation Problem

    Directory of Open Access Journals (Sweden)

    Ozlem COSGUN

    2013-01-01

    Full Text Available Television networks provide TV programs free of charge to the public. However, they acquire their revenue by telecasting advertisements in the midst of continuing programs or shows. A key problem faced by the TV networks in Turkey is how to accept and televise the advertisements reserved by a client on a specified advertisement break which we called “Advertisement Reservation Problem” (ARP. The problem is complicated by limited time inventory, by different rating points for different target groups, competition avoidance and the relationship between TV networks and clients. In this study we have developed a mathematical model for advertisement reservation problem and extended this model for some cases encountered in real business life. We have also discussed how these cases affect the decisions of a TV network. Mixed integer linear programming approach is proposed to solve these problems. This approach has been implemented to a case taken from one of the biggest TV networks of Turkey.

  18. Analysis of creative mathematic thinking ability in problem based learning model based on self-regulation learning

    Science.gov (United States)

    Munahefi, D. N.; Waluya, S. B.; Rochmad

    2018-03-01

    The purpose of this research identified the effectiveness of Problem Based Learning (PBL) models based on Self Regulation Leaning (SRL) on the ability of mathematical creative thinking and analyzed the ability of mathematical creative thinking of high school students in solving mathematical problems. The population of this study was students of grade X SMA N 3 Klaten. The research method used in this research was sequential explanatory. Quantitative stages with simple random sampling technique, where two classes were selected randomly as experimental class was taught with the PBL model based on SRL and control class was taught with expository model. The selection of samples at the qualitative stage was non-probability sampling technique in which each selected 3 students were high, medium, and low academic levels. PBL model with SRL approach effectived to students’ mathematical creative thinking ability. The ability of mathematical creative thinking of low academic level students with PBL model approach of SRL were achieving the aspect of fluency and flexibility. Students of academic level were achieving fluency and flexibility aspects well. But the originality of students at the academic level was not yet well structured. Students of high academic level could reach the aspect of originality.

  19. Comparison of student's learning achievement through realistic mathematics education (RME) approach and problem solving approach on grade VII

    Science.gov (United States)

    Ilyas, Muhammad; Salwah

    2017-02-01

    The type of this research was experiment. The purpose of this study was to determine the difference and the quality of student's learning achievement between students who obtained learning through Realistic Mathematics Education (RME) approach and students who obtained learning through problem solving approach. This study was a quasi-experimental research with non-equivalent experiment group design. The population of this study was all students of grade VII in one of junior high school in Palopo, in the second semester of academic year 2015/2016. Two classes were selected purposively as sample of research that was: year VII-5 as many as 28 students were selected as experiment group I and VII-6 as many as 23 students were selected as experiment group II. Treatment that used in the experiment group I was learning by RME Approach, whereas in the experiment group II by problem solving approach. Technique of data collection in this study gave pretest and posttest to students. The analysis used in this research was an analysis of descriptive statistics and analysis of inferential statistics using t-test. Based on the analysis of descriptive statistics, it can be concluded that the average score of students' mathematics learning after taught using problem solving approach was similar to the average results of students' mathematics learning after taught using realistic mathematics education (RME) approach, which are both at the high category. In addition, It can also be concluded that; (1) there was no difference in the results of students' mathematics learning taught using realistic mathematics education (RME) approach and students who taught using problem solving approach, (2) quality of learning achievement of students who received RME approach and problem solving approach learning was same, which was at the high category.

  20. Gender differences in algebraic thinking ability to solve mathematics problems

    Science.gov (United States)

    Kusumaningsih, W.; Darhim; Herman, T.; Turmudi

    2018-05-01

    This study aimed to conduct a gender study on students' algebraic thinking ability in solving a mathematics problem, polyhedron concept, for grade VIII. This research used a qualitative method. The data was collected using: test and interview methods. The subjects in this study were eight male and female students with different level of abilities. It was found that the algebraic thinking skills of male students reached high group of five categories. They were superior in terms of reasoning and quick understanding in solving problems. Algebraic thinking ability of high-achieving group of female students also met five categories of algebraic thinking indicators. They were more diligent, tenacious and thorough in solving problems. Algebraic thinking ability of male students in medium category only satisfied three categories of algebraic thinking indicators. They were sufficient in terms of reasoning and understanding in solving problems. Algebraic thinking ability group of female students in medium group also satisfied three categories of algebraic thinking indicators. They were fairly diligent, tenacious and meticulous on working on the problems.

  1. Working Together to Improve the Quality of Mathematics Education ...

    African Journals Online (AJOL)

    Prof

    Key words: Parents; mathematics education; perception; school climate; .... elementary school children, established that parents with higher college degrees ..... International Journal of Mathematical Education in Science and Technology,.

  2. The Impact of Problem-Based Learning Approach to Senior High School Students' Mathematics Critical Thinking Ability

    Science.gov (United States)

    Widyatiningtyas, Reviandari; Kusumah, Yaya S.; Sumarmo, Utari; Sabandar, Jozua

    2015-01-01

    The study reported the findings of an only post-test control group research design and aims to analyze the influence of problem-based learning approach, school level, and students' prior mathematical ability to student's mathematics critical thinking ability. The research subjects were 140 grade ten senior high school students coming from…

  3. The Transitory Phase to the Attainment of Self-Regulatory Skill in Mathematical Problem Solving

    Science.gov (United States)

    Lazakidou, G.; Paraskeva, F.; Retalis, S.

    2007-01-01

    Three phases of development of self-regulatory skill in the domain of mathematical problem solving were designed to examine students' behaviour and the effects on their problem solving ability. Forty-eight Grade 4 students (10 year olds) participated in this pilot study. The students were randomly assigned to one of three groups, each representing…

  4. Solicited versus Unsolicited Metacognitive Prompts for Fostering Mathematical Problem Solving Using Multimedia

    Science.gov (United States)

    Kramarski, Bracha; Friedman, Sheli

    2014-01-01

    The study examined how student control over metacognitive prompts in a multimedia environment affects students' ability to solve mathematical problems in immediate comprehension tasks using a multimedia program and a delayed-transfer test. It also examined the effect on metacognitive discourse, mental effort, and engagement with multimedia-based…

  5. Problem Solving Strategies of Girls and Boys in Single-Sex Mathematics Classrooms

    Science.gov (United States)

    Che, Megan; Wiegert, Elaine; Threlkeld, Karen

    2012-01-01

    This study examines patterns in middle-grade boys' and girls' written problem solving strategies for a mathematical task involving proportional reasoning. The students participating in this study attend a coeducational charter middle school with single-sex classrooms. One hundred nineteen sixth-grade students' responses are analyzed by gender…

  6. Using mathematics to solve real world problems: the role of enablers

    Science.gov (United States)

    Geiger, Vincent; Stillman, Gloria; Brown, Jill; Galbriath, Peter; Niss, Mogens

    2018-03-01

    The purpose of this article is to report on a newly funded research project in which we will investigate how secondary students apply mathematical modelling to effectively address real world situations. Through this study, we will identify factors, mathematical, cognitive, social and environmental that "enable" year 10/11 students to successfully begin the modelling process, that is, formulate and mathematise a real world problem. The 3-year study will take a design research approach in working intensively with six schools across two educational jurisdictions. It is anticipated that this research will generate new theoretical and practical insights into the role of "enablers" within the process of mathematisation, leading to the development of principles for the design and implementation for tasks that support students' development as modellers.

  7. Variable Neighbourhood Search and Mathematical Programming for Just-in-Time Job-Shop Scheduling Problem

    Directory of Open Access Journals (Sweden)

    Sunxin Wang

    2014-01-01

    Full Text Available This paper presents a combination of variable neighbourhood search and mathematical programming to minimize the sum of earliness and tardiness penalty costs of all operations for just-in-time job-shop scheduling problem (JITJSSP. Unlike classical E/T scheduling problem with each job having its earliness or tardiness penalty cost, each operation in this paper has its earliness and tardiness penalties, which are paid if the operation is completed before or after its due date. Our hybrid algorithm combines (i a variable neighbourhood search procedure to explore the huge feasible solution spaces efficiently by alternating the swap and insertion neighbourhood structures and (ii a mathematical programming model to optimize the completion times of the operations for a given solution in each iteration procedure. Additionally, a threshold accepting mechanism is proposed to diversify the local search of variable neighbourhood search. Computational results on the 72 benchmark instances show that our algorithm can obtain the best known solution for 40 problems, and the best known solutions for 33 problems are updated.

  8. Parallelization of mathematical library for generalized eigenvalue problem for real band matrices

    International Nuclear Information System (INIS)

    Tanaka, Yasuhisa.

    1997-05-01

    This research has focused on a parallelization of the mathematical library for a generalized eigenvalue problem for real band matrices on IBM SP and Hitachi SR2201. The origin of the library is LASO (Lanczos Algorithm with Selective Orthogonalization), which was developed on the basis of Block Lanczos method for standard eigenvalue problem for real band matrices at Texas University. We adopted D.O.F. (Degree Of Freedom) decomposition method for a parallelization of this library, and evaluated its parallel performance. (author)

  9. From immunology to MRI data anlysis: Problems in mathematical biology

    Science.gov (United States)

    Waters, Ryan Samuel

    This thesis represents a collection of four distinct biological projects rising from immunology and metabolomics that required unique and creative mathematical approaches. One project focuses on understanding the role IL-2 plays in immune response regulation and exploring how these effects can be altered. We developed several dynamic models of the receptor signaling network which we analyze analytically and numerically. In a second project focused also on MS, we sought to create a system for grading magnetic resonance images (MRI) with good correlation with disability. The goal is for these MRI scores to provide a better standard for large-scale clinical drug trials, which limits the bias associated with differences in available MRI technology and general grader/participant variability. The third project involves the study of the CRISPR adaptive immune system in bacteria. Bacterial cells recognize and acquire snippets of exogenous genetic material, which they incorporate into their DNA. In this project we explore the optimal design for the CRISPR system given a viral distribution to maximize its probability of survival. The final project involves the study of the benefits for colocalization of coupled enzymes in metabolic pathways. The hypothesized kinetic advantage, known as `channeling', of putting coupled enzymes closer together has been used as justification for the colocalization of coupled enzymes in biological systems. We developed and analyzed a simple partial differential equation of the diffusion of the intermediate substrate between coupled enzymes to explore the phenomena of channeling. The four projects of my thesis represent very distinct biological problems that required a variety of techniques from diverse areas of mathematics ranging from dynamical modeling to statistics, Fourier series and calculus of variations. In each case, quantitative techniques were used to address biological questions from a mathematical perspective ultimately providing

  10. Turkish Primary School Students' Strategies in Solving a Non-Routine Mathematical Problem and Some Implications for the Curriculum Design and Implementation

    Science.gov (United States)

    Erdogan, Abdulkadir

    2015-01-01

    Turkish primary mathematics curriculum emphasizes the role of problem solving for teaching mathematics and pays particular attention to problem solving strategies. Patterns as a subject and the use of patterns as a non-routine problem solving strategy are also emphasized in the curriculum. The primary purpose of this study was to determine how…

  11. Review of Street-Fighting Mathematics: The Art of Educated Guessing and Opportunistic Problem Solving by Sanjoy Mahajan

    Directory of Open Access Journals (Sweden)

    Thomas J. Pfaff

    2015-07-01

    Full Text Available Mahajan, Sanjoy. Street-Fighting Mathematics: The Art of Educated Guessing and Opportunistic Problem Solving (The MIT Press, Cambridge, Massachusetts, 2010. 152 pp. ISBN 978--0--262--51429--3 Street-Fighting Mathematics is an engaging collection of problem-solving techniques. The book is not for a general audience, as it requires a significant level of mathematical and scientific background knowledge. In particular, most of the book requires knowledge of Calculus I and there are examples that will require knowledge of Physics. At the same time, there are parts of the book that don't require this much background. While the title of the book may be misleading, as it is really street-fighting mathematics for people with a fair amount of training in the subject, there is a lot to be gained from reading this book, and calculus teachers may find it to be a useful resource.

  12. The Interrelationship of Sex, Visual Spatial Abilities, and Mathematical Problem Solving Ability in Grade Seven. Parts 1, 2, and 3.

    Science.gov (United States)

    Schonberger, Ann Koch

    This three-volume report deals with the hypothesis that males are more successful at solving mathematical and spatial problems than females. The general relationship between visual spatial abilities and mathematical problem-solving ability is also investigated. The research sample consisted of seventh graders. Each pupil took five spatial tests…

  13. THE EFFECT OF NON-ROUTINE GEOMETRY PROBLEM ON ELEMENTARY STUDENTS BELIEF IN MATHEMATICS: A CASE STUDY

    Directory of Open Access Journals (Sweden)

    Khoerul Umam

    2018-03-01

    Full Text Available Many learners hold traditional beliefs about perimeter and area that a shape with a larger area must have a larger perimeter while shape with the same perimeter must have the same area. To address this issue, non-routine geometry problem is given. This qualitative descriptive research used to reach the goal and to explore the effect of non-routine geometry problem on elementary student belief in mathematics. The instrument has been developed to accommodate intuitive student belief and student’s belief about the concept of perimeter. The results provide evidence that students’ intuitive belief about perimeter can be change through non-routine geometry problem which is required understanding and some mathematical analysis. Fortunately, the problem has helped the elementary students revise and correct their beliefs, thoughts, and understandings relating to the circumference of shape.

  14. Metacognitive experience of mathematics education students in open start problem solving based on intrapersonal intelligence

    Science.gov (United States)

    Sari, D. P.; Usodo, B.; Subanti, S.

    2018-04-01

    This research aims to describe metacognitive experience of mathematics education students with strong, average, and weak intrapersonal intelligence in open start problem solving. Type of this research was qualitative research. The research subject was mathematics education students in Muhammadiyah University of Surakarta in academic year 2017/2018. The selected students consisted of 6 students with details of two students in each intrapersonal intelligence category. The research instruments were questionnaire, open start problem solving task, and interview guidelines. Data validity used time triangulation. Data analyses were done through data collection, data reduction, data presentation, and drawing conclusion. Based on findings, subjects with strong intrapersonal intelligence had high self confidence that they were able to solve problem correctly, able to do planning steps and able to solve the problem appropriately. Subjects with average intrapersonal intelligence had high self-assessment that they were able to solve the problem, able to do planning steps appropriately but they had not maximized in carrying out the plan so that it resulted incorrectness answer. Subjects with weak intrapersonal intelligence had high self confidence in capability of solving math problem, lack of precision in taking plans so their task results incorrectness answer.

  15. Specifying theories of developmental dyslexia: a diffusion model analysis of word recognition

    NARCIS (Netherlands)

    Zeguers, M.H.T.; Snellings, P.; Tijms, J.; Weeda, W.D.; Tamboer, P.; Bexkens, A.; Huizenga, H.M.

    2011-01-01

    The nature of word recognition difficulties in developmental dyslexia is still a topic of controversy. We investigated the contribution of phonological processing deficits and uncertainty to the word recognition difficulties of dyslexic children by mathematical diffusion modeling of visual and

  16. There is More to the Teaching and Learning of Mathematics Than the Use of Local Languages: Mathematics Teacher Practices

    Directory of Open Access Journals (Sweden)

    Nancy Chitera

    2016-11-01

    Full Text Available In this article, we present a discussion about the type of mathematical discourse that is being produced in classrooms where the language of learning and teaching is local languages.  We also further explore the tensions in the mathematical discourse being produced. The study sample was 4 mathematics teachers from a semi-urban primary school in Malawi. The methods of data collection included classroom observations, pre-observation focus group discussions and reflective interviews. The results show that even though both students and teachers were able to communicate freely in local languages in the mathematics classroom, the mathematical discourse that came was distorted. This is mainly caused by lack of a well-developed mathematical discourse in local languages, which in turn takes away the confidence of mathematics teachers in the classroom. As a result, the mathematics classrooms are still being characterized by teachers not being creative, use of word by word from books, focus more on procedural than conceptual and thus teacher centered is still dominant in these classrooms. Furthermore, it is found that there are tensions between the formal and informal mathematical language in local languages. These results in turn have promoted a more in-depth understanding to the teaching and learning of mathematics when local language is the language of learning and teaching. Therefore, this article argues for a well-balanced approach when it comes to teaching and learning of mathematics rather than just focusing on the use of local languages.

  17. Enabling Metacognitive Skills for Mathematics Problem Solving: A Collective Case Study of Metacognitive Reflection and Awareness

    Science.gov (United States)

    Jagals, Divan; van der Walt, Marthie

    2016-01-01

    Metacognition encompasses knowledge and regulation that, through reflection, sustain problem solving behaviour. How metacognitive awareness is constructed from reflection on metacognitive knowledge and regulation and how these reflections enable metacognitive skills for Mathematics problem solving remain unclear. Three secondary schools…

  18. Searching for Authentic Context in Designing PISA-like Mathematics Problem: From Indoor to Outdoor Field Experience

    Science.gov (United States)

    Siswono, T. Y. E.; Kohar, A. W.; Rosyidi, A. H.; Hartono, S.; Masriyah

    2018-01-01

    Designing problem like in PISA is known as a challenging activity for teachers particularly as the use of authentic context within that type of problem. This paper aims to describe the experiences of secondary mathematics teachers in designing PISA-like problems within an innovative training program focusing on building teachers’ understanding on the concept of mathematical literacy. The teachers were engaged in a set of problem-solving and problem-posing activities using PISA-based problem within indoor and outdoor field experiences. Within indoor field experience, the teachers worked collaboratively in groups on designing PISA-like problems with a given context through problem generation and reformulation techniques. Within outdoor field experience, they worked on designing PISA-like problems with self-chosen context from the place where the outdoor field experience took place. Our analysis indicates that there were improvements on the PISA-like problems designed by teachers based on its level use of context from indoor to outdoor experience. Also, the teachers were relatively successful with creating appropriate and motivating contexts by harnessing a variety of context consisting of personal, occupational, societal, and scientific contexts. However, they still experienced difficulties in turning these contexts into an appropriate problem satisfying PISA framework such as regarding authenticity of context use, language structure, and PISA task profile.

  19. Hands-On Mathematics: Two Cases from Ancient Chinese Mathematics

    Science.gov (United States)

    Wang, Youjun

    2009-01-01

    In modern mathematical teaching, it has become increasingly emphasized that mathematical knowledge should be taught by problem-solving, hands-on activities, and interactive learning experiences. Comparing the ideas of modern mathematical education with the development of ancient Chinese mathematics, we find that the history of mathematics in…

  20. High profile students’ growth of mathematical understanding in solving linier programing problems

    Science.gov (United States)

    Utomo; Kusmayadi, TA; Pramudya, I.

    2018-04-01

    Linear program has an important role in human’s life. This linear program is learned in senior high school and college levels. This material is applied in economy, transportation, military and others. Therefore, mastering linear program is useful for provision of life. This research describes a growth of mathematical understanding in solving linear programming problems based on the growth of understanding by the Piere-Kieren model. Thus, this research used qualitative approach. The subjects were students of grade XI in Salatiga city. The subjects of this study were two students who had high profiles. The researcher generally chose the subjects based on the growth of understanding from a test result in the classroom; the mark from the prerequisite material was ≥ 75. Both of the subjects were interviewed by the researcher to know the students’ growth of mathematical understanding in solving linear programming problems. The finding of this research showed that the subjects often folding back to the primitive knowing level to go forward to the next level. It happened because the subjects’ primitive understanding was not comprehensive.

  1. The Elementary School Students’ Mathematical Problem Solving Based on Reading Abilities

    Science.gov (United States)

    Wulandari, R. D.; Lukito, A.; Khabibah, S.

    2018-01-01

    The aim of this research is to describe the third grade of elementary school students’ mathematical problem in solving skills based on their reading abilities. This research is a descriptive research with qualitative approach. This research was conducted at elementary school Kebraon II Surabaya in second semester of 2016-2017 academic years. The participants of this research consist of third grade students with different reading abilities that are independent level, instructional level and frustration level. The participants of this research were selected with purposive sampling technique. The data of this study were collected using reading the narration texts, the Ekwall and Shanker Informal Reading Inventory, problem solving task and interview guidelines. The collected data were evaluated using a descriptive analysis method. Once the study had been completed, it was concluded that problem solving skills varied according to reading abilities, student with independent level and instructional level can solve the problem and students with frustration level can’t solve the problem because they can’t interpret the problem well.

  2. Mathematical problem solving ability of sport students in the statistical study

    Science.gov (United States)

    Sari, E. F. P.; Zulkardi; Putri, R. I. I.

    2017-12-01

    This study aims to determine the problem-solving ability of sport students of PGRI Palembang semester V in the statistics course. Subjects in this study were sport students of PGRI Palembang semester V which amounted to 31 people. The research method used is quasi experiment type one case shoot study. Data collection techniques in this study use the test and data analysis used is quantitative descriptive statistics. The conclusion of this study shown that the mathematical problem solving ability of PGRI Palembang sport students of V semester in the statistical course is categorized well with the average of the final test score of 80.3.

  3. The role of mathematical models in understanding pattern formation in developmental biology.

    Science.gov (United States)

    Umulis, David M; Othmer, Hans G

    2015-05-01

    In a Wall Street Journal article published on April 5, 2013, E. O. Wilson attempted to make the case that biologists do not really need to learn any mathematics-whenever they run into difficulty with numerical issues, they can find a technician (aka mathematician) to help them out of their difficulty. He formalizes this in Wilsons Principle No. 1: "It is far easier for scientists to acquire needed collaboration from mathematicians and statisticians than it is for mathematicians and statisticians to find scientists able to make use of their equations." This reflects a complete misunderstanding of the role of mathematics in all sciences throughout history. To Wilson, mathematics is mere number crunching, but as Galileo said long ago, "The laws of Nature are written in the language of mathematics[Formula: see text] the symbols are triangles, circles and other geometrical figures, without whose help it is impossible to comprehend a single word." Mathematics has moved beyond the geometry-based model of Galileo's time, and in a rebuttal to Wilson, E. Frenkel has pointed out the role of mathematics in synthesizing the general principles in science (Both point and counter-point are available in Wilson and Frenkel in Notices Am Math Soc 60(7):837-838, 2013). We will take this a step further and show how mathematics has been used to make new and experimentally verified discoveries in developmental biology and how mathematics is essential for understanding a problem that has puzzled experimentalists for decades-that of how organisms can scale in size. Mathematical analysis alone cannot "solve" these problems since the validation lies at the molecular level, but conversely, a growing number of questions in biology cannot be solved without mathematical analysis and modeling. Herein, we discuss a few examples of the productive intercourse between mathematics and biology.

  4. The solution of the sixth Hilbert problem: the ultimate Galilean revolution

    Science.gov (United States)

    D'Ariano, Giacomo Mauro

    2018-04-01

    I argue for a full mathematization of the physical theory, including its axioms, which must contain no physical primitives. In provocative words: `physics from no physics'. Although this may seem an oxymoron, it is the royal road to keep complete logical coherence, hence falsifiability of the theory. For such a purely mathematical theory the physical connotation must pertain only the interpretation of the mathematics, ranging from the axioms to the final theorems. On the contrary, the postulates of the two current major physical theories either do not have physical interpretation (as for von Neumann's axioms for quantum theory), or contain physical primitives as `clock', `rigid rod', `force', `inertial mass' (as for special relativity and mechanics). A purely mathematical theory as proposed here, though with limited (but relentlessly growing) domain of applicability, will have the eternal validity of mathematical truth. It will be a theory on which natural sciences can firmly rely. Such kind of theory is what I consider to be the solution of the sixth Hilbert problem. I argue that a prototype example of such a mathematical theory is provided by the novel algorithmic paradigm for physics, as in the recent information-theoretical derivation of quantum theory and free quantum field theory. This article is part of the theme issue `Hilbert's sixth problem'.

  5. Evaluation of the Effect of Mathematical Routines on the Development of Skills in Mathematical Problem Solving and School Motivation of Primary School Students in Abitibi-Témiscamingue

    Science.gov (United States)

    Rajotte, Thomas; Marcotte, Christine; Bureau-Levasseur, Lisa

    2016-01-01

    In recent decades, the dropout rate in Abitibi-Témiscamingue is a worrying phenomenon. An analysis of ministerial examination results identifies that students in Abitibi-Témiscamingue have specific difficulties with mathematical problem solving tasks. Among the activities that develop those skills, the daily routines in mathematics seem to be a…

  6. Introductory discrete mathematics

    CERN Document Server

    Balakrishnan, V K

    2010-01-01

    This concise text offers an introduction to discrete mathematics for undergraduate students in computer science and mathematics. Mathematics educators consider it vital that their students be exposed to a course in discrete methods that introduces them to combinatorial mathematics and to algebraic and logical structures focusing on the interplay between computer science and mathematics. The present volume emphasizes combinatorics, graph theory with applications to some stand network optimization problems, and algorithms to solve these problems.Chapters 0-3 cover fundamental operations involv

  7. Gender differences in prospective teachers’ mathematical literacy: problem solving of occupational context on shipping company

    Science.gov (United States)

    Lestari, N. D. S.; Juniati, D.; Suwarsono, St.

    2018-04-01

    The purpose of this paper is to describe to what extent the prospective teachers can be considered as mathematically literate and how they communicate their reasoning in solving the problem based on the sex differences. Data were collected through mathematics literacy test on occupational context by 157 of prospective teachers from three universities in East Java, Indonesia. Their written responses were collected, organized based on the sex differences, analyzed and categorized to one of three levels of mathematical literacy. The examples of interesting students’ response altogether with the scoring are discussed to describe their characteristic on mathematical literacy and their communication. The result showed that in general the mathematical literacy of female prospective teachers tend to be better than male prospective math teachers. Female prospective teachers are more capable of logical reasoning, using concepts, facts and procedures and algebraic operations to draw conclusions; make an interpretations and evaluations. This study has an implication that gender differences in mathematical literacy of prospective math teachers do exist, therefore this issue should be given a serious concern from the development programs of the faculty.

  8. Cognitive mechanisms underlying third graders' arithmetic skills: Expanding the pathways to mathematics model.

    Science.gov (United States)

    Träff, Ulf; Olsson, Linda; Skagerlund, Kenny; Östergren, Rickard

    2018-03-01

    A modified pathways to mathematics model was used to examine the cognitive mechanisms underlying arithmetic skills in third graders. A total of 269 children were assessed on tasks tapping the four pathways and arithmetic skills. A path analysis showed that symbolic number processing was directly supported by the linguistic and approximate quantitative pathways. The direct contribution from the four pathways to arithmetic proficiency varied; the linguistic pathway supported single-digit arithmetic and word problem solving, whereas the approximate quantitative pathway supported only multi-digit calculation. The spatial processing and verbal working memory pathways supported only arithmetic word problem solving. The notion of hierarchical levels of arithmetic was supported by the results, and the different levels were supported by different constellations of pathways. However, the strongest support to the hierarchical levels of arithmetic were provided by the proximal arithmetic skills. Copyright © 2017 Elsevier Inc. All rights reserved.

  9. Multi-objective optimization problems concepts and self-adaptive parameters with mathematical and engineering applications

    CERN Document Server

    Lobato, Fran Sérgio

    2017-01-01

    This book is aimed at undergraduate and graduate students in applied mathematics or computer science, as a tool for solving real-world design problems. The present work covers fundamentals in multi-objective optimization and applications in mathematical and engineering system design using a new optimization strategy, namely the Self-Adaptive Multi-objective Optimization Differential Evolution (SA-MODE) algorithm. This strategy is proposed in order to reduce the number of evaluations of the objective function through dynamic update of canonical Differential Evolution parameters (population size, crossover probability and perturbation rate). The methodology is applied to solve mathematical functions considering test cases from the literature and various engineering systems design, such as cantilevered beam design, biochemical reactor, crystallization process, machine tool spindle design, rotary dryer design, among others.

  10. Finite Mathematics and Discrete Mathematics: Is There a Difference?

    Science.gov (United States)

    Johnson, Marvin L.

    Discrete mathematics and finite mathematics differ in a number of ways. First, finite mathematics has a longer history and is therefore more stable in terms of course content. Finite mathematics courses emphasize certain particular mathematical tools which are useful in solving the problems of business and the social sciences. Discrete mathematics…

  11. Critical Thinking Skills of an Eighth Grade Male Student with High Mathematical Ability in Solving Problem

    Science.gov (United States)

    Ismail

    2018-01-01

    This study aims to describe student’s critical thinking skill of grade VIII in solving mathematical problem. A qualitative research was conducted to a male student with high mathematical ability. Student’s critical thinking skill was obtained from a depth task-based interview. The result show that male student’s critical thinking skill of the student as follows. In understanding the problem, the student did categorization, significance decoding, and meaning clarification. In devising a plan he examined his ideas, detected his argument, analyzed his argument and evaluated his argument. During the implementation phase, the skill that appeared were analyzing of the argument and inference skill such as drawing conclusion, deliver alternative thinking, and problem solving skills. At last, in rechecking all the measures, they did self-correcting and self-examination.

  12. How Readability and Topic Incidence Relate to Performance on Mathematics Story Problems in Computer-Based Curricula

    Science.gov (United States)

    Walkington, Candace; Clinton, Virginia; Ritter, Steven N.; Nathan, Mitchell J.

    2015-01-01

    Solving mathematics story problems requires text comprehension skills. However, previous studies have found few connections between traditional measures of text readability and performance on story problems. We hypothesized that recently developed measures of readability and topic incidence measured by text-mining tools may illuminate associations…

  13. Words and possible words in early language acquisition.

    Science.gov (United States)

    Marchetto, Erika; Bonatti, Luca L

    2013-11-01

    In order to acquire language, infants must extract its building blocks-words-and master the rules governing their legal combinations from speech. These two problems are not independent, however: words also have internal structure. Thus, infants must extract two kinds of information from the same speech input. They must find the actual words of their language. Furthermore, they must identify its possible words, that is, the sequences of sounds that, being morphologically well formed, could be words. Here, we show that infants' sensitivity to possible words appears to be more primitive and fundamental than their ability to find actual words. We expose 12- and 18-month-old infants to an artificial language containing a conflict between statistically coherent and structurally coherent items. We show that 18-month-olds can extract possible words when the familiarization stream contains marks of segmentation, but cannot do so when the stream is continuous. Yet, they can find actual words from a continuous stream by computing statistical relationships among syllables. By contrast, 12-month-olds can find possible words when familiarized with a segmented stream, but seem unable to extract statistically coherent items from a continuous stream that contains minimal conflicts between statistical and structural information. These results suggest that sensitivity to word structure is in place earlier than the ability to analyze distributional information. The ability to compute nontrivial statistical relationships becomes fully effective relatively late in development, when infants have already acquired a considerable amount of linguistic knowledge. Thus, mechanisms for structure extraction that do not rely on extensive sampling of the input are likely to have a much larger role in language acquisition than general-purpose statistical abilities. Copyright © 2013. Published by Elsevier Inc.

  14. Circles Disturbed The Interplay of Mathematics and Narrative

    CERN Document Server

    Doxiadis, Apostolos

    2012-01-01

    Circles Disturbed brings together important thinkers in mathematics, history, and philosophy to explore the relationship between mathematics and narrative. The book's title recalls the last words of the great Greek mathematician Archimedes before he was slain by a Roman soldier--"Don't disturb my circles"--words that seem to refer to two radically different concerns: that of the practical person living in the concrete world of reality, and that of the theoretician lost in a world of abstraction. Stories and theorems are, in a sense, the natural languages of these two worlds--stories represent

  15. Utilizing geogebra in financial mathematics problems: didactic experiment in vocational college

    Science.gov (United States)

    Ghozi, Saiful; Yuniarti, Suci

    2017-12-01

    GeoGebra application offers users to solve real problems in geometry, statistics, and algebra fields. This studydeterminesthe effect of utilizing Geogebra on students understanding skill in the field of financial mathematics. This didactic experiment study used pre-test-post-test control group design. Population of this study were vocational college students in Banking and Finance Program of Balikpapan State Polytechnic. Two classes in the first semester were chosen using cluster random sampling technique, one class as experiment group and one class as control group. Data were analysed used independent sample t-test. The result of data analysis showed that students understanding skill with learning by utilizing GeoGeobra is better than students understanding skill with conventional learning. This result supported that utilizing GeoGebra in learning can assist the students to enhance their ability and depth understanding on mathematics subject.

  16. Personalized Computer-Assisted Mathematics Problem-Solving Program and Its Impact on Taiwanese Students

    Science.gov (United States)

    Chen, Chiu-Jung; Liu, Pei-Lin

    2007-01-01

    This study evaluated the effects of a personalized computer-assisted mathematics problem-solving program on the performance and attitude of Taiwanese fourth grade students. The purpose of this study was to determine whether the personalized computer-assisted program improved student performance and attitude over the nonpersonalized program.…

  17. Problem Solving Strategies of Selected Pre-Service Secondary School Mathematics Teachers in Malaysia

    Science.gov (United States)

    Yew, Wun Theam; Zamri, Sharifah Norul Akmar Syed

    2016-01-01

    Problem solving strategies of eight pre-service secondary school mathematics teachers (PSSMTs) were examined in this study. A case study research design was employed and clinical interview technique was used to collect the data. Materials collected for analysis consisted of audiotapes and videotapes of clinical interviews, subjects' notes and…

  18. Mathematics Education for Engineering Technology Students – A Bridge Too Far?

    Directory of Open Access Journals (Sweden)

    Noraishiyah Abdullah

    2013-03-01

    Full Text Available Trying to decide what is best suited for someone or something is an ever enduring task let alone trying to prepare students with the right engineering mind. So ‘how do you build an engineer?’ if that is the right word. What is the right ingredient? Mathematics has been said as the most important foundation in engineers’ life. Curriculum has been developed and reviewed over the years to meet this target. This work explores how much or lack of it has the curriculum prepares the future technologist to face the world of engineering technology as far as mathematics is concerned. Analysis of mathematics lectures, interviews of engineering technologist students and engineering technology subject lecturer is undertaken. Understand what each contributes help in understanding the picture that the current education is painting. Based on the theory of learning, APOS theory helps in explaining how students bridge their knowledge of mathematics when it comes to solving engineering technology problems. The question is, is it a bridge too far? 

  19. Effects of Intervention to Improve At-Risk Fourth Graders' Understanding, Calculations, and Word Problems with Fractions

    Science.gov (United States)

    Fuchs, Lynn S.; Schumacher, Robin F.; Long, Jessica; Namkung, Jessica; Malone, Amelia S.; Wang, Amber; Hamlett, Carol L.; Jordan, Nancy C.; Siegler, Robert S.; Changas, Paul

    2016-01-01

    The purposes of this study were to (a) investigate the efficacy of a core fraction intervention program on understanding and calculation skill and (b) isolate the effects of different forms of fraction word-problem (WP) intervention delivered as part of the larger program. At-risk 4th graders (n = 213) were randomly assigned at the individual…

  20. Effects of Intervention to Improve At-Risk Fourth Graders' Understanding, Calculations, and Word Problems with Fractions

    Science.gov (United States)

    Fuchs, Lynn S.; Schumacher, Robin F.; Long, Jessica; Namkung, Jessica; Malone, Amelia S.; Wang, Amber; Hamlett, Carol L.; Jordan, Nancy C.; Siegler, Robert S.; Changas, Paul

    2016-01-01

    The purposes of this study were to (a) investigate the efficacy of a core fraction intervention program on understanding and calculation skill and (b) isolate the effects of different forms of fraction word-problem (WP) intervention. At-risk fourth graders (n = 213) were randomly assigned to the school's business-as-usual program, or one of two…

  1. Comprehension and computation in Bayesian problem solving

    Directory of Open Access Journals (Sweden)

    Eric D. Johnson

    2015-07-01

    Full Text Available Humans have long been characterized as poor probabilistic reasoners when presented with explicit numerical information. Bayesian word problems provide a well-known example of this, where even highly educated and cognitively skilled individuals fail to adhere to mathematical norms. It is widely agreed that natural frequencies can facilitate Bayesian reasoning relative to normalized formats (e.g. probabilities, percentages, both by clarifying logical set-subset relations and by simplifying numerical calculations. Nevertheless, between-study performance on transparent Bayesian problems varies widely, and generally remains rather unimpressive. We suggest there has been an over-focus on this representational facilitator (i.e. transparent problem structures at the expense of the specific logical and numerical processing requirements and the corresponding individual abilities and skills necessary for providing Bayesian-like output given specific verbal and numerical input. We further suggest that understanding this task-individual pair could benefit from considerations from the literature on mathematical cognition, which emphasizes text comprehension and problem solving, along with contributions of online executive working memory, metacognitive regulation, and relevant stored knowledge and skills. We conclude by offering avenues for future research aimed at identifying the stages in problem solving at which correct versus incorrect reasoners depart, and how individual difference might influence this time point.

  2. Mathematical visualization process of junior high school students in solving a contextual problem based on cognitive style

    Science.gov (United States)

    Utomo, Edy Setiyo; Juniati, Dwi; Siswono, Tatag Yuli Eko

    2017-08-01

    The aim of this research was to describe the mathematical visualization process of Junior High School students in solving contextual problems based on cognitive style. Mathematical visualization process in this research was seen from aspects of image generation, image inspection, image scanning, and image transformation. The research subject was the students in the eighth grade based on GEFT test (Group Embedded Figures Test) adopted from Within to determining the category of cognitive style owned by the students namely field independent or field dependent and communicative. The data collection was through visualization test in contextual problem and interview. The validity was seen through time triangulation. The data analysis referred to the aspect of mathematical visualization through steps of categorization, reduction, discussion, and conclusion. The results showed that field-independent and field-dependent subjects were difference in responding to contextual problems. The field-independent subject presented in the form of 2D and 3D, while the field-dependent subject presented in the form of 3D. Both of the subjects had different perception to see the swimming pool. The field-independent subject saw from the top, while the field-dependent subject from the side. The field-independent subject chose to use partition-object strategy, while the field-dependent subject chose to use general-object strategy. Both the subjects did transformation in an object rotation to get the solution. This research is reference to mathematical curriculum developers of Junior High School in Indonesia. Besides, teacher could develop the students' mathematical visualization by using technology media or software, such as geogebra, portable cabri in learning.

  3. The Problem of the Object of Mathematics as Intelligible Substance in Aristotle's Metaphysics

    OpenAIRE

    Cattanei, Elisabetta

    2013-01-01

    The A. examines the problem of intermediat emathematical entities by analyzing Metaphysics l017a9-l4, since, according to Aristotle. this passage is both a source and a critique of Plato's theory. The goal is to identify four cardinal points that may ground a dialogue between two contesting positions regarding this problem. Through them, it becomes evident that Aristotle severs the question of the intelligible nature of mathematical entities by using the conceptual scalpel of his own ousiolog...

  4. The Analysis of Proportional Reasoning Problem in the Indonesian Mathematics Textbook for Junior High School

    Directory of Open Access Journals (Sweden)

    Rahmah Johar

    2017-06-01

    Full Text Available The lack of Indonesian students achievement in the international assessment is due to several factors. Students are not familiar with the problems requiring reasoning, in particular the proportional reasoning. This research aims to identify the distribution and the Level of Cognitive Demands (LCD of the proportional reasoning problems found in the Year 7 and Year 8 mathematics textbooks based on the 2013 curriculum (revised edition 2014. The data collection was conducted by identifying the proportional reasoning problems found in the whole chapters of the textbooks which are then analysed and classified using the Smiths and Stein’s criteria of LCD (1998. The results reveal that the proportional reasoning problems were only found in the three of 17 chapters namely ratio and proportion, rectangle and triangle, and Pythagorean Theorem, which represent different LCD including Lower-LCD (Low-M and Low-P and Higher-LCD (High-P. Out of 69 proportional reasoning problem found in the textbooks, the percentage of higher-LCD problems (n=29 ; 42.03% is less than lower-LCD (n=40;57.97%. In addition, the higher-LCD problems found were only the high-P type. None was found to meet the requirement of High-DM demanding students to conduct ‘doing mathematics’, complex approach and self-monitoring or self regulation of students’ cognitive process. It is recommended that the proportional reasoning problems, including some High-DM problems, should be provided in each topic in Indonesian mathematics textbooks.

  5. Doing physics with scientific notebook a problem solving approach

    CERN Document Server

    Gallant, Joseph

    2012-01-01

    The goal of this book is to teach undergraduate students how to use Scientific Notebook (SNB) to solve physics problems. SNB software combines word processing and mathematics in standard notation with the power of symbolic computation. As its name implies, SNB can be used as a notebook in which students set up a math or science problem, write and solve equations, and analyze and discuss their results. Written by a physics teacher with over 20 years experience, this text includes topics that have educational value, fit within the typical physics curriculum, and show the benefits of using SNB.

  6. Mathematics for the liberal arts

    CERN Document Server

    Brown, Jason I

    2014-01-01

    The Math in Your Life Health, Safety, and Mathematics Found in Translation The Essentials of Conversion Making Sense of Your World with Statistics Summarizing Data with a Few Good Numbers Estimating Unknowns Leading You Down the Garden Path with Statistics Visualizing with Mathematics Seeing Data A Graph Is Worth a Thousand Words Money and Risk Money - Now or Later Risk Taking and Probability The Life in Your Math! Deciding to Make the Best Decisions Making the Right Choices for You Game Theory - Coming Out on Top Making Joint Decisions Art Imitating Math The Math that Makes the Art Believing What You See (or Not) The Mathematics of Sound (and the Sound of Mathematics) The Mathematics of Listening The Mathematics of Composing Solving Musical Mysteries with MSI (Math Scene Investigations) Late Night Mathematics - Humor and Philosophy Laughing with Mathematics The Limits of Mathematics Bibliography Index Review questions appear at the end of each chapter.

  7. FORMULATION OF MATHEMATICAL PROBLEM DESCRIBING PHYSICAL AND CHEMICAL PROCESSES AT CONCRETE CORROSION

    Directory of Open Access Journals (Sweden)

    Sergey V. Fedosov

    2017-06-01

    Full Text Available The article deals with the relevance of new scientific research focused on modeling of physical and chemical processes occurring in the cement concrete at their exploitation. The basic types of concrete corrosion are described. The problem of mass transfer processes in a flat reinforced concrete wall at concrete corrosion of the first and the second types has been mathematically formulated.

  8. "Seeing It on the Screen Isn't Really Seeing It": Reading Problems of Writers Using Word Processing.

    Science.gov (United States)

    Haas, Christina

    An observational study examined computer writers' use of hard copy for reading. The study begins with a description, based on interviews, of four kinds of reading problems encountered by writers using word processing; formatting, proofreading, reorganizing, and critical reading ("getting a sense of the text"). Subjects, six freshmen…

  9. Degrees of infinite words, polynomials and atoms

    NARCIS (Netherlands)

    J. Endrullis; J. Karhumaki; J.W. Klop (Jan Willem); A. Saarela

    2016-01-01

    textabstractOur objects of study are finite state transducers and their power for transforming infinite words. Infinite sequences of symbols are of paramount importance in a wide range of fields, from formal languages to pure mathematics and physics. While finite automata for recognising and

  10. Degrees of infinite words, polynomials and atoms

    NARCIS (Netherlands)

    Endrullis, Jörg; Karhumäki, Juhani; Klop, Jan Willem; Saarela, Aleksi

    2016-01-01

    Our objects of study are finite state transducers and their power for transforming infinite words. Infinite sequences of symbols are of paramount importance in a wide range of fields, from formal languages to pure mathematics and physics. While finite automata for recognising and transforming

  11. The Effects of Group Monitoring on Fatigue-Related Einstellung during Mathematical Problem Solving

    Science.gov (United States)

    Frings, Daniel

    2011-01-01

    Fatigue resulting from sleep deficit can lead to decreased performance in a variety of cognitive domains and can result in potentially serious accidents. The present study aimed to test whether fatigue leads to increased Einstellung (low levels of cognitive flexibility) in a series of mathematical problem-solving tasks. Many situations involving…

  12. ASSESSING CONCEPTUAL UNDERSTANDING IN MATHEMATICS: Using Derivative Function to Solve Connected Problems

    Directory of Open Access Journals (Sweden)

    Nevin ORHUN

    2013-07-01

    Full Text Available Open and distance education plays an important role in the actualization of cultural goals as well as in societal developments. This is an independent teaching and learning method for mathematics which forms the dynamic of scientific thinking. Distance education is an important alternative to traditional teaching applications. These contributions brought by technology enable students to participate actively in having access to information and questioning it. Such an application increases students’ motivation and teaches how mathematics can be used in daily life. Derivative is a mathematical concept which can be used in many areas of daily life. The aim of this study is to enable the concept of derivatives to be understood well by using the derivative function in the solution of various problems. It also aims at interpreting difficulties theoretically in the solution of problems and determining mistakes in terms of teaching methods. In this study, how various aspects of derivatives are understood is emphasized. These aspects concern the explanation of concepts and process, and also their application to certain concepts in physics. Students’ depth of understanding of derivatives was analyzed based on two aspects of understanding; theoretical analysis and contextual application. Follow-up interviews were conducted with five students. The results show that the students preferred to apply an algebraic symbolic aspect instead of using logical meanings of function and its derivative. In addition, in relation to how the graph of the derivative function affects the aspect of function, it was determined that the students displayed low performance.

  13. Learning higher mathematics

    CERN Document Server

    Pontrjagin, Lev Semenovič

    1984-01-01

    Lev Semenovic Pontrjagin (1908) is one of the outstanding figures in 20th century mathematics. In a long career he has made fundamental con­ tributions to many branches of mathematics, both pure and applied. He has received every honor that a grateful government can bestow. Though in no way constrained to do so, he has through the years taught mathematics courses at Moscow State University. In the year 1975 he set himself the task of writing a series of books on secondary school and beginning university mathematics. In his own words, "I wished to set forth the foundations of higher mathematics in a form that would have been accessible to myself as a lad, but making use of all my experience as a scientist and a teacher, ac­ cumulated over many years. " The present volume is a translation of the first two out of four moderately sized volumes on this theme planned by Pro­ fessor Pontrjagin. The book begins at the beginning of modern mathematics, analytic ge­ ometry in the plane and 3-dimensional space. Refin...

  14. Comparing different kinds of words and word-word relations to test an habituation model of priming.

    Science.gov (United States)

    Rieth, Cory A; Huber, David E

    2017-06-01

    Huber and O'Reilly (2003) proposed that neural habituation exists to solve a temporal parsing problem, minimizing blending between one word and the next when words are visually presented in rapid succession. They developed a neural dynamics habituation model, explaining the finding that short duration primes produce positive priming whereas long duration primes produce negative repetition priming. The model contains three layers of processing, including a visual input layer, an orthographic layer, and a lexical-semantic layer. The predicted effect of prime duration depends both on this assumed representational hierarchy and the assumption that synaptic depression underlies habituation. The current study tested these assumptions by comparing different kinds of words (e.g., words versus non-words) and different kinds of word-word relations (e.g., associative versus repetition). For each experiment, the predictions of the original model were compared to an alternative model with different representational assumptions. Experiment 1 confirmed the prediction that non-words and inverted words require longer prime durations to eliminate positive repetition priming (i.e., a slower transition from positive to negative priming). Experiment 2 confirmed the prediction that associative priming increases and then decreases with increasing prime duration, but remains positive even with long duration primes. Experiment 3 replicated the effects of repetition and associative priming using a within-subjects design and combined these effects by examining target words that were expected to repeat (e.g., viewing the target word 'BACK' after the prime phrase 'back to'). These results support the originally assumed representational hierarchy and more generally the role of habituation in temporal parsing and priming. Copyright © 2017 Elsevier Inc. All rights reserved.

  15. The main problem solving differences between high school and university in mathematical beliefs and professional behavior

    Directory of Open Access Journals (Sweden)

    Reza Akhlaghi Garmjani

    2016-10-01

    Full Text Available Teaching science and math has been underdeveloped in nurturing the talents and motivations of young people who are in search of professions in these fields. Identifying and strengthening the students' problem solving beliefs and behaviors, can be a great help to those involved in teaching mathematics. This study investigates on the university and high school students, teachers and professors' problem solving beliefs and behaviors. Considering the research method, this study is a field research in which questionnaire is used. Participants in this research were senior high school and university students, math teachers and math professors. Data collection method for beliefs and behavior variables was via the use of a questionnaire. The Mann-Whitney test results showed that problem solving in high school and university was different and the main difference was in mathematical professional beliefs and behaviors.

  16. Understanding in mathematics

    CERN Document Server

    Sierpinska, Anna

    1994-01-01

    The concept of understanding in mathematics with regard to mathematics education is considered in this volume, the main problem for mathematics teachers being how to facilitate their students'' understanding of the mathematics being taught.

  17. Utilization of mathematics amongst healthcare students towards problem solving during their occupational safety health internship

    Science.gov (United States)

    Umasenan a/l Thanikasalam

    2017-05-01

    Occupational safety health is a multidisciplinary discipline concentrating on the safety, health and welfare of workers in the working place. Healthcare Students undergoing Occupational Safety Health internships are required to apply mathematical in areas such as safety legislation, safety behavior, ergonomics, chemical safety, OSH practices, industrial hygiene, risk management and safety health practices as problem solving. The aim of this paper is to investigate the level of mathematics and logic utilization from these students during their internship looking at areas of Hazard identification, Determining the population exposed to the hazard, Assessing the risk of the exposure to the hazards and Taking preventive and control. A total of 142 returning healthcare students from their Occupational Safety Health, internship were given a questionnaire to measure their perceptions towards mathematical and logic utilization. The overall results indicated a strong positive skewed result towards the use of Mathematics during their internship. The findings showed that mathematics were well delivered by the students during their internship. Mathematics could not be separated from OSH practice as a needed precision in quantifying safety, health an d welfare of workers in addition to empiricism.

  18. New Readings in Greek Mathematics: Sources, Problems, Publications.

    Science.gov (United States)

    Knorr, Wilbur R.

    1990-01-01

    The field of ancient Greek mathematics is discussed in terms of how representative is the surviving corpus of the ancient achievement in mathematics, the patterns of thought by which they were discovered, and the construction of mathematics during this period. The research being done in this field is described. (KR)

  19. Finding Words and Word Structure in Artificial Speech: The Development of Infants' Sensitivity to Morphosyntactic Regularities

    Science.gov (United States)

    Marchetto, Erika; Bonatti, Luca L.

    2015-01-01

    To achieve language proficiency, infants must find the building blocks of speech and master the rules governing their legal combinations. However, these problems are linked: words are also built according to rules. Here, we explored early morphosyntactic sensitivity by testing when and how infants could find either words or within-word structure…

  20. Teachers' Perceptions of Teaching Mathematics at the Senior Secondary Level in Fiji

    Science.gov (United States)

    Dayal, Hem Chand

    2013-01-01

    In recent times, there has been considerable interest shown in the affective domain of mathematics education with research findings pointing out that affective variables have profound impact on classroom practices of mathematics teachers. In other words, teachers' conceptions of mathematics and mathematics teaching are greatly influenced by…

  1. Examination of Gifted Students' Probability Problem Solving Process in Terms of Mathematical Thinking

    Science.gov (United States)

    Baltaci, Serdal

    2016-01-01

    It is a widely known fact that gifted students have different skills compared to their peers. However, to what extent gifted students use mathematical thinking skills during probability problem solving process emerges as a significant question. Thence, the main aim of the present study is to examine 8th grade gifted students' probability…

  2. Problem-Based Learning in K-8 Mathematics and Science Education: A Literature Review

    Science.gov (United States)

    Merritt, Joi; Lee, Mi Yeon; Rillero, Peter; Kinach, Barbara M.

    2017-01-01

    This systematic literature review was conducted to explore the effectiveness of problem-based and project-based learning (PBL) implemented with students in early elementary to grade 8 (ages 3-14) in mathematics and science classrooms. Nine studies met the following inclusion criteria: (a) focus on PBL, (b) experimental study, (c) kindergarten to…

  3. Evaluation of Students' Mathematical Problem Solving Skills in Relation to Their Reading Levels

    Science.gov (United States)

    Özsoy, Gökhan; Kuruyer, Hayriye Gül; Çakiroglu, Ahmet

    2015-01-01

    The purpose of the current study is to investigate the correlation between students' reading levels and mathematical problem solving skills. The present study was conducted in line with a qualitative research method, i.e., the phenomenological method. The study group of the current research is composed of six third grade students with different…

  4. Teacher Formation in the Mathematical Thinking through Problem Solving in the Second Phase of the CCyM Network of Reading Comprehension and Mathematics

    Directory of Open Access Journals (Sweden)

    LUZ STELLA LÓPEZ

    2008-12-01

    Full Text Available This article shares the design, implementation, and evaluation of theLesson Study process used for the professional development of teachers of mathematics, through the Red de Comprensión Lectora y Matemáticas – CCyM Network, in ways to teach mathematics through problem solving. The program began with a course on the implementation of the Thinking Classroom, followed by the semi-presencial Lesson Study process. An analysis of teacher interactions during the Lesson Study process yielded these categories of study: Group Collective Thinking, Mathematical Pedagogical Content Knowledge, Subject Matter Knowledge, Knowledge about Technology, and Expert Support. The analysis reflected variations in group interactions, in the command of concepts, in reflective practice, in the ability to make arguments and to propose changes in practice, and in the ability to self-regulate.

  5. Problem solving in the borderland between mathematics and physics

    DEFF Research Database (Denmark)

    Jensen, Jens Højgaard; Niss, Martin; Jankvist, Uffe Thomas

    2017-01-01

    The article addresses the problématique of where mathematization is taught in the educational system, and who teaches it. Mathematization is usually not a part of mathematics programs at the upper secondary level, but we argue that physics teaching has something to offer in this respect, if it fo......The article addresses the problématique of where mathematization is taught in the educational system, and who teaches it. Mathematization is usually not a part of mathematics programs at the upper secondary level, but we argue that physics teaching has something to offer in this respect...

  6. Does Cognitive Strategy Training on Word Problems Compensate for Working Memory Capacity in Children with Math Difficulties?

    Science.gov (United States)

    Swanson, H. Lee

    2014-01-01

    Cognitive strategies are important tools for children with math difficulties (MD) in learning to solve word problems. The effectiveness of strategy training, however, depends on working memory capacity (WMC). Thus, children with MD but with relatively higher WMC are more likely to benefit from strategy training, whereas children with lower WMC may…

  7. The solution of the sixth Hilbert problem: the ultimate Galilean revolution.

    Science.gov (United States)

    D'Ariano, Giacomo Mauro

    2018-04-28

    I argue for a full mathematization of the physical theory, including its axioms, which must contain no physical primitives. In provocative words: 'physics from no physics'. Although this may seem an oxymoron, it is the royal road to keep complete logical coherence, hence falsifiability of the theory. For such a purely mathematical theory the physical connotation must pertain only the interpretation of the mathematics, ranging from the axioms to the final theorems. On the contrary, the postulates of the two current major physical theories either do not have physical interpretation (as for von Neumann's axioms for quantum theory), or contain physical primitives as 'clock', 'rigid rod', 'force', 'inertial mass' (as for special relativity and mechanics). A purely mathematical theory as proposed here, though with limited (but relentlessly growing) domain of applicability, will have the eternal validity of mathematical truth. It will be a theory on which natural sciences can firmly rely. Such kind of theory is what I consider to be the solution of the sixth Hilbert problem. I argue that a prototype example of such a mathematical theory is provided by the novel algorithmic paradigm for physics, as in the recent information-theoretical derivation of quantum theory and free quantum field theory.This article is part of the theme issue 'Hilbert's sixth problem'. © 2018 The Author(s).

  8. Making Sense of Math: How to Help Every Student become a Mathematical Thinker and Problem Solver (ASCD Arias)

    Science.gov (United States)

    Seeley, Cathy L.

    2016-01-01

    In "Making Sense of Math," Cathy L. Seeley, former president of the National Council of Teachers of Mathematics, shares her insight into how to turn your students into flexible mathematical thinkers and problem solvers. This practical volume concentrates on the following areas: (1) Making sense of math by fostering habits of mind that…

  9. Examining mathematical discourse to understand in-service teachers’ mathematical activities

    Directory of Open Access Journals (Sweden)

    Margot Berger

    2013-04-01

    Full Text Available In this article I use Sfard’s theory of commognition to examine the surprising activities of a pair of in-service mathematics teachers in South Africa as they engaged in a particular mathematical task which allowed for, but did not prescribe, the use of GeoGebra. The (pre-calculus task required students to examine a function at an undefined point and to decide whether a vertical asymptote is associated with this point or not. Using the different characteristics of mathematical discourse, I argue that the words that students use really matter and show how a change in one participant’s use of the term ‘vertical asymptote’ constituted and reflected her learning. I also show how the other participant used imitation in a ritualised routine to get through the task. Furthermore I demonstrate how digital immigrants may resist the use of technology as the generator of legitimate mathematical objects.

  10. THE EFFECT OF A READING COMPREHENSION SOFTWARE PROGRAM ON STUDENT ACHIEVEMENT IN MATHEMATICS

    Directory of Open Access Journals (Sweden)

    David E. Proudfoot

    2016-06-01

    Full Text Available In an effort to increase student achievement, research was conducted to determine the degree in which a reading comprehension software program effected the reading and math abilities of fourth and fifth grade students. Cognitive and educational studies were examined to select a reading comprehension software program as an intervention that would produce positive results in reading comprehension and possibly transfer positive results to achievement in other academic areas, specifically in math. The effects of the intervention were measured by assigning subjects to an experimental group. The total sample consisted of 39 students who were deficient in reading comprehension, and also exposed a significant weakness with word problem items on mathematical assessments. Four instruments were used to collect data before and after the treatment to measure student achievement. To determine the degree to which the software program effected student achievement, data from the four instruments were analyzed using SPSS software. A paired-samples dependent t test and a Pearson Product Moment Correlation Coefficient was computed with ratio level data to test for a correlation between increased math scores and reading comprehension scores. Results yielded statistically significant and positive results in increasing reading comprehension skills that could possibly benefit students in reading and understanding mathematical problems. Results did not conclusively support that the increase of reading-comprehension skills had a collateral effect on students scoring higher with math word problems. The results are conducive to providing insight to educational leaders who plan to implement software as a means for increasing student achievement.

  11. On the Formal-Logical Analysis of the Foundations of Mathematics Applied to Problems in Physics

    Science.gov (United States)

    Kalanov, Temur Z.

    2016-03-01

    Analysis of the foundations of mathematics applied to problems in physics was proposed. The unity of formal logic and of rational dialectics is methodological basis of the analysis. It is shown that critical analysis of the concept of mathematical quantity - central concept of mathematics - leads to the following conclusion: (1) The concept of ``mathematical quantity'' is the result of the following mental operations: (a) abstraction of the ``quantitative determinacy of physical quantity'' from the ``physical quantity'' at that the ``quantitative determinacy of physical quantity'' is an independent object of thought; (b) abstraction of the ``amount (i.e., abstract number)'' from the ``quantitative determinacy of physical quantity'' at that the ``amount (i.e., abstract number)'' is an independent object of thought. In this case, unnamed, abstract numbers are the only sign of the ``mathematical quantity''. This sign is not an essential sign of the material objects. (2) The concept of mathematical quantity is meaningless, erroneous, and inadmissible concept in science because it represents the following formal-logical and dialectical-materialistic error: negation of the existence of the essential sign of the concept (i.e., negation of the existence of the essence of the concept) and negation of the existence of measure of material object.

  12. Meta-hierarchical-heuristic-mathematical- model of loading problems in flexible manufacturing system for development of an intelligent approach

    Directory of Open Access Journals (Sweden)

    Ranbir Singh

    2016-04-01

    Full Text Available Flexible manufacturing system (FMS promises a wide range of manufacturing benefits in terms of flexibility and productivity. These benefits are targeted by efficient production planning. Part type selection, machine grouping, deciding production ratio, resource allocation and machine loading are five identified production planning problems. Machine loading is the most identified complex problem solved with aid of computers. System up gradation and newer technology adoption are the primary needs of efficient FMS generating new scopes of research in the field. The literature review is carried and the critical analysis is being executed in the present work. This paper presents the outcomes of the mathematical modelling techniques for loading of machines in FMS’s. It was also analysed that the mathematical modelling is necessary for accurate and reliable analysis for practical applications. However, excessive computations need to be avoided and heuristics have to be used for real-world problems. This paper presents the heuristics-mathematical modelling of loading problem with machine processing time as primary input. The aim of the present work is to solve a real-world machine loading problem with an objective of balancing the workload of the FMS with decreased computational time. A Matlab code is developed for the solution and the results are found most accurate and reliable as presented in the paper.

  13. Students’ Relational Thinking of Impulsive and Reflective in Solving Mathematical Problem

    Science.gov (United States)

    Satriawan, M. A.; Budiarto, M. T.; Siswono, T. Y. E.

    2018-01-01

    This is a descriptive research which qualitatively investigates students’ relational thinking of impulsive and reflective cognitive style in solving mathematical problem. The method used in this research are test and interview. The data analyzed by reducing, presenting and concluding the data. The results of research show that the students’ reflective cognitive style can possibly help to find out important elements in understanding a problem. Reading more than one is useful to identify what is being questioned and write the information which is known, building relation in every element and connecting information with arithmetic operation, connecting between what is being questioned with known information, making equation model to find out the value by using substitution, and building a connection on re-checking, re-reading, and re-counting. The impulsive students’ cognitive style supports important elements in understanding problems, building a connection in every element, connecting information with arithmetic operation, building a relation about a problem comprehensively by connecting between what is being questioned with known information, finding out the unknown value by using arithmetic operation without making any equation model. The result of re-checking problem solving, impulsive student was only reading at glance without re-counting the result of problem solving.

  14. BioWord: A sequence manipulation suite for Microsoft Word

    Directory of Open Access Journals (Sweden)

    Anzaldi Laura J

    2012-06-01

    Full Text Available Abstract Background The ability to manipulate, edit and process DNA and protein sequences has rapidly become a necessary skill for practicing biologists across a wide swath of disciplines. In spite of this, most everyday sequence manipulation tools are distributed across several programs and web servers, sometimes requiring installation and typically involving frequent switching between applications. To address this problem, here we have developed BioWord, a macro-enabled self-installing template for Microsoft Word documents that integrates an extensive suite of DNA and protein sequence manipulation tools. Results BioWord is distributed as a single macro-enabled template that self-installs with a single click. After installation, BioWord will open as a tab in the Office ribbon. Biologists can then easily manipulate DNA and protein sequences using a familiar interface and minimize the need to switch between applications. Beyond simple sequence manipulation, BioWord integrates functionality ranging from dyad search and consensus logos to motif discovery and pair-wise alignment. Written in Visual Basic for Applications (VBA as an open source, object-oriented project, BioWord allows users with varying programming experience to expand and customize the program to better meet their own needs. Conclusions BioWord integrates a powerful set of tools for biological sequence manipulation within a handy, user-friendly tab in a widely used word processing software package. The use of a simple scripting language and an object-oriented scheme facilitates customization by users and provides a very accessible educational platform for introducing students to basic bioinformatics algorithms.

  15. BioWord: A sequence manipulation suite for Microsoft Word

    Science.gov (United States)

    2012-01-01

    Background The ability to manipulate, edit and process DNA and protein sequences has rapidly become a necessary skill for practicing biologists across a wide swath of disciplines. In spite of this, most everyday sequence manipulation tools are distributed across several programs and web servers, sometimes requiring installation and typically involving frequent switching between applications. To address this problem, here we have developed BioWord, a macro-enabled self-installing template for Microsoft Word documents that integrates an extensive suite of DNA and protein sequence manipulation tools. Results BioWord is distributed as a single macro-enabled template that self-installs with a single click. After installation, BioWord will open as a tab in the Office ribbon. Biologists can then easily manipulate DNA and protein sequences using a familiar interface and minimize the need to switch between applications. Beyond simple sequence manipulation, BioWord integrates functionality ranging from dyad search and consensus logos to motif discovery and pair-wise alignment. Written in Visual Basic for Applications (VBA) as an open source, object-oriented project, BioWord allows users with varying programming experience to expand and customize the program to better meet their own needs. Conclusions BioWord integrates a powerful set of tools for biological sequence manipulation within a handy, user-friendly tab in a widely used word processing software package. The use of a simple scripting language and an object-oriented scheme facilitates customization by users and provides a very accessible educational platform for introducing students to basic bioinformatics algorithms. PMID:22676326

  16. BioWord: a sequence manipulation suite for Microsoft Word.

    Science.gov (United States)

    Anzaldi, Laura J; Muñoz-Fernández, Daniel; Erill, Ivan

    2012-06-07

    The ability to manipulate, edit and process DNA and protein sequences has rapidly become a necessary skill for practicing biologists across a wide swath of disciplines. In spite of this, most everyday sequence manipulation tools are distributed across several programs and web servers, sometimes requiring installation and typically involving frequent switching between applications. To address this problem, here we have developed BioWord, a macro-enabled self-installing template for Microsoft Word documents that integrates an extensive suite of DNA and protein sequence manipulation tools. BioWord is distributed as a single macro-enabled template that self-installs with a single click. After installation, BioWord will open as a tab in the Office ribbon. Biologists can then easily manipulate DNA and protein sequences using a familiar interface and minimize the need to switch between applications. Beyond simple sequence manipulation, BioWord integrates functionality ranging from dyad search and consensus logos to motif discovery and pair-wise alignment. Written in Visual Basic for Applications (VBA) as an open source, object-oriented project, BioWord allows users with varying programming experience to expand and customize the program to better meet their own needs. BioWord integrates a powerful set of tools for biological sequence manipulation within a handy, user-friendly tab in a widely used word processing software package. The use of a simple scripting language and an object-oriented scheme facilitates customization by users and provides a very accessible educational platform for introducing students to basic bioinformatics algorithms.

  17. Does Early Algebraic Reasoning Differ as a Function of Students' Difficulty with Calculations versus Word Problems?

    Science.gov (United States)

    Powell, Sarah R.; Fuchs, Lynn S.

    2014-01-01

    According to national mathematics standards, algebra instruction should begin at kindergarten and continue through elementary school. Most often, teachers address algebra in the elementary grades with problems related to solving equations or understanding functions. With 789 second-grade students, we administered: (1) measures of calculations and…

  18. Mathematics for the imagination

    CERN Document Server

    Higgins, Peter

    2002-01-01

    Mathematics for the Imagination provides an accessible and entertaining investigation into mathematical problems in the world around us. From world navigation, family trees, and calendars to patterns, tessellations, and number tricks, this informative and fun new book helps you to understand the maths behind real-life questions and rediscover your arithmetical mind.This is a follow-up to the popular Mathematics for the Curious, Peter Higgins's first investigation into real-life mathematical problems.A highly involving book which encourages the reader to enter into the spirit of mathematical ex

  19. Exploring Teachers' Process of Change in Incorporating Problem Solving into the Mathematics Classroom

    Science.gov (United States)

    Rutherford, Vanessa

    2012-01-01

    This study explores how a problem-solving based professional learning community (PLC) affects the beliefs, knowledge, and instructional practices of two sixth-grade mathematics teachers. An interview and two observations were conducted prior to beginning the year-long PLC in order to gather information about the participants' beliefs,…

  20. Possibilities of mathematical models in solving flow problems in environmental protection and water architecture

    Energy Technology Data Exchange (ETDEWEB)

    1979-01-01

    The booklet presents the full text of 13 contributions to a Colloquium held at Karlsruhe in Sept. 1979. The main topics of the papers are the evaluation of mathematical models to solve flow problems in tide water, seas, rivers, groundwater and in the earth atmosphere. See further hints under relevant topics.